◆ わからない問題はここに書いてね 6 ◆

このエントリーをはてなブックマークに追加
1スレ引越しセンター

 γ∞γ~  \   / ̄ ̄ ̄ ̄ ̄ ̄ ̄ ̄ ̄ ̄ ̄ ̄ ̄ ̄ ̄ ̄ ̄
 人w/ 从从) )  わからない問題はここに書いてね♪
  ヽ | | l  l |〃 それから、数学記号の書き方の例は >>2
  `wハ~ ーノ)   その他・業務連絡は >>3 を読んでね♪
   / \`「   \_________________

【過去のスレッド】
http://cheese.2ch.net/test/read.cgi?bbs=math&key=967755172 その1
http://cheese.2ch.net/test/read.cgi?bbs=math&key=970795775 その2
http://cheese.2ch.net/test/read.cgi?bbs=math&key=974911042 その3
http://cheese.2ch.net/test/read.cgi?bbs=math&key=978209589 その4
http://cheese.2ch.net/test/read.cgi?bbs=math&key=981372834 その5
2スレ引越しセンター:2001/03/26(月) 17:11
【数学記号の書き方の例】

●変数・定数:x,y,z,..., a,b,c,... (← スカラー・ベクトルの区別は基本的にしない)
●関数:f(x), f[x]
●数列:a(n), a[n], a_n
●行列・テンソル:A, A(i,j), A[i,j], A[i,j,...;p,q...]
●足し算:a+b
●引き算:a-b
●掛け算:a*b, ab (← "*"を使い,"x"は使わない.)
●割り算・分数:a/b, a/(b+c), a/(bc) (← "/"を使い,"÷"は使わない.)
●平方根:√(a+b)=(a+b)^(1/2) (← "√"を使う.「るーと」で変換可)
●指数・指数関数:a^b, x^(n+1), exp(x+y)=e^(x+y) (← "^"を使う."exp"はeの指数.)
●対数・対数関数:log_a(b), log<a>b, log(x/2)=log_10(x/2), ln(x/2)=log_e(x/2) (← 底を省略する場合,"log"は常用対数,"ln"は自然対数.)
●三角比・三角関数:sin(a), cos(x+y), tan(x/2)
●内積・外積・スカラー3重積:a・b, axb, [a,b,c]
●行列式・トレース:|A|, det(A), tr(A)
●絶対値:|x|
●組合せ:nCk, C[n.k] (← P, Π, H も同様)
●微分・偏微分:y', dy/dx, ∂y/∂x (← "∂"は「きごう」で変換可)
●積分:∫[0,1]f(x)dx, ∫[y=0,x]f(x,y)dy
●数列和・数列積:Σ[k=1,n]a(k), Π[k=1,n]a(k)
●極限:lim[x->∞]f(x)

※その他"≠≧≦≒∈±≡∩"などは「きごう」を変換して使う.
3スレ引越しセンター:2001/03/26(月) 17:14
【その他・業務連絡】

●新スレへの移行
・900を超えたら新スレに移行準備
・旧スレの終了宣言と新スレへの誘導
・注意書きの変更

●注意書きの変更依頼
http://teri.2ch.net/test/read.cgi?bbs=accuse&key=974985294

●数学板の削除依頼
http://teri.2ch.net/test/read.cgi?bbs=saku&key=975411628
4嵐山堪三郎:2001/03/26(月) 17:21
さくらたんと、オマンコしたい!!!
5132人目の素数さん:2001/03/26(月) 17:28
>>1-3
ご苦労様です。
>>3のリンク先が落ちてます。現在使われているのはここです。

●注意書きの変更依頼
http://teri.2ch.net/test/read.cgi?bbs=accuse&key=981797747&ls=50

●数学板の削除依頼
http://teri.2ch.net/test/read.cgi?bbs=saku&key=974165593&ls=50
6サム:2001/03/26(月) 23:25
さっそく質問させてください
{(x+y)^7}-(x^7)-(y^7)を整数係数の範囲で因数分解せよ。
また
{(x+y)^n}-(x^n)-(y^n)が{(x^2)+(xy)+(y^2)}*(x..yの多項式)
の形に表される様な2以上の自然数nを小さい方から3つ求めよ。
7>6:2001/03/26(月) 23:37
Mathematica によると
{(x+y)^7}-(x^7)-(y^7) = 7xy(x+y){(x^2)+(xy)+(y^2)}^2
だそーです。
8サム:2001/03/26(月) 23:46
>>6
なるほど。
しかしMathematicaは反則ですな(笑
羨ましい。。
9f(n) = {(x+y)^n}-(x^n)-(y^n):2001/03/26(月) 23:49
>>6
f(5) = 5xy(x+y)(x^2+xy+y^2)
f(7) = 7xy(x+y)(x^2+xy+y^2)^2
f(11) = 11xy(x+y)(x^2+xy+y^2)(x^6+3x^5y+7x^4y^2+9x^3y^3+7x^2y^4+3xy^5+y^6)
10サム:2001/03/26(月) 23:52
>>9
レスありがとうございます。
この5.7.11という数はどうやって求めたのでしょうか?
1から全部入れてやっていくというわけではないですよね、、
11はむはむ:2001/03/27(火) 00:17
積分なのですが質問お願いいたします

f(x)は次数が1以上の整式とする。ある定数Cに対して、
等式∫[-x→x]{f(t+1)-f(t)}dt =cf(x)が任意の実数xで成立しているとする。
(1)f(x)の次数が3以下のとき、cおよびf(x)を求めよ

(2)cおよびf(x)を求めよ
12132人目の素数さん:2001/03/27(火) 00:28
         ∩
         //
        //
        |:| Λ_Λ  / ̄ ̄ ̄ ̄ ̄
        |:|( ´Α`)< ln(x^2+a^2)の不定積分を御願いします。
        |:|_):∵:(   \_____
        \:∵:∴:\
          |∴:∵ l::|
          |∵:∴ |::|
         /∴:∵/|::|
         |ii;∵;i/ |:|
         |llll||lll|  U
         |llll||lll|
         /ll/ |ll|
        /l/  |ll|
        /l/  |ll|
       /l/   |ll|
      ν    ν
13嘉郎:2001/03/27(火) 00:37
>>12
何について積分したいの?
14132人目の素数さん:2001/03/27(火) 00:43
>>13
nについてです。御願いします。
15嘉郎:2001/03/27(火) 00:54
∫(x^2+a^2)dn
⇔(x^2+a^2)n

16132人目の素数さん:2001/03/27(火) 00:56
ネタにマジレスをつけるというネタですか?
17132人目の素数さん:2001/03/27(火) 01:17
>>6
{(x+y)^n}-(x^n)-(y^n)が{(x^2)+(xy)+(y^2)}*(x..yの多項式)
の形に表される様な2以上の自然数nは
n=6k+1, n=6k-1 (kは自然数)でいいんじゃないかなあ。
1817:2001/03/27(火) 01:51
f_n(x,y)={(x+y)^n}-(x^n)-(y^n) とおく。
z^2 + z + 1 = 0 の根を w とおく。

f_n(x,y)が{(x^2)+(xy)+(y^2)}*(x..yの多項式)
の形に表されたとすると f_n(w,1)=0 である。
自然数 n に対して
「{(w+1)^n}-(w^n)-(1^n)=0」⇔「n=6k+1, n=6k-1(kは自然数)」
であることは複素平面を睨んでればわかる。
(注:w は1の原始3乗根、w+1 は1の原始6乗根。)

逆に n=6k+1 または n=6k-1(kは自然数)ならば
f_n(wy,y)=(y^n)f_n(w,1)=0
f_n((w^2)y,y)=(y^n)f_n(w^2,1)=0
だから f_n(x,y) は (x-wy)(x-(w^2)y)=(x^2)+xy+(y^2) を因子に持つ。

…証明は大体こんな感じだけど頭が半分寝てる状態で
書いたんで変なこと言ってるかもしれん。
19132人目の素数さん:2001/03/27(火) 01:51
>>6 >>17
((t+1)^n - t^n -1)が(t^2+t+1)で割り切れるようなnを探す。

(t^2+t+1)=0の解の一つはa=cos(π/3)+isin(π/3)
f(n,t)=((t+1)^n - t^n -1)とすると
f(n,a)
=(cos(π/6)+isin(π/6))^n - (cos(π/3)+isin(π/3))^n - 1
=(cos(nπ/6)-cos(nπ/3)-1) + i(sin(nπ/6)-sin(nπ/3))

cos(nπ/6)-cos(nπ/3)-1=0 }
sin(nπ/6)-sin(nπ/3)=0  } ⇒ 略 ⇒ n=6k±1

略したとこきぼーん。
和→積の公式なんて覚えてないっす。
20132人目の素数さん:2001/03/27(火) 04:13
>>11
f(x) が奇関数であることはすぐ分かる。

(1)は、f(x)=Ax^3+Bx と置いて計算すればよい。

(2)は、次数が3より大きい解があるとして、
f(x)=Ax^(2n+1)+Bx^(2n-1)+.....
と置いて計算してみればよい(A≠0,n≧2)。
x^(2n+1) と x^(2n-1) の係数を考えるだけで、あり得ないと分かる。
21132人目の素数さん:2001/03/27(火) 11:30
6,11は学コンだってさ。
22132人目の素数さん:2001/03/27(火) 13:00
マジ?>>21
汚ねぇ奴だな>>6, >>11
23132人目の素数さん:2001/03/27(火) 15:36
         ∩
         //
        //
        |:| Λ_Λ  / ̄ ̄ ̄ ̄ ̄
        |:|( ´Α`)< >>13 xについてです >>14 勝手に答えないで下さい
        |:|_):∵:(   \_____
        \:∵:∴:\
          |∴:∵ l::|
          |∵:∴ |::|
         /∴:∵/|::|
         |ii;∵;i/ |:|
         |llll||lll|  U
         |llll||lll|
         /ll/ |ll|
        /l/  |ll|
        /l/  |ll|
       /l/   |ll|
      ν    ν
24132人目の素数さん:2001/03/27(火) 16:30
交換留学生です。アメリカ人に馬鹿にされたくないのでどなたか教えてください。
よろしくお願いいたします。
Let M be an n by n matrix with independent identically distributed
entries Mij, where
P(Mij = ±1) = 1/2,
Determine Var(det(M)).
25132人目の素数さん:2001/03/27(火) 16:31
-2x+2a*arctan(x/a)+x*ln(x^2+a^2)+c
26132人目の素数さん:2001/03/27(火) 16:39
Consider the Markov chain Xn on S={0,1,2} with transition matrix:
P=[1/2 1/4 1/4; 1/2 0 1/2; 1/4 1/4 1/2]

1. Is the chain irreducible and aperiodic?
2. Is the chain reversible?
3. How many solutions π does the matrix equation πP=π have?
4. Compute: lim[n→∞]Po(Xn=1).
27132人目の素数さん:2001/03/27(火) 16:55
Let Xn, n≧0, be the 3-state Markov chain on S={0,1,2} with transition matrix
P=[1 0 0; 7/12 1/4 1/6; 5/6 1/6 0]
Show that
lim[n→∞](3^n)*P1(Xn=2) exists, and compute the limit.
28132人目の素数さん:2001/03/27(火) 17:29
         ∩
         //
        //
        |:| Λ_Λ  / ̄ ̄ ̄ ̄ ̄
        |:|( ´Α`)< >>23 解答ありがとう。逝って来ます
        |:|_):∵:(   \_____
        \:∵:∴:\
          |∴:∵ l::|
          |∵:∴ |::|
         /∴:∵/|::|
         |ii;∵;i/ |:|
         |llll||lll|  U
         |llll||lll|
         /ll/ |ll|
        /l/  |ll|
        /l/  |ll|
       /l/   |ll|
      ν    ν
29132人目の素数さん:2001/03/27(火) 22:04
x/3=y/5=z/6 のとき3x+y-3x/x-y+zの値を求めなさい。

これを姪(17歳)にわかりやすく解説するにはどうしたらいいでしょう?
自分の説明では「わからない」と言われた主婦です。
もう解き方忘れちゃいました……(T-T)よろしくお願いします。
30>29:2001/03/27(火) 22:12
x/3=y/5=z/6 = t とおく
t=0 のとき 分母0だから計算できない
t=0でないとき、全部の項をtだけで表す。
        計算しておわり
31132人目の素数さん:2001/03/27(火) 22:31
>>30
ありがとうございました!

実は問題式、間違ってました(^-^;)
3x+y-3z/x-y+z でした。答えは-1ですよね?
32132人目の素数さん:2001/03/27(火) 23:36
A(n) を 1,2,3,...,n の最小公倍数とするとき、Σ1/A(n) は無理数
であることを示せ、という問題があったはずだけれど、答は、がいしゅつなの?
これで合っていると思うんだけれど、どう?

b/a=Σ1/A(n) と表せたとする(a,b は自然数)。

p を a より大きい素数とし、上の式に A(p-1) を掛ける。

A(p-1)b/a=A(p-1)/A(1)+A(p-1)/A(2)+...+A(p-1)/A(p-1)+ΣA(p-1)/A(n)

最後のΣは、n≧p についての和。
最後の項以外はすべて整数。よって、最後の項も整数。正の整数だから、
ΣA(p-1)/A(n)≧1 でなくてはならない。
以下で、「<1」となることを示し証明が終わる。

p の次にくる素数を q とする。q≦2p-1 である。
(一般に2以上の自然数 p について、p<q<2p を満たす素数 q が存在するから。)
いま、p≦n≦q-1 の範囲にある n を考える。q≦2p-1 によりそのような n は高々 p-1 個。
A(p-1)/A(n)≦1/p は明らか。よってこの範囲の n について和をとれば、
ΣA(p-1)/A(n)≦(p-1)/p ・・・(1)

同様にして、q の次にくる素数を r とすれば、q≦n≦r-1 の範囲の n について、
A(p-1)/A(n)≦1/pq であり、
ΣA(p-1)/A(n)≦(q-1)/pq ・・・(2)

次の素数を s とすれば、r≦n≦s-1 の範囲で、
ΣA(p-1)/A(n)≦(r-1)/pqr ・・・(3)

同様にどんどん不等式を出して、(1),(2),(3),... をすべて加える。
ただし、これらの不等式は、A(p-1)/A(n) の評価の際、A(n) に素数が平方フリーで
現れることしか考えていない。実際は、p^2 という因子などもどこかに現れるはずだから、
すべての式で等号が成り立つことはあり得ないことに注意する。

ΣA(p-1)/A(n)<(p-1)/p+(q-1)/pq+(r-1)/pqr+...=1
33名無しさん:2001/03/28(水) 12:35
>>32
無限級数ですよ
34名無しさん:2001/03/28(水) 12:38
age
35名無しさん:2001/03/28(水) 12:44
有限和が有理数なことは自明ですよ
36132人目の素数さん:2001/03/28(水) 14:00
age
37132人目の素数さん:2001/03/28(水) 14:43
age
38名無しさん:2001/03/28(水) 19:01
age
とかないと
スレッドたっちゃう
39132人目の素数さん:2001/03/28(水) 22:05
あげ
4032:2001/03/28(水) 23:10
>>33,35
ん?
俺、なんか間違ってるの?
無限和で考えているけど。
41132人目の素数さん:2001/03/28(水) 23:21
33=35は途中までしか読んでないんで無いの?
4233=35:2001/03/29(木) 00:21
失礼しました
43名無しさん:2001/03/29(木) 00:42
>>32
なんか感動
ポイントはなんですか?
44名無しさん:2001/03/29(木) 01:35
次の計算ができません。教えて下さい。
@B^2=A*C  
A2C=B+D
B2(B+2)=A+C+3
C(C+2)^2=(B+2)(D+3)
上記の@〜Cを満たす、A,B、C、Dを求めてほしいんですが。
45132人目の素数さん:2001/03/29(木) 01:58
>>44
(A,B,C,D)=(1,2,4,6)
4644:2001/03/29(木) 02:32
ありがとうどざいました。
47132人目の素数さん:2001/03/29(木) 02:56
B+2=b,D+3=d,C+2=c,A=a

c^2=bd
2c+1=b+d

(t-b)(t-d)=t^2-(2c+1)t+c^2=0
t={(2c+1)±√(4c+1)}/2

2b=(2c+1)+√(4c+1)=a+c+1
√(4c+1)=a-c ⇔ (a-c)^2=4c+1 ⇔ a^2+c^2-2ac-4c-1=0
2(b-2)=a+c-3 ⇔ 4B^2=(a+c-3)^2=4a(c-2) ⇔ a^2+c^2-2ac+2a-6c+9=0

a^2+c^2-2ac-4c-1=0
a^2+c^2-2ac+2a-6c+9=0

2a-2c+10=0 c=a+5
a^2+(a+5)^2-2a(a+5)-4(a+5)-1=0 ⇔ -4a+4=0 ∴a=1,c=6,C=4

2b=a+c+1=8 ∴b=4,B=2
bd=c^2 ∴4d=36,d=9,D=6
48132人目の素数さん:2001/03/29(木) 03:31
その5にも書きましたが、数セミ4月号の86ページの2番って、
条件式に不備がありませんか?
49これ?>>48:2001/03/29(木) 03:39
http://www.nippyo.co.jp/m_sugaku_s/index.htm
※お詫びと訂正

「エレガントな解答をもとむ」
出題2
に条件の不備がありました.お詫びいたします.

実数 y>0,z>0
s≦t
(sとtの大小関係が抜けていました.)

以上の条件を補って解答を考えてください.
50132人目の素数さん:2001/03/29(木) 03:48
>>49
ありがとうございます。
51特命希望:2001/03/29(木) 15:25
分からない問題ではないのですが,ここにて質問させて頂きます.
1.時系列データのLiapunov指数を計算したいと思っているのですが,
何かよいソフトはありませんか.教えて下さい.

2.あと,FFTを分析するソフトもあれば,教えて下さい.フリーソフト
なんかがあるのですが,どうも,結果を信用できなかったりします.
ほぼ正解なのでしょうが.....周期の端を見るとなんかうそくさい
部分もあるような気がするのです.

3.パワースペクトルの値が負の値を取る結果があります.
これはどんな計算しているのでしょうか?
52数学の問題か、、:2001/03/29(木) 18:22
どうかよくわからないのですが、、、、
「ひつじが1000匹います。1匹目をa、2匹目をb、3匹目をcと
 して、以下4匹目からは必ずa、b、cのどれかに属します。
 さて、64匹目はa,b,cのどのグループに所属するでしょう?」
ちなみに64匹目はbらしいのですが、こういうのを、
「1匹目、2匹目、3匹目(a,b,c)は『○○○○』が違う」というのでしょう?
『○○○○』を教えてください(ひらがなだと4文字)。
集合の問題ですかね?

53132人目の素数さん:2001/03/29(木) 18:33
『ぐるーぷ』が違う
54132人目の素数さん:2001/03/29(木) 18:34
『しょぞく』が違う
55132人目の素数さん:2001/03/29(木) 18:35
『はいぞく』が違う
56132人目の素数さん:2001/03/29(木) 18:36
『じょうよ』が違う
57132人目の素数さん:2001/03/29(木) 18:39
だいたいなんでこれだけの条件で64匹目がbに属してることが
分るんだ?a,b,cの順で入れてったらaに入るんだが、、、
58132人目の素数さん:2001/03/29(木) 18:42
1000匹というのは何か意味が…
59だまされてるのかな?:2001/03/29(木) 19:00
52です。皆さん考えてくださりありがとうございます。
ちなみに、
「4匹目はbである
 5匹目もbである
 6匹目はaである
 7匹目はbである
 8匹目はaである
 9匹目はbである
 10匹目はaである」
だそうです。
まるで規則性はわかりません。
やっぱ数学ではないのかな。
60132人目の素数さん:2001/03/29(木) 19:07
cに入る奴が何故でてこないんだろう、、、
61132人目の素数さん:2001/03/29(木) 19:07
ぴき ひき びき
62132人味の素さん:2001/03/29(木) 19:11
 有名ななぞなぞだな。こんなのもある。
1=a,2=a,3=a,4=a,5=a,6=b,7=a,8=c,9=b,10=b,…
63132人目の素数さん:2001/03/29(木) 19:14
○○○○はなんだったの?
64132人目の素数さん:2001/03/29(木) 19:15
>>62
∞=c ?

65132人目の素数さん:2001/03/29(木) 19:32
>>64
4=aだよ?
66132人目の素数さん:2001/03/29(木) 21:22
>>63
よみかた?
67132人目の素数さん:2001/03/29(木) 21:29
>>65
||
└┼
きっとこういう感じの4なんだらう。
 
68お騒がせしました:2001/03/29(木) 23:10
52です。やっぱり騙されてました。
なぞなぞでした。66さん正解でした。
考えてくださった皆さんありがとうございました。
わたしは逝ってよしですよね。
ハイ、逝ってきます。
69132人目の素数さん:2001/03/30(金) 01:45
>わたしは逝ってよしですよね。
>ハイ、逝ってきます

面白いです。ほのぼの
70132人目の素数さん:2001/03/30(金) 08:07
等差数列、等比数列はなんとなくわかるんですが、
もうひとつの階差数列というのがよくわかりません。
教えて下さい。
71132人目の素数さん:2001/03/30(金) 10:17
>>51
Ans.1 そろばん、電卓、スクリプト、Mathmatica、どれでも好きなものを
Ans.2 上記と同じ、それに自分の五感が加わる
Ans.3 基準値が題意に無い問題、数学ではなく物理、それも電磁気の問題に多し
72工房:2001/03/30(金) 11:48
単独の階差数列というものはない。
ある数列a(n)の隣り合う2項の差
a(2)-a(1),a(3)-a(2),a(4)-a(3) ・・・を各項とする
数列b(n)を、数列a(n)の階差数列という。


3,5,7,9,11・・・の階差数列は
2,2,2,2・・・
2,6,18,54,162・・・の階差数列は
4,12,36,108・・・

これでいいですか?
ちなみに調和数列というものもあるが、こちらは
実用性があるのかよくわからん。
73132人目の素数さん:2001/03/30(金) 15:07
age
74132人目の素数さん:2001/03/31(土) 00:47
∞って実数なんですか?
あと超越数ってどーゆー定義ですか?
75132人目の素数さん :2001/03/31(土) 02:13
>74
∞ってのは数とは呼ばないと思うが・・・。
超越数ってやつは一言でいうなら
「有理数でよく近似できる数」
というところだろーか。
76132人目の素数さん:2001/03/31(土) 02:22
>>74
∞は実数ではありません。そもそも数ではありません。

超越数というのは有理数係数の代数方程式の解にならない実数という定義です。
解になる数を代数的数といいます。
7774:2001/03/31(土) 02:48
なるほど。簡潔なレスさんくす。
∞>これは俺の勘違いっていうか思い違いでしたか。
超越数>てことは広さ(濃度?)としては実数>無理数>超越数
といった感じでしょうか。
78132人目の素数さん:2001/03/31(土) 02:56
>>77
包含関係としてはそんな感じです。
濃度はどれも実数と同じ連続濃度です。
79132人目の素数さん:2001/03/31(土) 03:22
eやπが超越数である事の証明はどうやりますか?
80sage:2001/03/31(土) 08:32
eの超越性は
「an:0でない代数的数 pn:互いに異なる代数的数 このとき
a1*e^p1+a2*e^p2+・・・+an*e^pn+・・・・
は0にはならない」(リンデマンの定理)
から出る。
(eの超越性自体は、係数と指数が整数のときを言えばよい。それを最初に示したのはエルミート)
これの証明はちょっと大変。この定理を使えばπの超越性がEulerの公式からただちにでる。
81まじめに:2001/03/31(土) 14:40
age
82132人目の素数さん:2001/03/31(土) 18:43
わからない問題はここに書いてね
8330:2001/03/31(土) 18:50
>>79
北大の田口さんのページに証明があったような
84132人目の素数さん:2001/04/01(日) 01:51
定期age
85さくら:2001/04/01(日) 18:08

    ,   ― ノ)
 γ∞γ~  \   / ̄ ̄ ̄ ̄ ̄ ̄ ̄ ̄ ̄ ̄ ̄ ̄ ̄ ̄ ̄ ̄ ̄
 人w/ 从从) ) みんな,おっはよー. 今日4/1は私の誕生日♪
  ヽ | | l  l |〃 わからない問題は,今日もさくらといっしょに
  `wハ~ ーノ)   レリーズ!
   / \`「   \_________________
86さくら >6:2001/04/01(日) 19:04
>{(x+y)^n}-(x^n)-(y^n)が{(x^2)+(xy)+(y^2)}*(x..yの多項式)
>の形に表される様な2以上の自然数nを小さい方から3つ求めよ。

    ,   ― ノ)
 γ∞γ~  \   / ̄ ̄ ̄ ̄ ̄ ̄ ̄ ̄ ̄ ̄ ̄ ̄ ̄ ̄ ̄ ̄ ̄
 人w/ 从从) )  x^2+xy+y^2=0を解くとy=ωxより,f(x,ωx)=((1+ω)^n-1^n-ω^n)*x^n
  ヽ | | l  l |〃 =((-ω^2)^n-1^n-ω^n)*x^n=(-1-ω^n+(-1)^n*ω^(2n))*x^n
  `wハ~ ーノ)   これが0となるのはnが3の倍数でない奇数のとき(*)だから,n≡1,5 (mod.6).
   / \`「   \_________________

# すでに答えは出ているけど解いてみたので答えてみました.
f(x,y)=(x+y)^n-x^n-y^n,ωは1の虚数立方根で1+ω+ω^2=0,ω^3=1.
(*)は,-1-ω^n+(-1)^n*ω^(2n) と 1+ω+ω^2=0に注目してね.
87132人目の素数さん:2001/04/02(月) 02:24
>>85
おめでとうございます。微妙な日のお生まれですねぇ。
88132人目の素数さん:2001/04/02(月) 18:30
中学校でやる回転体の問題です。

http://widech.virtualave.net/upload/img-box/img20010402192927.jpg
89132人目の素数さん:2001/04/02(月) 19:35
1+1=2を証明してください。
90132人目の素数さん:2001/04/02(月) 21:22
[証]
義務教育初等学習過程において、
1〜10までの数を習う ・・・・@
林檎が 1個と1個で2個あると教わる ・・・・A
足し算とは数を足すことだと分かる  ・・・・B
1足す1は 1+1で表すと知る。   ・・・・C
Aより、林檎が1個と1個なので
林檎は2個、
@ABCより1+1=2である。
                   <終了>

証明下手だが、こんなんでも反論がきそうだから笑える。
91132人目の素数さん:2001/04/02(月) 23:08
仮定は何ですか?>>89
92132人目の素数さん:2001/04/02(月) 23:51
>>89
「証明」って言葉は違いませんか
93132人目の素数さん:2001/04/03(火) 00:19
むかしから疑問だったので教えてください。
0<=x<1の実数の集合から任意の一点を選んだときに、その点が有理数である
確率っていくつなのでしょうか?
有理数と実数とが1対1対応つかないことは知っています。
#そもそも確率とは何かわかっていないのかもしれません。
94132人目の素数さん:2001/04/03(火) 00:27
>>93
Yahoo
95132人目の素数さん:2001/04/03(火) 00:42
>>93
[0,1)の有理数は可算個だから
確率(測度)は0
96132人目の素数さん:2001/04/03(火) 00:52
>>93
俺もよくわかんないけど
0≦x<1の実数の集合から任意の一点aを選んだときに、
(1)1/3≦a<1/2 である確率はいくつだとおもいますか?
(2)a=1/2 である確率はいくつだとおもいますか?

カンケイないけど
「任意に選ぶ」=「無作為に選ぶ」
ということなのかなあ・・・。
97132人目の素数さん:2001/04/03(火) 01:10
>>96
(1)1/2-1/3=1/6←確率
(2)確率は0

「任意に選ぶ」=「無作為に選ぶ」です。
98132人目の素数さん:2001/04/03(火) 01:16
>>93
公理では

1点を選ぶ確率が0
 ↓
有限個の点を選ぶ確率が0
 ↓
可算無限個の点を選ぶ確率が0

でも感覚的にはうーん・・・
9993:2001/04/03(火) 01:17
>>95
今ある本を見ていたら、A. Kolmogorovによる確率論の測度論による公理的な
基礎づけ、というのが出ていました。きっとその辺を見ると、0となるのが
わかるのですね?ありがとうございました。
10096:2001/04/03(火) 01:29
なんだか話が終わっちゃったみたいだけど、こういう話は
「無作為に選ぶ」というのを

0≦s<t≦1なる実数s、tに対して
0≦x<1の実数の集合から一点aを“無作為に”選んだときに、
s≦a<t である確率をt−sとする。

と定式化(定義?)して、ここから話を進めるんだとおもいます。
101tr:2001/04/03(火) 01:31
>>88 さん
l に関する C の対称点を C'
AC と l の交点を P, C'P と l の交点を Q
A, C, Q から l に下ろした垂線の足を D, E, R
と定める。このとき
  △ADP ∽ △CEP (相似比 2 : 1)
  △CEP ≡ △C'EB
なので
  PA : BC' = 2 : 1
これと PA // BC' から
  △APQ ∽ △BC'Q (相似比 2 : 1)
  ⇒ △C'PE ∽ △QPR (相似比 3 : 2)
  ⇒ PR = 2/3, RE = 1/3, RQ = 2/3

# あとは計算するだけです♪
102tr@訂正:2001/04/03(火) 02:07
誤) C'P と l の交点を Q
正) C'P と AB の交点を Q
103132人目の素数さん:2001/04/03(火) 12:44
N角数の逆数の無限和はどういった値になるかわかっているのでしょうか?
3角数(n(n+1)/2),四角数(1/n^2)についてはわかりますがそれ以上はどうなのでしょうか。
104132人目の素数さん:2001/04/03(火) 14:06
Q:立方体の少なくとも3辺の中点を通る平面は全部で何個あるか?
105え?重複してる?:2001/04/03(火) 15:12
>>104
中点が12個あるから・・・・・<考え中>・・・・・12C3=220個です!
106132人目の素数さん:2001/04/03(火) 16:04
1.面を含むもの 6個
2.面に平行で1じゃないもの 3個
3.各頂点に最も近い3中点を通るもの 8個
4.中点4個を通り1、2じゃないもの12個
5.中点6個を通り、切り口が6角形になるもの 4個
6.切り口が5角形になるもの 24個
7.1辺を含むもの 24個
計81個

12C3から重複分を除くことを考えると
1,2,4の21個は4中点を通るので (4C3-1)*21=63個重複
5の4個は6中点を通るので (6C3-1)*4=76個重複
220-63-76=81個
107132人目の素数さん:2001/04/03(火) 16:35
麻雀の天和と、ポーカーで最初からロイヤルストレートフラッシュになるのはどっちが起こりやすい?
108132人目の素数さん:2001/04/03(火) 22:59
最初からロイヤルストレートフラッシュになる確率

ジョーカーを含まない52枚のカードから5枚もらう場合の総数は
C{52,5}=311875200(通り)
ロイヤルストレートフラッシュになる場合の総数は、
絵柄が4つあるので4通り
よって確率は4/311875200=1/77968800

天和はどういう役か知らん。役立てなくてスマソ
109132人目の素数さん:2001/04/03(火) 23:20
>>108
>C{52,5}=311875200(通り)

5!で割ってくれよん
52C5=2598960通りだよん

ロイヤルストレートフラッシュの確率
=4/2598960
=1/649740
≒65万ぶんの1<

天和の確率は約33万ぶんの1らしい
110132人目の素数さん:2001/04/03(火) 23:28
>>89
数0の存在をGaitiとする。
ここで、0とは違う数1を考える。
ある数が0と1の差と同じく、1の次に存在するとき、その数を1’とする。
同じように、0と1の差と同じく、1’の次に存在する数を1’’とする。
こうして出来た集合X={x|1(’*X)}の全ての数に、
1=1、1’=2、・・となる記号を1対1対応させる。(2以降便宜上省略
ここで、「ある数Aに’をn個加え、集合X内の別の数A(’*n)に変える」
という演算を表す記号「+」を考える。
すなわちA+n=A(’*n)とする。
すると、2=1’=1+1
よって、1+1=2
111132人目の素数さん:2001/04/03(火) 23:39
等式 nPr= n-1Pr + R n-1Pr-1を証明せよ.

よろしくお願いします...
112>111:2001/04/03(火) 23:49
定義  nPr = n (n-1)・・・(n-r+1) = n!/(n-r)!

計算するだけっす
113132人目の素数さん:2001/04/03(火) 23:53
>>111

左辺nPrは「n人からr人を選んで一列に並べる方法の総数」・・・★
だが、いまn人の中の1人(A君とおく)に着目して、★を
「A君を選ばない場合の並べ方」と「A君を選ぶ場合の並べ方」
に分けたものが右辺。

「A君を選ばない場合」は、A君以外のn-1人からr人選んで並べるので
n-1Pr 通り。
「A君を選ぶ場合」は、A君が何番目にくるかでr通りの決め方があり
さらに残るr−1人の並べ方がn-1Pr-1 通りなので、この場合は
r×n-1Pr-1 通り。
114ひろ:2001/04/04(水) 00:55
天和とは麻雀で自分が親でちょんちょんを
自分がとった14個目ですでにロンの状態になっている
ことですね。
これは難しい・。
自分が親で確立4分の1、・・・・??わからん。
115132人目の素数さん:2001/04/04(水) 01:49
(*'ー')ノ分数の割り算は、どうして反対にして掛け算に
    なるんですか
116132人目の素数さん:2001/04/04(水) 01:49
(*'ー')ノ分数の割り算は、どうして反対にして掛け算に
    なるんですか
117132人目の素数さん:2001/04/04(水) 02:05
生活全般板から来たんですが、この解き方ってあってるの?間違ってるの?

元スレ http://cheese.2ch.net/test/read.cgi?bbs=kankon&key=986020427
148 名前:おさかなくわえた名無しさん投稿日:2001/04/03(火) 17:06
ワシが小学4年生のとき、朝顔の観察日記をつけさせられた。
どうせ10歳かそこらのガキが書く日記である。何日目に双葉が
開いた、とか何日目に本葉が出てきた、とか、その本葉の数を
記録したり、とか、まぁそんな程度のものが多かったのだが、
ひとりだけ「朝顔の葉の面積」を細かくつけている奴がいた。
それも「25,5平方センチメートル」とか、「31,5平方センチメートル」
と、極めて細かい数値まで出していた。
ワシは「(゚д゚)ハァ???」となった。
担任の先生も「(゚д゚)ハァ???」となったらしく、
「おい、K下、どうやってこんなに細かい面積を出したんだ?」
と、その同級生に聞いた。
するとそいつの答えは、
「まず画用紙を5mm四方の正方形二枚と、10mm四方の正方形
 多数に切っておく。
 次に画用紙に朝顔の葉の輪郭を写し、その輪郭に沿って切り抜く。
 そうしておいて、その葉の形をした画用紙を、天秤ばかりの一方に
 乗せ、釣り合うまで正方形の画用紙を乗せていく。紙の厚みは
 同じなので、釣り合った所で10mm四方の画用紙の数と5mm四方の
 画用紙の数を数える」
だった。そいつの家は薬局で、天秤ばかりが有ったからこその方法だが、
これを聞いたときは、水戸黄門に印籠を見せられた悪代官になってしま
った。

長くてスマソ。
118132人目の素数さん:2001/04/04(水) 02:33
面積比=底面積*高さ(等しい)の比=体積の比=重さの比
119132人目の素数さん:2001/04/04(水) 04:48
A先生は「来週、抜き打ちテストをします。」といいました。
B君はいいました。
・土曜日に抜き打ちテストはできないから、月〜金のどれか
 にしなければならない。
・土曜日にできないことはわかっているので、金曜日に
 に抜き打ちテストをすることはできない。
・以下続く。
・よって、結局抜き打ちテストはできない。

どこがおかしいですか?
120>119:2001/04/04(水) 04:51
>土曜日に抜き打ちテストはできない

なんで?
121132人目の素数さん:2001/04/04(水) 06:29
>>114
33万分の1と言われている天和の確率は
親のときに牌配で和了れる場合を計算しています

(国士無双+一般形+七対子−ニ並向)/(136C14)

こんな感じできっちり計算してたサイトを見かけたけど
今探したら見つかりません(><)
122119ではないが・・・:2001/04/04(水) 07:49
>>120
これは昔からある有名問題(パズル?)です。
土曜日にやるということは金曜日まではないということ。すると金曜日
にないということが分かった時点で、土曜日にテストがあるということ
が分かってしまうので、「抜き打ち」テストにならないという理屈です。
123132人目の素数さん:2001/04/04(水) 08:57
>>113
これで証明になるの?
124132人目の素数さん:2001/04/04(水) 11:24
>>119
ある日にテストがあるとわかった→抜き打ちにならないのでその日にテストはない
→その日にテストがないと思っているのでテストを行えば抜き打ちになる
→以上の過程をすべて予測した上で、テストはないと判断する
→以上の過程をすべて予測した上で、抜き打ちテストを行う
→・・・・(以後永遠に続く)
125132人目の素数さん:2001/04/04(水) 11:41
>>123
nCr
の定義が
n個のものからr個取る組み合わせなんだから

それに
たとえば
nC0 + nC1 + ・・・ + nCn
を求めるときなんかは
組み合わせ論的解釈をしないと
めんどうじゃない?
126132人目の素数さん:2001/04/04(水) 11:43
nC0 + nC1 + ・・・ + nCn=2^n
これは二項定理を使って導くのが常套では?
127132人目の素数さん:2001/04/04(水) 11:45
>>117

こういう奴、スキっす。
128132人目の素数さん:2001/04/04(水) 11:45
n人から何人か委員を選ぶ
ことを行うには
一人一人が委員になるかならないかが決まればよい
よって
2^n
と解釈する方がスマートでしょう
129132人目の素数さん:2001/04/04(水) 11:48
意味より計算式を第一に考えるのはナンセンスですよ
130132人目の素数さん:2001/04/04(水) 11:50
計算中は意味を捨象しないと、そもそも計算という手法を採用する意味がないですよ。
131132人目の素数さん:2001/04/04(水) 11:53
まあ覚えるならばわかりやすい方を、
厳密な証明を求めるならば直感を排除した数式の方を
それぞれ採用すれば
132132人目の素数さん:2001/04/04(水) 11:53
数式という形が問題で取られているからと言って
計算という手法を取れと言う命令ではありませんよ
要はその数式が「読めれば」いいんですから

計算と意味は臨機応変に取るべき手法であって
意味の捨象が必要な考察もあると思いますが
ここでは関係ないでしょう
133132人目の素数さん:2001/04/04(水) 11:55
そもそもその様な言い方では
組み合わせ、順列の記号の計算式が先立っているように思えて
非常に不自然ですが>厳密な証明
134132人目の素数さん:2001/04/04(水) 11:56
ナンセンスっていうのは取り消しておきます
135132人目の素数さん:2001/04/04(水) 12:01
>>132

131さんの意見が妥当だと思いますよ。

・一般の問題を解決するのに、数学的手法を使うかどうかという選択がある。
・数学的な手法を用いる場合は、問題をモデル化して、モデルについて計算し、結果をモデルの習性や実際の解決に結びつけなければならない。
・数学的手法を用いる場合には、計算を手段として用いる必要がある。計算そのものは数字や演算子、それらの拡張されたものの意味のみを扱い
その他の部分を捨象することができ、この事が数学的手法に一定の優位性を与えている。

一般的に言うと依存関係というのはこういう感じだと思いますよ。
計算内容に依存しない汎用性の高い計算方式や記述方式を編み出すというのは数学家の独壇場でしょう。
それらに対する信頼がなければ、工学も物理学も天文観察の様な神話的な要因以外に余り意味のないような学問も
信頼されることはないでしょうし。
136132人目の素数さん:2001/04/04(水) 12:03
で、この場合は抜き打ちテストの対策には一日あれば十分だというのが暗黙の前提になって、
条件から抜けてるんですね。
137132人目の素数さん:2001/04/04(水) 12:09
物理学の特殊な方程式たちのようにそれも一理ありますが

こんな基礎的なところで直感を排除するのはどうでしょうか?
人間が直感で得た事実からtermを定義して、そこで
初めて得た計算式じゃないですか。
数式処理の信頼性に依存するのはもっと後でいいと思われます。
これはdefinitionと、このtermの意味するところの確認として
あるべきだと思われます。
138132人目の素数さん:2001/04/04(水) 12:15
完全な数式処理と完全な論理的考察
を求めるのは確かに数学を生業とする
人間を養うには必要かもしれませんが
教育に於いて、数学が"養える"思考力はその範囲のみでは
無いと思います。
それに、そういう思考力もまた、実際のところ
数学者たちは持っており、必要な気がします。
ちょっと不安ですが
139132人目の素数さん:2001/04/04(水) 12:23
物理学などは学者の妄想にすぎないものが
実証を得ることで信頼を受けて一般に通用することは
たしかにそうですが、
先端を開拓している人間たちは直感的思考を持った
人たちも多いじゃないですか。
古典物理学においてはそれが殆どですし
現代物理学を開いたA.Einsteinなんかは、人間的直感による
業績に他ならないのではないでしょうか?
140132人目の素数さん:2001/04/04(水) 13:01
少なくともアインシュタインが活躍した頃の物理の方程式はごく普通の線形な方程式で、
特殊な直観的能力を要求されるものではないのでは?
アインシュタインが物理学に功績を残したのは物理学に必要な直観を持っていたからだという可能性はありますが、
アインシュタインが数学に必要な何らかの直観を介して数学に業績を残したと言う事はないと思われます。

それはそうと、抜き打ちテストの予習には一日で十分である。抜き打ちテストの予習をしても一日たったら忘れてしまう。
どうせしないと行けない勉強でも一日でも送らせてやりたい、など定式化を待っている条件がいろいろあると思うのですが、
そちらの方では何かアイデアはないでしょうか?
141132人目の素数さん:2001/04/04(水) 14:21
数学のこといったんじゃないんだけど>Einsteinの話
142132人目の素数さん:2001/04/04(水) 15:04
>>141
別の板へどうぞ。
143132人目の素数さん:2001/04/04(水) 16:51
>>142
変な揚げ足取るな
しね
144132人目の素数さん:2001/04/04(水) 16:53
ごめんなさい

??
145132人目の素数さん:2001/04/04(水) 17:31
アインシュタインが数学に業績をもたらしたと言えるかどうかはどーでもいーけど
少なくともアインシュタインは数学者に飯の種を沢山与えたと思われ
146132人目の素数さん:2001/04/04(水) 17:36
>>142 ごもっとも
147132人目の素数さん:2001/04/04(水) 18:31
数学への影響から言ったら、Einsteinよりもδ関数なんていう厄介かつ便利なシロモノ創り出したDiracの方が大きいと思われ
148132人目の素数さん:2001/04/04(水) 21:13
>アインシュタインが活躍した頃の物理の方程式はごく普通の線形な方程式

おまえ、いいかげんな事ほざいてんじゃねえ。
Einstain方程式は非線型偏微分方程式だぞ。
149132人目の素数さん:2001/04/04(水) 21:22
y = a - b・exp{-((x+c) / d))^6}

データ(x, y)があって、このデータを上のワイブル関数にカーブフィッティングしたい。
4個のパラメータa,b,c,d(a,b,c,d > 0)を最小ニ乗法で決定したいのですが、どうすればいいのでしょうか。
150132人目の素数さん:2001/04/04(水) 21:48
>>142
直感的解釈が物理学では必要だと言った
Einsteinが数学に業績を残したことは言及していない
->数学のことを言ったのではない
板違いじゃない
しんどけ
151132人目の素数さん:2001/04/04(水) 23:14
Z/2Zは射影的でないことをしめせ
代数問題ですがよろしくお願いします
152132人目の素数さん:2001/04/05(木) 01:12
P(x)−Q(x)がxについての2次式になり、
P(x)=0と、Q(x)=0がただ一つの共通解αを持つとき、
「P(α)−Q(α)=0、かつP(α)−Q(α)のD=0」
ってあってますか?
153132人目の素数さん:2001/04/05(木) 01:21
Z、Z/2ZはZ加群とみなすとする。
p:Z→Z/2Z 標準的射影(もちろん全射準同型)
id:Z/2Z→Z/2Z 恒等写像(もちろん準同型)
f:Z/2Z→Z を準同型とすると
[0]、[1]に対して
f([0])=0、f([0])=f([1]+[1])=f([1])+f([1])=0 よりf([1])=0
すなわちfは零写像。よってid=p・fとならないのでZ/2Zは射影的でない。
154132人目の素数さん:2001/04/05(木) 01:27
>>152
なんか、勘違いしてるかな?
条件から
P(x)−Q(x)=a (x-α) (x-β)

でしょ。P(x)=a (x-α) f(x)、Q(x)=a (x-α)g(x) とおいて
f(x)=g(x)+(x-β)ととって、g(β)≠0とでもしておけばf(x)とg(x)は共通根を持たず
このときP(α)−Q(α)=0は当然成り立ち
α≠βのときD≠0
155132人目の素数さん:2001/04/05(木) 02:21
Einstain博士曰く、
「数学者が口を出すようになってから、私にも相対性理論がよくわからない
代物となってしまった」
なんかの本で読みましたよ。
156132人目の素数さん:2001/04/05(木) 02:27
なんか、物理の人って数学の人をよく煽るよね。
なんだか、文系が理系を煽ってるみたいだ。
所詮物理。煽り方も幼稚だ。
157156:2001/04/05(木) 02:29
ごめんレスする場所間違った。
>>156
は無視してくれ。
158132人目の素数さん:2001/04/05(木) 17:21
3 ネター環だがアルティン環ではない例をあげ証明せよ
159132人目の素数さん:2001/04/05(木) 19:34
>>158

えー?そんなのいくらでもあるじゃん。
たとえば整数環Z。
160152:2001/04/05(木) 20:35
>>154
あああ、わかりました。ありがとう。
161132人目の素数さん:2001/04/05(木) 21:10
僕は浪人生なんですがひとつ解らない問題があって悩んでいます。
それは

3^210/3^10+1
の桁数と1の位の数字を求めよ。

という問題です。
どうかお願いします。
問題がわかりにくいなどありましたら書き込んでください。
162132人目の素数さん:2001/04/05(木) 21:18
3^210/(3^10+1)
じゃないんですか?
163161:2001/04/05(木) 21:37
>>162
おっしゃる通りです。
すいませんでした。
164y^2=x^3+ax+b:2001/04/05(木) 21:44
3^200-1 < 3^210/(3^10+1) < 3^200
が成り立つ

3の階乗の1の位は3,9,7,1の順に循環するから
3^200の1の位は1
よって 3^210/(3^10+1) の1の位は0

log10 3^200 = 200 * log10 3
から桁数を計算すればいい

・ポイントはよくわからない値の評価
(この場合、分母の小さい値を消して大小評価の式を作ってみた)
165y^2=x^3+ax+b:2001/04/05(木) 21:45
>>164
あー
すいません
大嘘です
ごめんなさい
166y^2=x^3+ax+b:2001/04/05(木) 22:08
よく見たら誤差だらけだ
ばかばかしい
ほんとにすいません
167y^2=x^3+ax+b:2001/04/05(木) 22:22
3^199 < 3^210/(3^10+1) < 3^200
が成り立つので
これで桁数は出るはず
log10 3=0.4771
あたりでやってみてください
168y^2=x^3+ax+b:2001/04/05(木) 22:24
・・・出ない・・・
もうだめだ
169132人目の素数さん:2001/04/05(木) 22:37
っつーか
これ東京か京都の入試問題じゃないの?
170161:2001/04/05(木) 22:41
神戸大学です。
171132人目の素数さん:2001/04/05(木) 22:47
常用対数は与えられてますか?
172161:2001/04/05(木) 22:49
log102とかのことですよね。与えられてます。
173分からん鬱山車悩・・・:2001/04/05(木) 22:49
n
2/n+2
k=1
174y^2=x^3+ax+b:2001/04/05(木) 22:51
2 * 3^199< (与式) <3^200
で評価できました
175132人目の素数さん:2001/04/05(木) 22:55
>>173
kがシグマの中にありません
176問題書き間違えた鬱山車悩:2001/04/05(木) 23:00
n
2/k+2
k=1

177y^2=x^3+ax+b:2001/04/05(木) 23:47
1の位は2かな
178y^2=x^3+ax+b:2001/04/05(木) 23:48
最高位だった・・・なにやってんだろ・・・
179132人目の素数さん:2001/04/06(金) 00:27
3^210/(3^10+1)
=(3^210-1+1)/(3^10+1)
=(3^210-1)/(3^10+1)+1/(3^10+1)
=(3^200+3^190+3^180+・・・+3^20+3^10+1)+1/(3^10+1)
かな???
180152:2001/04/06(金) 00:29
3^210=(3^10)^21・・・@
3^10+1=xとおくと
@=(x-1)^21
これをxで割ると、
@=x(x^20+・・・+1^20)+1^21なので、余りは1。
よってこの割り算の結果、即ち3^210/(3^10+1)は、
整数部分=((3^10+1)^20+・・・+1^20)、小数部分=1/(3^10+1)
で与えられる。

・・・で、ここからどうにかなりませんか(汗
181180:2001/04/06(金) 00:31
>>179
かぶった?(汗汗
182180:2001/04/06(金) 00:36
なんか違う。
@=x(x^20+・・・+1^20)+1^21→x(x^20+・・・+“20*”1^20)+1^21
183tr:2001/04/06(金) 01:29
y^2=x^3+ax+b さん, 152さんのアイディアをまとめました♪

>>161 の問
3^10+1 = x として 3^210 = (x-1)^21 を 2項展開
  3^210 = x^21 + (21C20)*x^20*(-1)^1 + … + (21C1)*x*(-1)^20 + (-1)^21
これより 3^210/(3^10+1) = (x-1)^21/x の整数部は
  x^20 + (21C20)*x^19*(-1)^1… + (21C2)*x*(-1)^19 + (21C1)*(-1)^20 …(#)
ここで、3^n について
    n  | 1 2 3 4 5 6 7 8 9 10
  1の位 | 3 9 7 1 3 9 7 1 3 9
なので、3^10+1 = x の 1の位は 0 であり
  (#) ≡ (21C1)*(-1)^20 ≡ 1 (mod10)
184y^2=x^3+ax+b:2001/04/06(金) 01:43
二項展開か・・・気づかなかったです
>>183 さん
ちょっとした訂正ですが
整数部は1引いた値です
185132人目の素数さん:2001/04/06(金) 01:55
>>179
わりきれませんよぉ>因数分解したところ
186tr:2001/04/06(金) 02:20
>>184 = y^2=x^3+ax+b さん
あ、ホントだ。ご指摘ありがとうございます♪

>>161 さん
と言うことで >>183 には修正が必要です。
流れはつかめると思うので、答案作成に使ってください。
187132人目の素数さん:2001/04/06(金) 13:01
かいてね
188180:2001/04/06(金) 14:30
そうか、xの1の位が0なのか・・やたらやばい計算がいるのかと思った(笑(えない
189>>176:2001/04/06(金) 15:09
2/3+2/4+2/5+2/6+2/7+2/8+2/9+2/10+・・・・2/n+1+2/n+2
190こぴぺ侍:2001/04/06(金) 15:15
>>161

> A=3^210/(3^10+1)とおく.
> 3^210/(2*3^10)<A<3^210/(3^10)
> → 3^200/2<A<3^200
> → log_{10}(3^200/2)<log_{10}(A)<log_{10}(3^200)
> → 200*log_{10}(3)-log_{10}(2)<log_{10}(A)<200*log_{10}(3)
> log_{10}(2)≒0.3010,log_{10}_(3)≒0.4771 より
> 95<log_{10}(A)<96
> よって,Aの桁数は96.
>
> A=((3^10)^21+1)/(3^10+1)-1/(3^10+1)
> =(3^10)^20-(3^10)^19+・・・+(3^10)^2-3^10+1-1/(3^10+1)
> =3^200-3^190+・・・+3^20-3^10+1-1/(3^10+1)
> この数の整数部分[A]は,
> [A]=3^200-3^190+・・・+3^20-3^10
> 3^4≡1 (mod.10)に注目すると,
> [A]≡(1-9)*5≡0 (mod.10)
> よって,Aの1の位の数字は0.
191>>189:2001/04/06(金) 15:22
ワラた
192132人目の素数さん:2001/04/06(金) 18:19
>>176
分母=(n+2)!/6
分子=2*(1+(n+1)+(n*(n+1))・・・
分子=An=(n+1)!/k!=2*(n+1)P(k) (k<n+2)
の和*2。
Sn=12*Σ((n+1)P(k))/(n+2)!

できてない。
193132人目の素数さん:2001/04/06(金) 19:03
m=k+2として、
2*Σ[3, n]1/m=2*Σ[1, n]1/m - 3
Σ1/mを表す簡明な式ってのは無いのでは?
194132人目の素数さん:2001/04/06(金) 20:13
自分てやれや
195132人目の素数さん:2001/04/06(金) 23:17
f(z)=0という方程式の解(複素数の範囲で解を考える)を頂点とする多角形を考える。
導関数の方程式 f’(z)=0 の解はこの多角形の内部または周上にのみ存在する。

この定理はなんと言う名前でしょうか?

196132人目の素数さん:2001/04/06(金) 23:42
便乗で定理名質問。
三角形ABCで角Aの二等分線とBCの交点=Mのとき、
AB:BM=AC:CM
「角の二等分線に関する定理」
みたいな複合名詞句的な名前じゃなくて。
197132人目の素数さん:2001/04/06(金) 23:54
それってその複合名詞句的な名前以外にあるのかなぁ
それも一応正式名称でしょ
198132人目の素数さん :2001/04/07(土) 00:06
>>195
“Gauss-Lucas theorem” または “Lucas's Theorem” みたい。
アールフォルスの「複素解析」では「ルカスの定理」になっていた。
199132人目の素数さん:2001/04/07(土) 00:44
>Σ1/mを表す簡明な式ってのは無いのでは?

簡明な式じゃないけど、ガンマ関数の導関数を使えば表せるみたい。

Γ(x+n+1)/Γ(x+1)=(x+1)(x+2)(x+3)・・・(x+n)

この式を x で微分してから x=0 として、n! で割れば Σ[m=1,n]1/m になる。
実際計算して、Γ(n+1)=n!、Γ(1)=1、Γ'(1)=-γ (γはオイラー定数)を使うと、

Σ[m=1,n]1/m=Γ'(n+1)/n!+γ

になると思う。
200132人目の素数さん:2001/04/07(土) 01:08
ガンマ関数って自然数定義域だと階乗のことだよね?
導関数ってあるんですか?
201132人目の素数さん:2001/04/07(土) 01:24
定義域が階乗と違って自然数じゃないからね…(アホ
202200:2001/04/07(土) 01:42
すいません;;
わからなかったもので質問しただけなんです
実数の関数値を補完したりするんでしょうか
203132人目の素数さん:2001/04/07(土) 01:56
個の板の常連って何人くらいなんだろう
204132人目の素数さん:2001/04/07(土) 04:50
>203
知るかヴぁか!
205132人目の素数さん:2001/04/07(土) 11:29
>>204
ありがとうございます
206132人目の素数さん:2001/04/07(土) 11:59
>>203
任意の自然数
207132人目の素数さん:2001/04/07(土) 12:56
age
208132人目の素数さん:2001/04/07(土) 18:35
囲碁の先後の決め方。一方の人が何個かの石を握り、もう一方の人がその石の個数が偶数であるか奇数であるかを当てることによって先後を決める。
後者の立場の人は「偶数」と「奇数」どちらを予想すべきか
209132人目の素数さん:2001/04/07(土) 19:00
>>202

>>199のxは自然数とは限らないよ
210132人目の素数さん:2001/04/07(土) 19:02
>>208

どっち選んでも同じでしょう。
211132人目の素数さん:2001/04/07(土) 19:02
いや、だからどんなのかと思って質問したんです
私は自然数の階乗しか知りませんので。
・・・
自分で勉強します。ごめんなさい
212132人目の素数さん:2001/04/07(土) 20:32
>>208
有限個の石であれば
奇数のほうが若干確率が高いな。
213132人目の素数さん:2001/04/07(土) 21:21
m×nのマス目がある。次の条件を満たすように各マスを黒または白に塗る。

条件:すべての黒マスについて、そのマスに隣接する黒マスの個数は奇数である。

このとき、黒マスの総数は、偶数であることを示せ。ただし、2つのマスが隣接するとは、それらが異なり、かつ一辺を共有することである。


やり方と答え教えて
214132人目の素数さん:2001/04/07(土) 21:27
215132人目の素数さん:2001/04/07(土) 21:30
各黒マスが隣接する黒マスの個数を、全黒マスについて足した総数を考える。
隣接する黒マスのペアに対して1つの隣接する辺があるので、総数は全黒マスの個数の2倍、つまり偶数。
黒マスの個数が奇数個だとすると、先程の総数は 奇数*奇数=奇数となり、矛盾する。
216132人目の素数さん:2001/04/07(土) 21:36
既出だがlogがみつからない
217132人目の素数さん:2001/04/07(土) 21:39
各黒マスが隣接する黒マスの個数を

各黒マスが隣接する黒マスの個数とその黒マス自体をあわせた個数
じゃないの?
218132人目の素数さん:2001/04/07(土) 21:43
反例
□■□□
■■■■
□□■□
219132人目の素数さん:2001/04/07(土) 21:46
>>215
全黒マスの個数じゃなくて、隣接辺の総数の2倍だろう
220132人目の素数さん:2001/04/07(土) 22:26
【今年の数学オリンピック】
55 名前:132人目の素数さん投稿日:2001/02/11(日) 19:33
>>54 本選の問題ではないよね?
隣接する黒桝の組の数を p とする。
黒桝が q 個、うち隣接する黒桝が1個のもの r 個とすれば
2p=1*r+3*(q-r)=3q-2r より、 3q = 2(p+r) なので q は偶数。
221132人目の素数さん:2001/04/08(日) 07:09
テンソルってなに?
222132人目の素数さん:2001/04/08(日) 07:51
>テンソル

おおざっぱには、多重線型写像
223132人目の素数さん:2001/04/08(日) 15:24
定期age
224132人目の素数さん:2001/04/08(日) 16:48
実数を係数とする整式f(X)g(X) h(X) にたいして
P(X)={f(X)}^3+{g(X)}^3+{h(X)}^3−3f(X)g(X)h(X)
Q(X)=f(X)+g(X)+h(X) とおく
(1)P(X)はQ(X)で割り切れることを示せ
(2)aを実数とするとき
  P(X)は(X−a)^2で割り切れるが(X−a)^3で割り切れないものとする
  このときQ(X)がX−aで割り切れるならば、(X−a)^2でも割り切れることを示せ

この問題の(2)をお願いします
(1)はa^3+b^3+c^3-3abc=(a+b+c)(a^2+b^2+c^2-ab-b-ca)の公式
  でで来たと思うんですが(2)がうまくいかないのです
225y^2=x^3+ax+b:2001/04/08(日) 17:32
P(x) = { f(x) + g(x) + h(x) }[ {f(x)}^2 + {g(x)}^2 + {h(x)}^2 - f(x)g(x) - g(x)h(x) - f(x)h(x) ]
と因数分解できるので
R(x) ≡ {f(x)}^2 + {g(x)}^2 + {h(x)}^2 - f(x)g(x) - g(x)h(x) - f(x)h(x)
   = (1/2) * [ {f(x)-g(x)}^2 + {g(x)-h(x)}^2 + {h(x)-f(x)}^2 ]
と定義し
P(x) = Q(x)R(x)
と書く
条件より、Q(x)は適当な実数係数多項式q(x)を取ってきて
Q(x)=(x-a)q(x)
と書けるので
(x-a)^2 | R(x)q(x)(x-a) かつ (x-a)^3 メ R(x)q(x)(x-a)

(x-a) | R(x)q(x) かつ (x-a)^2 メ R(x)q(x)
メは「割り切らない」の記号のつもり

(x-a) | R(x) と仮定すると
f(x), g(x), h(x)は実数係数の多項式より
R(a)=0 ⇔ f(a)=g(a)=h(a)
ところが、(x-a) | Q(x) より
f(a)+g(a)+h(a)=3f(a)=0
よってf(a)=g(a)=h(a)=0より
f(x), g(x), h(x)は(x-a)を因数に持つが、そうすると
R(x)が(x-a)^2を因数に持ってしまい矛盾
よって
(x-a) | q(x)
つまり
(x-a)^2 | Q(x)
226132人目の素数さん:2001/04/08(日) 20:42
age
227132人目の素数さん:2001/04/08(日) 20:54
>>>225>>224の解答です
228132人目の素数さん:2001/04/08(日) 21:05
>>224
(やってることは[>>225]と同じだとおもうけど…)
R = f + wg + (w^2)h
S = f + (w^2)g + (w^4)h
とおく。但し w は w^2 + w + 1 = 0 の根とする。
このとき P=QRS が成り立つ。ところで
f, g, h はすべて実係数で w(=w^4)とw^2 は互いに共役。
よって実数 a に対して「R(a)=0 ⇔ S(a)=0」である。
したがって P が a で2重の零で Q(a)=0 ならば
Q も a で2重の零となる。
229132人目の素数さん:2001/04/09(月) 15:06
ε-δ論法で厳密に極限とか微積分は定義されていると言い切っていいすか?
230132人目の素数さん:2001/04/09(月) 17:01
225さん228さんありがとうございました
231132人目の素数さん:2001/04/09(月) 23:18
>>229
いいかどうかはわかんないけど
ε-δ使えば明快にはなるとおもいます
232Mr.Bear:2001/04/10(火) 00:33
剰余定理、因数定理の問題だと思うのですが、全くわかりません。
教えて下さい!

整数を係数とするxの整式Aを
x^3+x^2+x+1で割ると余りは-3x^2-x+2であり、
x^2+2x+3で割ると余りは5x+3であるという。
このようなAの中で、次数が最小のものを求めよ。
233Z会添削指導員:2001/04/10(火) 01:03
224にある問題は、Z会MA4−1旬の問題で、まだ締め切りがきていないので、できればコメントしないようお願いいたします。
234↑言っても無駄:2001/04/10(火) 01:11
こんなもん日常茶飯事だよ。Z会にタレコミすれば?
235234:2001/04/10(火) 01:12
でもそういう情報はありがたいです
236tr:2001/04/10(火) 01:30
>>232 = Mr.Bear さん
B = x^3 + x^2 + x + 1, C = x^2 + 2x + 3
A を B で割った余りを Q(x) として,
  A = B*Q(x) + (-3x^2 - x + 2)
    = {C*(x - 1) + 4}Q(x) +{-3*C + (5x + 11)}
    = C*{(x - 1)Q(x) - 3} + {4Q(x) + 5x + 11}
なので, 第2項を C で割った余りが 5x + 3 となればよく
もとめる最小次数の A は, Q(x) = a (定数) として
  4a + 5x +11 = 5x + 3 ⇒ a = -2
  ∴ A = B*(-2) + (-3x^2 - x + 2)
      = -2x^3 - 5x^2 - 3x

# 剰余定理も因数定理も使ってません(汗)
237Mr.Bear:2001/04/10(火) 01:57
>>236 = trさん
レスありがとうございます。助かりました!!
ただすべてを理解するには至りませんので
質問よろしいでしょうか?

A を B で割った余りを Q(x) として,

A を B で割った商を Q(x) として,
ですよね?
で、5行目の
= {C*(x - 1) + 4}Q(x) +{-3*C + (5x + 11)}
の意味がわからないんですけど…。
238tr:2001/04/10(火) 02:10
>>237 = Mr.Bear さん
修正ありがとうございます。<余り →商

ええと, 実際に割り算すると
  B = C*(x - 1) + 4
  -3x^2 - x + 2 = -3*C + (5x + 11)
となるから書き換えたんですよ。
239Mr.Bear:2001/04/10(火) 02:24
>>238 = trさん
なるほど!!
簡単に言うと
y=3x+10
y=5x+2
があったとしたら
y=5x+2
=3x+(2x+2)
として
2x+2=10

x=4
ってわけですね!(^-^)v
240tr > Mr.Bearさん:2001/04/10(火) 02:35
そんな感じです。^-^)b バッチシ!
241132人目の素数さん:2001/04/10(火) 17:51
くだらないスレッドに書いたのですが、スレッドが少し違ったようなので、クロスポストご容赦ください。

p/q(ただし、pとqは互いに素)が有限小数で表せるとき、1/qが無限小数になる(有限小数で表せない)ことってありますか?
242ご冗談でしょう?名無しさん:2001/04/10(火) 19:01
あるにきまっているだろ
ばーか
243132人目の素数さん:2001/04/10(火) 19:20
>>242
じゃあ例を出して見ろよ。
ばーか。
244ご冗談でしょう?名無しさん:2001/04/10(火) 19:24
じぶんでさがせ
ばーか
245132人目の素数さん:2001/04/10(火) 19:33
互いに素ってどういう意味ですか??
246:2001/04/10(火) 20:05
ふたつの整数 p, q に対して、「pとqは互いに素」とは
「pとq の公約数であるような自然数は 1 だけだ」ということ。
247>245:2001/04/10(火) 20:06
1しか共約数がないってことです.p/qが既約ってことです.
248132人目の素数さん:2001/04/10(火) 20:07
>>245
ヴァカ?
249247:2001/04/10(火) 20:07
かぶった.しかも,公約数だった.
250246:2001/04/10(火) 21:00
>>241
>p/q(ただし、pとqは互いに素)が有限小数で表せるとき、
>1/qが無限小数になる(有限小数で表せない)ことってありますか?

答:ない。

ふたつの自然数 p, q に対して
p/q が有限小数で表せるということは
p/q = a/(10^n) となる 自然数 a, n があるということ。
ここでさらにこの p, q が互いに素だとすると
1/q = b/(10^m)
となる 自然数 b, m があるということが
ちょっと考えればわかる。
251132人目の素数さん:2001/04/10(火) 21:01
5と7は互いに素である
252132人目の素数さん:2001/04/10(火) 21:13
>>241
   p/qが有限小数
 ⇔ p/qが小数点以下n桁で表せる
 ⇔ (p*10^n)はqの倍数

pとqは互いに素だから
10^nがqの倍数になって
1/qも小数点以下n桁以内で表せるので
無限小数になることはない□

なんか不安・・・適切な表現に直してkure
253132人目の素数さん:2001/04/10(火) 21:15
かぶり負け
まちがってなsageだったので安心sage
254>250, 252:2001/04/10(火) 21:22
よく分かりました.ありがとう.
255ご冗談でしょう?名無しさん:2001/04/10(火) 21:31
>>252

ばーか
256132人目の素数さん:2001/04/10(火) 22:03
[248:132人目の素数さん(2001/04/10(火) 20:07)]>>245
ヴァカ?

ヴァカってどういう意味ですか?『あなたはヴァですか?』の略ですか?
257132人目の素数さん:2001/04/10(火) 22:32
242=244=255
物理板から電波が届いている模様。
証明が理解できない厨房はキエテクダサイ
258132人目の素数さん:2001/04/10(火) 22:39
答えを教えて
ここに 普通に市販されている渦巻き型の蚊取線香かある。この香取り線香は60分キッカリで燃え尽きる。
この香取り線香 2本を使って、45分計りたい。どうすればよいでしょう? ただし、香取り線香は折ったり してはいけない。
わかんね〜????
259132人目の素数さん:2001/04/10(火) 22:48
>>258
片方の線香にはお尻と頭と両方に火を付けて
片方の線香には頭にだけ火を付ける。

両方に火を付けた線香は30分で燃え尽きるので、
燃え尽きた時点でもう片方のお尻に火を付けると
ちょうど15分で燃え尽きる。都合45分。
260ご冗談でしょう?名無しさん:2001/04/10(火) 23:23
>>259
ばーか
261132人目の素数さん:2001/04/10(火) 23:30
242=244=255=260
262132人目の素数さん:2001/04/11(水) 01:28
>>260
馬鹿は消えてください。
このスレに無意味な書き込みをしないだけ
嵐山の方が「まだ」ましです。
>>259
一応後半部をもう少し詳しく書くと、
片方の線香は半分だけ残っているので反対側に火をつけると
その半分の時間で燃え尽きるということです。
駄レスです。
263ご冗談でしょう?名無しさん:2001/04/11(水) 01:42
>>262
ばーか
37分30秒だろうが
264132人目の素数さん:2001/04/11(水) 02:23
>>260

>>4を見ろ
ばーか
265242=244=255=260=263の母:2001/04/11(水) 02:26
242=244=255=260=263の母です。あの子に悪気はなかったんです。
根は良い子なんです。許してageて下さい。
266262:2001/04/11(水) 02:39
>>263
はっ?
いつから
30分/2=7分30秒になったのですか?
>>264
私に対するですね?
見落としてました。

ところで
>>258 の条件(ただし線香の数は自由)で計れる1時間以下の時間は
60*(1-(1/2)^n)分
だけなのでしょうか?
267259:2001/04/11(水) 04:22
>>266

バカは相手にしないようにしませう。

> 60*(1-(1/2)^n)分

おそらくそうでしょう。
基本的に火を付けられるのがお尻と頭だけですから。
たとえば、三十分を計って他の線香と比べて、
真ん中の火が付けられるとかならもーちょっとバリエーションがあるかも。
268132人目の素数さん:2001/04/11(水) 11:15
>>266
いえ。一時間以下だったら60分も存在します。
269132人目の素数さん:2001/04/11(水) 13:03
有界と有限はどう違うんですか?
270132人目の素数さん:2001/04/11(水) 15:08
次の級数の収束・発散を判定せよ。
Σ[n=1,∞] 1/n(2n-1)

答え、これで合ってますか? 教えてください。
a(n+1)/a(n)=n(2n-1)/(n+1)(2n+1)<1
よって、ダランベールの判定法により級数は収束する。
27172*5人目の素数さん:2001/04/11(水) 16:22
定数項及び係数をabcdとする4次方程式について簡単な解のようでむずかしくて解けない
与式、f(x)=x^4+a(x^3)+b(x^2)+cx+d=0 の1つの解が (√3)+i の時、残りの3解を求めよというので
まず2つ目は解の対称性より(√3)-iとでる。
f((√3)+i)-f((√3)-i)= 16ai+4(√3)bi+2ci=0 となるから
abcdの関係式を作り連立で解こうとして
{x-((√3)+i)}*{x-((√3)-i)}=(x^2)-2(√3)x+4より
与式をこれで割ると商は(x^2)+(a+2√3)x+(2√3a)+b+8
余りは(8a+2√3b+c+8√3)x-(8√3a+4b+32)
となり商の方程式の2解が残りの解でその解はx=a+2√3±√・・・・・/2
また余り=0より最初に出た式を用いて連立しようとして
わけがわからなくなりました
お願いします
272132人目の素数さん:2001/04/11(水) 16:39
a(n+1)/a(n)=n(2n-1)/(n+1)(2n+1)
=2-1/n / (1+1/n)(2+1/n) →(n→∞)→1
この判定法では収束発散はわからない
オイラー・マクローリンの判定法を使う
∫[1,u]1/x(2x-1)dx=log{(2u-1)/u} →(u→∞)→log2
広義積分は収束する。よってこの級数は収束する。
273132人目の素数さん:2001/04/11(水) 17:06
(x^2-2√3x+4)(x^2+tx+s)=x^4+ax^3+bx^2+cx+d
とおいて、左辺バラして両辺比較すると、
4s=d、t=a+2√3
27472*5人目の素数さん :2001/04/11(水) 17:59
>273
といっても余計な変数を増やしただけで答えにつながりません。
ちなみにその方法で
t-2√3=a s-2√3t+4=b -2√3s+4t=c 4s=dとなり
残りの2解をαβとしてα+β=t αβ=sとして
x^4+ t-2√3*x^3 + s-2√3t+4*x^2 + -2√3s+4t*x + 4s
にx=√3+i,√3-iを代入して2つの方程式から解を出そうとしても
代入した段階で0になりますね。
27572*5人目の素数さん:2001/04/11(水) 18:03
訂正
-2(α+β)=tでした
276132人目の素数さん:2001/04/11(水) 18:06
未知数が4つで、与えられた解が2つだから、
残りの2解には未知定数が2つ含まれる
277132人目の素数さん:2001/04/11(水) 18:25
>>271
a,b,c,dに関する条件は無いんですか?
整数(有理数)等が無いと解は決まらないっぽい・・・
27872*5人目の素数さん :2001/04/11(水) 18:25
残りの解は未知定数2個を含む式で表されるということですか
ということはabcdを含んだ形でで解を表す?
{t-2√3=a s-2√3t+4=b -2√3s+4t=c 4s=d(t=-2(α+β),s=αβ)}
から導けるような気がするができません


27972*5人目の素数さん:2001/04/11(水) 18:27
有理数abcdでした
280132人目の素数さん:2001/04/11(水) 18:28
a,b,c,dは実数という条件が無ければ
(√3-i)も解になるとは言えないので・・・
281132人目の素数さん:2001/04/11(水) 18:41
残りの2解はx^2+tx+s=0 の解。解の公式と
4s=d、t=a+2√3
を使えばa、dを含んだ答えになる
282132人目の素数さん:2001/04/11(水) 18:48
>>274
> ちなみにその方法で
> t-2√3=a s-2√3t+4=b -2√3s+4t=c 4s=dとなり

もうちょっとです。
a,b,c,dが有理数になるにはsとtが・・・と考えてください。
sとtが決まります。
28372*5人目の素数さん:2001/04/11(水) 18:52
定数項及び係数を有理数abcdとする4次方程式f(x)=x^4+a(x^3)+b(x^2)+cx+d=0 の1つの解が (√3)+i の時、
残りの3解を求めよ(ちなみに佛教大)
まず2つ目は解の対称性より(√3)-iとでる。
残りは281の方法でx={(-a-2√3)±√(a^2+4√3a+12-d)}/2
でいいの?
284132人目の素数さん:2001/04/11(水) 19:00
>>283
入れ違いですね。
sとtは一意に決まります。

> t-2√3=a s-2√3t+4=b -2√3s+4t=c 4s=dとなり

第1式よりt=a+2√3
第4式よりs=d/4

これらを第2、第3式に入れてsとtを消去し
A+B√3=0 のかたちにします
A、Bが有理数ならA=B=0なので(以下略)
28572*5人目の素数さん:2001/04/11(水) 19:16
>282
t=2√3 s=4 で
x^2+2√3x+4
となり
結局(√3)±iの重解がこたえですか?
28672*5人目の素数さん:2001/04/11(水) 19:20
>284
A+B√3=0→A、Bが有理数ならA=B=0
というのを学校でならった記憶が無い
というより有理数という概念と条件を
持ち込んでとくことがあまりなかったかもしれない。

287y^2=x^3+ax+b:2001/04/11(水) 19:33
実数α=a+b√3 (a, b∈Q+Qi iは虚数単位)
に対して
conj(α)=a-b√3

複素数β=c+di (iは虚数単位, c, d∈R)
に対して
_
α=c-di

と定義すると(括弧内の条件からa, b, c, dは一意に決まる)
これらは共役となる
実際計算すると
conj(α) * conj(γ) = conj(αγ)
conj(α) ± conj(γ) = conj(α+γ)
が成り立つ
共役複素数に関してはご存知の通り

以上の道具を使っていく

f(x)=x^4+a(x^3)+b(x^2)+cx+d=0 の1つの解が (√3)+i
より、α = (√3)+i
と置くと
α^4+a(α^3)+b(α^2)+cα+d=0・・・(*)
が成り立つので、両辺共役とって
_   _    _   _
α^4+a(α^3)+b(α^2)+cα+d=0・・・(**)
    _  _
よってαはf(α)=0を満たすので解である

また、(*)の両辺conj()をとって
(conj(α))^4+a(conj(α))^3+b(conj(α))^2+c*conj(α)+d=0
となり、conj(α)も解となる

また、(**)の両辺conj()をとって
   _
conj(α)
も解となる

以上から4つの解 √3±i, -√3±i
が求まる。
よって、
(x-√3-i)(x-√3+i)(x+√3+i)(x+√3-i)
=x^4-8x^2+16
となりa,b,c,dが求まった
288y^2=x^3+ax+b:2001/04/11(水) 19:33
>>287
一行目実数じゃなかった
訂正
28972*5人目の素数さん:2001/04/11(水) 19:34
やっぱり佛教大だから
はじめから解の対称性と一言で済ませば
10秒で解ける問題だったんですね
290132人目の素数さん:2001/04/11(水) 19:35
>>285
>結局(√3)±iの重解がこたえですか?

-(√3)±iですね
(書き損じだったらスマソ)

>>286
自分も遠い昔のことなんで・・・
っていうかもっとスマートな解き方があるに違いない

夜中に出入りしてる人達が詳しいので
疑問を書いて寝かせておくと明晩にはレスが付いてるでしょう
29172*5人目の素数さん:2001/04/11(水) 19:42
またニアミスしてしまった
±√3±iだった
292270:2001/04/11(水) 19:42
>>272さん、レスありがとうございます。

オイラー・マクローリンの判定法はまだ習っていないのですが、
0<1/(2n^2-n)<1/n^2で、1/n^2が収束するということで示すことはできますか?
293132人目の素数さん:2001/04/11(水) 20:02
もう一度>>271を読んでみたら・・・

> {x-((√3)+i)}*{x-((√3)-i)}=(x^2)-2(√3)x+4より
> 与式をこれで割ると商は(x^2)+(a+2√3)x+(2√3a)+b+8
> 余りは(8a+2√3b+c+8√3)x-(8√3a+4b+32)

ともに√3+iを解に持つf(x)=0と((x^2)-2(√3)x+4)=0に対して
f(x)を((x^2)-2(√3)x+4)で割ったのだから
その余り=0がxに関して恒等的に成り立つ

> 余りは(8a+2√3b+c+8√3)x-(8√3a+4b+32)

dが出てこないのでこの余りの式はまちがってるけど
正しい余りの式をpx+qとするとp=q=0という2式を得る

a,b,c,dが有理数であることから
2式をA+B√3=0に書き直して(以下略)

というわけで最初の方針でもなんとかなりました。
294132人目の素数さん:2001/04/11(水) 20:08
> また余り=0より最初に出た式を用いて連立しようとして

って書いてありましたね。スマソ
295132人目の素数さん:2001/04/11(水) 20:26
>>292
それでもいいと思います。
ただし、Σ1/n^2 が収束することの証明をきちんとしなくては。
An=1+1/2^2+・・・+1/n^2 とおくと、数列{An}がCauchy列になることから証明できます。
296π:2001/04/11(水) 21:21
何年か前にアメリカでこんな問題が議論を呼びました。数学者の中にも
正解を得られない人がいたとか・・・。


3枚の封筒のうち1枚だけに、あなたが行きたいと思っているコンサー
おチケットが入っています。どの封筒に入っているか当てたら差し上げ
ましょう。ただくじびきするだけえはおもしろくないので、次のように
してください。まず、3枚の中から好きなものをいれてください。あな
たが選ばなかった2枚の封筒のうち少なくとも一方はチケットが入って
いませんから、それを私が教えてあげます。そのあと、もう一度封筒を
選びなおすチャンスを選びますか、それとも残された封筒を選びますか
。根拠も示してください。


297π:2001/04/11(水) 21:53
何年か前にアメリカでこんな問題が議論を呼びました。数学者の中にも
正解を得られない人がいたとか・・・。


3枚の封筒のうち1枚だけに、あなたが行きたいと思っているコンサー
おチケットが入っています。どの封筒に入っているか当てたら差し上げ
ましょう。ただくじびきするだけえはおもしろくないので、次のように
してください。まず、3枚の中から好きなものをいれてください。あな
たが選ばなかった2枚の封筒のうち少なくとも一方はチケットが入って
いませんから、それを私が教えてあげます。そのあと、もう一度封筒を
選びなおすチャンスをあたえます。さ、最初に選んだ同じ封筒を選びま
すか、それとも残された封筒を選びますか 。根拠も示してください。


298π:2001/04/11(水) 22:10
何年か前にアメリカでこんな問題が議論を呼びました。数学者の中にも
正解を得られない人がいたとか・・・。


3枚の封筒のうち1枚だけに、あなたが行きたいと思っているコンサー
おチケットが入っています。どの封筒に入っているか当てたら差し上げ
ましょう。ただくじびきするだけえはおもしろくないので、次のように
してください。まず、3枚の中から好きなものを選んでください。あな
たが選ばなかった2枚の封筒のうち少なくとも一方はチケットが入って
いませんから、それを私が教えてあげます。そのあと、もう一度封筒を
選びなおすチャンスをあたえます。さ、最初に選んだ同じ封筒を選びま
すか、それとも残された封筒を選びますか 。根拠も示してください。


すんません、ミスしまくりで。
299132人目の素数さん:2001/04/11(水) 22:29
>>298
超がいしゅつ。
もう見飽きたよ、この問題は・・・
300132人目の素数さん:2001/04/11(水) 22:39
しかもまだミスしている。
301π:2001/04/11(水) 22:45
教えてください・・・
302132人目の素数さん:2001/04/11(水) 22:49
コンサーおチケット

くじびきするだけえは
303132人目の素数さん:2001/04/11(水) 23:02
>>302
コンサーおチケットって普通に使うよ。
知らない??
304π:2001/04/11(水) 23:04
みんなわかんないんだ。ばかだなあ
305132人目の素数さん:2001/04/11(水) 23:16
うn
306132人目の素数さん:2001/04/11(水) 23:20
次の方程式を満たす角θ(0°<θ<90°)を求めよ.
 sin60°sin(30°+θ) = 2sin280°sinθ
307132人目の素数さん:2001/04/11(水) 23:24
>>306
東京出版のページで出されてる問題(の写しまちがえ)
相手にしないよーに
308132人目の素数さん:2001/04/11(水) 23:32
>>306
左辺>0 右辺<0 ∴解ナシ(w
309303:2001/04/12(木) 00:48
>コンサーおチケットって普通に使うよ。
>知らない??

今更ながらだが、
うそなんだけどね。みんなごめんね。だまして。>特にπ
310132人目の素数さん:2001/04/12(木) 01:17
誰かダマされてたの?(ワラ
311132人目の素数さん:2001/04/12(木) 01:30
>>307
>写しまちがえ
って、活用おかしいですよ。2ch用語だったらスマソ。
数学と関係無いのでsage
312132人目の素数さん:2001/04/12(木) 01:34
>>311
「まちがえ」も「まちがい」もおかしくはないよ
なんだったら語学板かどっかで聞いてみなんしょ
313132人目の素数さん:2001/04/12(木) 01:52
まちがう → まちがい
まちがえる → まちがえ
314tr:2001/04/12(木) 01:54
@307 > の写しまちがえ
と直前に 「の」 があるから 「まちがい」 と続けたほうがいいかも?(謎)
315高校生:2001/04/12(木) 02:03
xの3次式p(x)があってp(1)=3とおくp(x)+3 が(x+a)^2で割り切れ
p(x)-3 が(x-a)^2で割り切れるときp(x)を求めよと言うので
p(x)+3=(x+a)^2*(sx+t) → p(x)=sx^3+(2as+t)x^2+(sa^2+2at)x+ta^2-3
p(x)+3=(x-a)^2*(px+q) → p(x)=px^3+(-2ap+q)x^2+(pa^2-2aq)x+qa^2+3
係数比較から
p=s を経て 4pa^3=-6 にたどりつく
またp(x)+3=0 が x=-a で重解を持つと言うことです。
つまり、y=p(x) のグラフは、x=-a で極値 -3 を持ちます。
同様に x=a で極値 3 を持ちます。
よってa>0としてp´(x)=-(x+a)(x-a)=-x^2+a^2
p(x)=-(x^3)/3+(a^2)x+C
p(1)=(-1/3)+a^2+C=3 ∴C=(10/3)-a^2
∴p(x)=-(x^3)/3+(a^2)x+(10/3)-a^2
となったんですが
これでもaを消去できません消去しないまま答えになるんでしょうか
316高校生:2001/04/12(木) 02:11
p(a)=3 →2a^3-6a^2+1=0
p(-a)=-3 →2a^3+3a^2-19=0
aだけの3次式に導けました。
317高校生:2001/04/12(木) 02:17
ですが2通りの式が出ることはありえないので
後者の方針は脱線ですかね、もともと方程式の分野ですから
318132人目の素数さん :2001/04/12(木) 03:07
>>315
>よってa>0としてp´(x)=-(x+a)(x-a)=-x^2+a^2

ここで間違えているんよ。
p(x)=sx^3+… なので、p'(x)=3sx^2+… のはずだから、
p'(x)=3s(x+a)(x-a)
と置くのが正しい。

係数比較でやるなら、そのまま、
s=p
as+t=-2ap+q
sa^2+2at=pa^2-2aq
ta^2-3=qa^2+3
を解く方がよさそう。なお、a も求まり答えは二つあるようです。
319132人目の素数さん:2001/04/12(木) 03:50
y=3x2乗-6x+1の頂点の座標の出し方教えてください
320132人目の素数さん:2001/04/12(木) 04:00
完全平方すると、

y=3(x-1)2乗-2

となるから、頂点の座標は、(1,-2) です。
っていうか、このくらい教科書か参考書で調べたらわかるでしょ。
321320:2001/04/12(木) 04:11
×完全平方すると
○平方完成すると
322132人目の素数さん:2001/04/12(木) 05:51
>>320
ありがとうございました。
…好きです
323132人目の素数さん:2001/04/12(木) 06:05
>>270, >>292
1/n(2n-1) < 1/n(n-1) = 1/(n-1) - 1/n
を使えば簡単
324303:2001/04/12(木) 09:28
なんてね。
ほんとはコンサーおチケットって普通に使うよ。
325132人目の素数さん:2001/04/12(木) 12:54
あげ
326π:2001/04/12(木) 16:16
何年か前にアメリカでこんな問題が議論を呼びました。数学者の中にも
正解を得られない人がいたとか・・・。


3枚の封筒のうち1枚だけに、あなたが行きたいと思っているコンサー
トチケットが入っています。どの封筒に入っているか当てたら差し上げ
ましょう。ただくじびきするだけではおもしろくないので、次のように
してください。まず、3枚の中から好きなものを選んでください。あな
たが選ばなかった2枚の封筒のうち少なくとも一方はチケットが入って
いませんから、それを私が教えてあげます。そのあと、もう一度封筒を
選びなおすチャンスをあたえます。さ、最初に選んだ同じ封筒を選びま
すか、それとも残された封筒を選びますか 。根拠も示してください。


すんません、ミスしまくりで。

327270:2001/04/12(木) 17:40
>>323
なるほど〜。そんな簡単にできたんですね。
お答え、ありがとうございます。
328y^2=x^3+ax+b:2001/04/12(木) 18:02
>>287>>271の解です
長々と書いたから無視されたかも・・・
329犬釘:2001/04/12(木) 23:04
>326

応用問題を。

前提は以下の3点。
・3本のくじ引きを3人で順番に引く。
・当たりは1本しかない。
・あなたは2番目に引く。

何の気なしに引けば、当たりを引く確率は(2/3)*(1/2)で1/3になる。
しかし

・1人目が引く前にあなたは1本のくじに(心の中で)しるしをつけておく。
・1人目がハズレを引いてあなたの番が回ってきた時に、しるしをつけたくじが残っていたとする。

この条件を追加すると>326と同じ要領で
しるしをつけたくじではなく、
逆を選んだ方が当たる確率は高くなる。

このことから、
心の中でしるしをつけるという行為が確率の変動に関わる、
という結論が得られる。

・・・・・・

以上、間違いを指摘してください。
特にπ氏に
330ご冗談でしょう?名無しさん:2001/04/12(木) 23:38
ラッセルのパラドックスを説明して下さい。
331132人目の素数さん:2001/04/13(金) 00:15
「しるしをつけたくじが残ってい」る確率は?
332高校生:2001/04/13(金) 00:17
もういいです
頼みません
2度と
自力で解きゃあいいんだろ
しょうがねえなあ
三日連続徹夜で
死ぬほど睡魔に襲われるし、
ひどい頭痛とめまいがする
ああこんな問題があるとは
333132人目の素数さん:2001/04/13(金) 00:24
>頼みません
>2度と

いい心掛けだ
334132人目の素数さん:2001/04/13(金) 00:41
>>332
数学の部屋掲示板でちゃんとレスついたじゃん。
なんで続きをそっちで聞かないのかわからん。
(ヨッシー氏のレスを反古にするとはおそれおおい奴・・・)

それにここでも解き方の指針レスついたじゃん。
答えだけが欲しかったのかい?

ほれ。p(x)=(-3x^3+9x)/2 , 12x^3-9x
335おっπ:2001/04/13(金) 01:13
>π
どれも1/2じゃねえの?
違うの?頭悪いからわからん
336ハミπ:2001/04/13(金) 01:31
>おっπ
1/3と2/3とちゃう?
337たれπ(電話可能):2001/04/13(金) 01:42
どうなの?
338綾小路π:2001/04/13(金) 04:02
>・1人目が引く前にあなたは1本のくじに(心の中で)しるしをつけておく。
>・1人目がハズレを引いてあなたの番が回ってきた時に、しるしをつけたくじが残っていたとする。
>この条件を追加すると>326と同じ要領で
>しるしをつけたくじではなく、
>逆を選んだ方が当たる確率は高くなる。

ここ全然理解できん。なんで逆を選んだ方が当る確率は高くなるのか
説明して。
339>330:2001/04/13(金) 10:22
 ある図書館に、そこにあるすべての図書を記載した図書目録があるとします。
その図書目録は、なぜか二分冊になっていて、「目録A」と「目録B」があるとします。
ここがいかにも作為的ですね。怪しげですね。騙されないように気をつけて読んでください。
 目録Aには、書中のどこかで自分自身についての記述をしている図書をすべて収録します。
こんな馬鹿なことを本当にやるとしたら、図書館の職員さんは死ぬほど大変でしょうね。
あくまで仮定の話です。目録Bには、そうでない図書、
すなわち自分自身についての記述をいっさいしていない図書をすべて収録します。
これで図書館にあるすべての図書は目録AかBかに必ず収まることになります。
 ところで、目録Aと目録Bも図書館の蔵書です。だから、この二冊も目録の中に記載しておかなければなりません。
目録Aは、目録Aに記載します。これはいいですね。だって、これで、目録Aは自分自身に言及していることになるし、
目録Aには、自分自身に言及した図書のみを収録することになっているのですから。
 さて、目録Bはどうでしょうか。
目録Bを目録Aに記載してみましょう。
 すると、目録Aのきまりによって、目録Bは自分自身に言及した図書でなければなりません。
 でも、自分自身を記載していないのだから、これには違反しています。
では、目録Bは目録Bに記載すべきでしょうか。
でも、そうしてしまうと、目録Bに自分自身に言及した図書を収録することになって、
やはり違反です。アレーッ、これは困ったことになったぞ。
目録Bを図書として登録する場所がないじゃないか。目録AとBで図書館の図書は全部収録できると言ったのに。

−−−−−−−−−−−−−−−−
ってのは、どっかの大学の先生が、した例え話。
この話の書物を集合と置き換えてみよう。
340高校生:2001/04/13(金) 13:35
指針あってたのに
解けないのはなぜ?
計算力無いから??これが数学偏差値69の力か??
マア文系だから数学は偏差値69だけでいいし
341132人目の素数さん:2001/04/13(金) 13:44
>>340

もう来ないんじゃないの?
342132人目の素数さん:2001/04/13(金) 13:57
>>340
おまえの偏差値などどうでもいい。
教えてもらって礼も言えんのか?
↓こっちでも書きっぱなしだよな。
http://diver.miffy.to/freebbs/mkres5.cgi?aoki
343132人目の素数さん:2001/04/13(金) 14:21
>>340
偏差値って母集団によるからなぁ(ワラ
344132人目の素数さん:2001/04/13(金) 14:30
>解けないのはなぜ?

馬鹿だからに決まってんじゃン
345がんばれ高校生:2001/04/13(金) 14:52
>>315
318 がほとんど答えだよ。
結局 p は x = a で極大値 3 x=-a で極小値 -a の3次関数なので原点対称
であることがわかる。このあと一般に原点対象の3次関数が極大(または極小)
を取る点が原点からみて反対側の2倍いったとこ(本問の場合 x = -2a で 3
x = 2a で -3) だというわりと受験数学でよくでてくるやつをしってれば
条件 p(1) = 3 からこの 1 ってのは a か -2a のどっちかとわかる。
ゆえに p(x) = c(1/3x^3 - 1/4) または p(x) = c(1/3x^3 - 1) なる
形までもってこれてあとは条件 p(1) = 3 から c をもとめればよいです。
...参考までに...
3倍角の公式ででてくる多項式 f(x) = 4x^3 - 3x は原点対称で他の
原点対称3次関数はこれを縦横に何倍かひきのばしたものです。
f(-1/2) = f(1) = 1, f(1/2) = f(-1) = -1 です。受験でよくでる
3次関数なのでグラフに書いて記憶の片隅にかすらしておくといいかも。
346がんばれ高校生:2001/04/13(金) 14:57
>345
訂正
誤 c(1/3x^3 -1/4)
正 c(1/3x^3 - 1/4x)
誤 c(1/3x^3 -1)
正 c(1/3x^3 -x)
347132人目の素数さん:2001/04/13(金) 15:49
>>315=高校生
バッシングが気になるなら数学の部屋掲示板に戻った方がいいですよ。(^^;

>係数比較でやるなら、そのまま、
>s=p
>as+t=-2ap+q
>sa^2+2at=pa^2-2aq
>ta^2-3=qa^2+3

318氏の指針通りに進めるなら、
この4つとf(1)=3から得られる5つの式から
5つの未知数a,p,q,s,tを出せるはずです。

解けなかったのはどこかに計算ミスがあったのでしょう。
f(1)=3を使うときはp(x)-3=(x-a)^2*(px+q)に代入した方が
ちょっと得です。

機械的な計算の前に
345氏や最初に教えてくれたヨッシー氏のような
グラフのイメージをつかむことができれば
p(x)が奇関数になることや答えは2つあるはずだという
見通しが立ちます。

(別解?)
「f(x)が(x+a)^2で割り切れればf ’(x)は(x+a)で割り切れる」
というのを知っていれば、本問ではa=0が不適なので
(p(x)±3)’=p ’(x)は(x±a)で割り切れることがすぐに見えて
p(x)=∫s(x+a)(x-a)dx=s(x^3)/3-s(a^2)x+tと置くことができます。
あとはp(a)=p(1)=3,p(-a)=-3と連立させれば
s,t,aが求まってp(x)が決まります。
348文系:2001/04/13(金) 16:19
コレを聞こうと思って 数学版にやってきた文系なんだけど
マイナス1とマイナス1を掛けると 何で プラスになるの?
子供に聞かれても 答えられないよーー。
349123人目の素数さん:2001/04/13(金) 16:31
350123人目の素数さん:2001/04/13(金) 16:34
偏差値69で、こんな問題が解けないなんて
酷い世の中になったもんだね
偏差値50くらいだと掛け算できなかったりしてな(藁
351132人目の素数さん:2001/04/13(金) 21:26
>>348
これで納得するかどうかはしらんが。
(-1)*(-1) = (-1)*(1/(-1)) = (-1)/(-1) = 1
352132人目の素数さん:2001/04/13(金) 22:26
歴史は繰り返す・・・
353132人目の素数さん:2001/04/13(金) 23:24
1/(-1)=-1
が言えるのは(-1)*(-1)=1を前提にしているのでは?
354132人目の素数さん:2001/04/13(金) 23:45
>>348

こんなのはどうよ。

 0=(-1)×0
  =(-1)×{1+(-1)}
  =(-1)×1 + (-1)×(-1)
  =-1 + (-1)×(-1)
355132人目の素数さん:2001/04/13(金) 23:55
>>354
どうして(-1)×1 =-1になるの?
356132人目の素数さん:2001/04/13(金) 23:55
>>353
だから正確な話ではない。
ちゃんと証明するより納得する理由が欲しいだけなら
いろいろとごまかす方法があるけど、その一つという意味です。
357131人目の素数さん:2001/04/13(金) 23:58
1は単位元なの
358お好きなとこへどーぞ:2001/04/14(土) 00:01
359理系のおちこぼれ:2001/04/14(土) 00:18
教えてください

△ABCの内心Iから各辺BC、CA、ABに下ろした垂線の足を
それぞれP,Q、Rとおく。
BC=a、CA=b、AB=cとするとき、
a(ベクトルIP)+b(ベクトルIQ)+c(ベクトルIR)=0
となることを示せ。
360132人目の素数さん:2001/04/14(土) 00:47
>>355
0=0*1=(1-1)*1=1+(-1)*1
361132人目の素数さん:2001/04/14(土) 01:01
>>315=高校生
よっしータンがページ作ってくれてるぞ。あっちでお礼してくるべし。
http://www.geocities.co.jp/Playtown-Dice/5061/sansu/kokosei1.htm
362132人目の素数さん:2001/04/14(土) 01:02
>>360
どうして 1×1 = 1 になるの?
363132人目の素数さん:2001/04/14(土) 01:34
1は単位元なの
364tr:2001/04/14(土) 02:21
>>359 さん
内接円の半径 r とすると
  △BCI : △CAI : △ABI
  = (1/2)ar : (1/2)br : (1/2)cr = a : b : c
であり, 直線 AI と BC の交点 D として
  BD : DC = △ABI : △CAI = c : b
  AI : ID = △ABI : △BDI = (b+c) : a
から
  AI = {(bAB + cAC)/(b+c)}*{(b+c)/(a+b+c)}
  ⇔ (a+b+c)IA + bAB + cAC = 0 …(#)
が成り立つ。

次に線分の長さを
  RB = BP = l, PC = CQ = m, QA = AR =n
  [注) l+m = a, m+n = b, n+l = c です]
と表すことにすると
  aIP = a{IA + (mAB + lAC)/(l+m)}
  bIQ = b{IA + n/(m+n)AC}
  cIR = c{IA + n/(l+n)AB}
と変形できるので
  aIP + bIQ + cIR = (a+b+c)IA + (m+n)AB + (l+n)AC
            = (a+b+c)IA + bAB + cAC = 0 (∵(#))
365理系のおちこぼれ:2001/04/14(土) 10:53
>>364さん
なるほど。ありがとうございます。
366 :2001/04/14(土) 12:22
367132人目の素数さん:2001/04/14(土) 12:37
age
368132人目の素数さん:2001/04/14(土) 13:10
>>359
こんな解法もあるよん。
(以下、大文字2つ並んでるのはベクトルと思われよ)

まず、AB+BC+CA=0が当然成り立つ。ここで
 IB'=AB、IC'=BC、IA'=CA
となるように点B'、C'、A'をとれば
 IB'+IC'+IA'=0 ・・・(★)
が成り立つ(Iを始点にそろえただけ)。
いま、Iを中心とする90°回転を表わす変換をfとして、★より
 f(IB')+f(iC')+f(IA')=0
(∵fは線型変換)が得られる。そして、
 f(IB')=(c/r)IR、f(IC')=(a/r)IP、f(IA')=(b/r)IQ
なので、題意は示される。
369132人目の素数さん:2001/04/14(土) 18:27
実数の区間[a,b]を任意に取ります。
この時、有理数の稠密性よりこの区間内に有理数があることは
証明できるのですが、無理数があることを証明できません。
基本的な問題で申し訳ないのですが、誰か証明を教えて下さい。
370132人目の素数さん:2001/04/14(土) 19:04
>>369
1<a<bの場合。
1<a<c<d<bとなる任意の有理数c,dについて、
root(c*d)が有理数でないとき、
それは無理数であり、
c<root(c*d)<dである。
371132人目の素数さん:2001/04/14(土) 19:04
ここでいう「実数」「有理数」「無理数」の定義は?
372132人目の素数さん:2001/04/14(土) 19:10
[a,b]にひとつも無理数がないとすれば[a,b]は可算集合となり矛盾
373山苺:2001/04/14(土) 19:18
>>369
もちろん a<b でないとだめっす。
解法I
[a,b]にある有理数は可算無限個しかないが[a,b]は非可算無限集合
なので有理数しかないっていうことはない。
解法II
(a,b)のなかの有理数 p をとってくる。e を p-a, b-p より小さい
正の数とする。すると a<p-e<p+e<b。
有理数の稠密性より √2 - e < q < √2 + e なる
有理数 q をとる。すると -e<q+√2<e。
よって p-e<p+q+√2<p+e。
よって a<p+q+√2<b。
よって無理数 p+q+√2 が (a,b) のなかにある。
374山苺:2001/04/14(土) 19:22
>>373
訂正
誤:(9、10、11行目)q+√2
正:-q+√2
375132人目の素数さん:2001/04/14(土) 19:50
1+1=2って証明できるんかな?
376132人目の素数さん:2001/04/14(土) 20:44
↑ごめんちゃい。他に板ありました。
377132人目の素数さん:2001/04/14(土) 21:21
369です。答えて下さったみなさんありがとうございました。
わかりました。

あと、ここでは実数とは連続性の公理とか可換体の公理とか
順序の公理で定義されるもので、有理数は既約分数で表される
もの、無理数は実数であって有理数でないものという風に考えて
いました。
378132人目の素数さん:2001/04/15(日) 02:14
「代数的数の最小多項式は既約である」
みたいな記述を目にしたのですが、ここでいう「既約」
ってのは実数範囲でのことでしょうか?

「実超越数は無理数である」
・・・すいません、実超越数ってナニモノですか?
379132人目の素数さん:2001/04/15(日) 03:04
>ここでいう「既約」ってのは実数範囲でのことでしょうか?

範囲は有理数。

>実超越数ってナニモノですか?

有理数を係数とする多項式の解になるものを「代数的数」という。
たとえば、
1,2,-1/3,√2,1+2i(i は虚数単位)
なんかは代数的数。それぞれ、
x-1=0,x-2=0,x+1/3=0,x^2-2=0,x^2-2x+5=0
の解だから。

で、代数的数以外の数を「超越数」という(当然、無理数になる)。
そのうち実数であるものが実超越数。
円周率πとか、自然対数の底 e がその例。
380凄いどうでもいいけど:2001/04/15(日) 09:20
可換環って、「かかんかん」って読むんですか?
381132人目の素数さん:2001/04/15(日) 10:16
>>380
違います。「かかんわ」です。
それくらい分からないでどうする?
382132人目の素数さん:2001/04/15(日) 10:45
かーねるかーねるかかんかん
383132人目の素数さん:2001/04/15(日) 11:18
   ○○○
  ×○○○
 ――――――
   ○○○
  ○○○
 ○○○
 ――――――
 ○○○○○


虫食い算です。
○には0〜9の数字がそれぞれ2回ずつ使われています。
どう考えればいいでしょうか?
384383:2001/04/15(日) 11:21
ちょっとずれちゃった。すみません。
でも、問題の意味は伝わります・・・よね?
(3桁の数)×(3桁の数)です。
385Ms.名無しさん:2001/04/15(日) 12:03
Give asymptotic bound for T(n)
1).T(n) = 3T(n/3 +5) +n/2
2).T(n) = T(n-1) +1/n

please.
386132人目の素数さん:2001/04/15(日) 12:19
次の級数の収束・発散を判定せよ。
Σ[n=2,∞] 1/{n^2-2√n}

方針を教えてください。お願いします。
387Ms.名無しさん:2001/04/15(日) 12:39
1 + 1/2 + 1/3 + ... + 1/n + ...
のΣはどうやって表せますか?
388132人目の素数さん:2001/04/15(日) 12:53
>>386
たぶんだけど...
これ次の定理

  定理 正数列 a_n > 0 においてもし a_n < c となる定数
     がとれるなら 納n=1,∞]a_n は収束する。

を使わないとめちゃむずい?これ出典はどこ?まさか大学受験の
問題集?高校数学の範囲でとけるのかな?

389132人目の素数さん:2001/04/15(日) 12:55
>>388
まちがえた。
誤:a_n < c
正:納k=1,∞]a_k < c
390132人目の素数さん:2001/04/15(日) 12:56
>  定理 正数列 a_n > 0 においてもし a_n < c となる定数
>     がとれるなら 納n=1,∞]a_n は収束する。

ハァ?
391132人目の素数さん:2001/04/15(日) 13:22
>>386 n^2-2n^{1/2}>n^2/2
392収束する>386:2001/04/15(日) 13:39
>>323のまねをしてみる
1/(n^2-2√n) < 1/(n^2-2n) = {1/(n-2)-1/n}/2 ( n >= 3 )
393386:2001/04/15(日) 13:40
>>388-391
レスありがとうございます。
>>391
これで逆数をとって、納n=1,∞]2/n^2が収束するから、
与式も収束するってことですね。
その不等式がどうやって出てくるのかが知りたいです。
数学的センスの問題なのでしょうか?
394132人目の素数さん:2001/04/15(日) 13:45
>>387
納n=1,∞] 1/n
395132人目の素数さん:2001/04/15(日) 13:55
>>391
> n^2-2n^{1/2}>n^2/2

ちょい修正
n^2-2n^{1/2}>n^2/4 (n>=2)
396132人目の素数さん:2001/04/15(日) 14:58
東京出版のホームページにある算数王神社
http://www.tokyo-shuppan.co.jp/susanowo/index.html
にある問題の2

問題2 2∫[0,π/3]1/√(1-k sin^2x)dx
=∫[0,π/2]1/√(1-k sin^2x)dx
をみたす定数 k を求めよ。

さっぱりわからんぬ。ヒントきぼんぬ。参考資料もきぼんぬ。
397132人目の素数さん:2001/04/15(日) 17:48
>>396
第1種完全楕円積分だっけ?
楕円積分とかその辺を調べてみては・・・
398132人目の素数さん:2001/04/15(日) 18:50
>>397
んぬっ。レスありがとんぬ。楕円積分ネットでしらべてみたんぬ。
すると、

∫[0,θ]1/√(1-k sin^2x)dx、∫[0,θ](1-k sin^2x)dx

の形の積分を第一種、第二種不完全楕円積分というらしんぬ。
とくに θ=π/2 の場合を完全というらしいんぬ。第二種不完全
楕円積分は楕円を (a cos t,b sin t) とパラメータ表示したとき
の 0≦t≦θ の部分の周長をあたえるらしいんぬ。そこまでは
わかったんぬ。でも第一種のほうはさっぱりネット上ではわからんぬ。
あるホームページで値を計算してくれるとこがあったんぬ。
みつけたときちょっとうれしかったんぬ。でもそのあと悲しかったんぬ。
やっぱり本かなんかしらべんとあかんみたいんぬ。
なんかどっかに参考資料なかりしかんぬ。
Help meんぬ。
399Ans: 8/9:2001/04/15(日) 19:38
>>396
楕円関数 sn の加法定理の問題。
参考資料は楕円関数の教科書。
正解者の方々「132人目の素数さん」って誰だよ?
俺じゃねーぞ(w
400396:2001/04/15(日) 19:43
>>397 >>399
ありがとございました。396 です。398 ではちょっとわるのり
しすぎて我ながらうざい。すいません。
sn の加法定理しらべてみます。それでもだめならもっかい聞きます。
たぶんもっかい聞くかも... いや絶対聞くな。でもしばし自分で
調べてみます。ありがとございました。


401うーん:2001/04/15(日) 20:04
すみません。どうしてもわからないのですが・・
問題は点列の物で・・

n次元空間R^nの集合Q(A;r)は閉集合、
C(A;r)は開集合という事を示せ。

です。
Q(A;r)={(X1,X2,... ,Xn)∈R^n| |Xi-Ai|≦r}
C(A;r)={(X1,X2,... ,Xn)∈R^n| |Xi-Ai|<r}
と定義してます。

宜しくお願いします。
402132人目の素数さん:2001/04/15(日) 23:57
>>379
遅ればせながら、どうもです。
403zk:2001/04/16(月) 02:59
y^2=x^6+1は複素空間上で穴が2つ開いた曲面になると聞き,
あれこれ試しましたが、さっぱりです。
どういう空間に射影すれば良いのでしょうか?
また、一般的な2変数多項式がどういう曲面になるか分かる方法がございましたら、
教えて頂きたくお願いします。
ただし、できれば、高校生程度の知識で。
404132人目の素数さん:2001/04/16(月) 08:46
メビウス関数の反転公式
f(n)=Σg(d)->g(n)=Σμ(n/d)g(d)
の応用例を教えてください。
405383:2001/04/16(月) 20:11
age
406132人目の素数さん:2001/04/16(月) 22:22
lim[An]=+∞→lim[1/An]の証明と
An>0,lim[An]=0→lim[1/An]=+∞の証明を教えてください。
大学に内部推薦で入ったため授業についてけません。
407132人目の素数さん:2001/04/16(月) 22:24
lim[An]=+∞→lim[1/An]=0でしたすいません
408132人目の素数さん:2001/04/16(月) 23:08
紀宮清子内親王  1969/04/18 31歳
三笠宮彬子女王  1981/12/20 19歳
三笠宮瑶子女王  1983/10/25 17歳
高円宮承子女王  1986/03/08 15歳
高円宮典子女王  1988/07/22 12歳
高円宮絢子女王  1990/09/15 10歳
秋篠宮眞子内親王 1991/10/23  9歳
秋篠宮佳子内親王 1994/12/29  6歳

36年間にコインの表が続けて8回でました。
9回目にコインの表が出る確率はいくつでしょう?
409でじこ@数学板:2001/04/16(月) 23:48
>>406
「lim[An]=+∞→lim[1/An]=0 を証明せよ」

これは、言い換えると

 「任意の実数 K に対して、ある整数 m が存在して

   n > m ⇒ An > K

  を満たす」

 との仮定から

 「任意の正数 ε に対して、ある整数 m が存在して

   n > m ⇒ |1/An| < ε

  を満たす」

 を導け

との問にょ。

まず、任意の正数 ε が与えられたとして、仮定より、

 1/ε に対して、ある整数 m が存在して

  n > m ⇒ An > 1/ε ( ⇒ 1/An < ε . ∵ 1/ε >0 )

 を満たす

が言えるので、このことから

 任意の正数 ε に対して、ある整数 m が存在して

  n > m ⇒ |1/An| < ε

 を満たす

が言えるにょ。
410132人目の素数さん:2001/04/17(火) 01:28
ある本曰く、
「Q(α,β,…,γ)=Q(ω)となる代数的数ωが存在する」
とのこと。どーしてですか?
あ、ちなみにQ(α,β,…,γ)は代数体のことです。
411132人目の素数さん:2001/04/17(火) 01:28
ある本曰く、
「Q(α,β,…,γ)=Q(ω)となる代数的数ωが存在する」
とのこと。どーしてですか?
あ、ちなみにQ(α,β,…,γ)は代数体のことです。
412410=411:2001/04/17(火) 01:30
あれ?失敗?すいませんです。
413132人目の素数さん:2001/04/17(火) 01:36
数体の有限拡大はつねに単純拡大
414いきなり6次で考えんでも…:2001/04/17(火) 03:07
>>403
>y^2=x^6+1は複素空間上で穴が2つ開いた曲面になると聞き,
>あれこれ試しましたが、さっぱりです。

やりかたは
y^2=(x-a)(x-b)(x-c)(x-d)
(a,b,c,d は定数、どのふたつも相異なる)
の場合と同じです。
415132人目の素数さん:2001/04/17(火) 05:03
閉集合、開集合の定義は?>>401
416名無しさん@祝ご懐妊:2001/04/17(火) 12:51
下記のサンプル1及び2において、Aに対するB及びCの関係を導き出してください。
文字は16進数(多分)。

・Sample1
A「7313a6e7bf7f111b604696f20c83c224」
B「99b94581ec3091d98bb675e3c86f82e0」
C「09854958b4eb69d8fd8cc8571fb9231801d0a8e304b6fc12」

・Sample2
A「5a2e304a327717223bd6cc0c296c0629」
B「8fc3ac89373cbcca302a794d2f069a54」
C「6bbd9b9cf4577c175900aeb781b9908001d0a8e369f75a89」
417132人目の素数さん:2001/04/17(火) 13:00
「空事象でない2つの事象AとBが独立、このときAとBは排反
でないことを定義に基づいて示せ。」これは数学的に解ける問題
なのでしょうか?
418132人目の素数さん:2001/04/17(火) 14:27
>>417
事象 A,B が独立:⇔P(A∩B)=P(A)P(B)
事象 A,B が排反:⇔P(A∩B)=0
やったっけ?
ex.
Joker なしのトランプ52枚をひいて
A:=“奇数を引く”B:=“偶数を引く”C:=“ハートを引く”
なら
P(A)=28/52,P(B)=24/52,P(A∩B)=0 なので A,B は背反やけど独立でない。
P(A)=28/52,P(C)=13/52,P(A∩C)=7/52 なので A,C は独立やけど背反でない。
かな?受験数学みたい。なぜそれぞれのカードを引く確率が 1/52 なのか...
そっちのほうが数学では手がでんかも...
419132人目の素数さん:2001/04/17(火) 15:21
数2の三角関数の初歩的なことなんですが
cosθ-sinθ/cosθ+sinθ=1-tanθ/1+tanθ
の変形がわからないです。
420132人目の素数さん:2001/04/17(火) 15:44
>>419
・分母&分子をcosθで割る
・(sinθ/cosθ)をtanθに直す
421132人目の素数さん:2001/04/17(火) 17:36
1+1=2 の証明は今井のHPにあるぞ。
422132人目の素数さん:2001/04/17(火) 21:51
Q(A;r)={(X1,X2,... ,Xn)∈R^n| |Xi-Ai|≦r}
C(A;r)={(X1,X2,... ,Xn)∈R^n| |Xi-Ai|<r}

C(A;r)の点xをとり、max|Xi-Ai|=Rxとおけば、開球|X-A|<r - RxはC(A;r)に含まれる。xは任意なのでC(A;r)は開集合.
Q(A;r)はC(A;r)の閉包だから閉集合。
423132人目の素数さん:2001/04/17(火) 23:02
>>383

179×224
424132人目の素数さん:2001/04/18(水) 00:56
>>421
今井のHPって信用出来るのか?
425うーん:2001/04/18(水) 01:06
>422
御返事遅くなりすみません。
ありがとうございます。
少し自分の頭で 考え直して見ます。
それでわからなかったらまた質問します。

それでは失礼いたしました。
426132人目の素数さん:2001/04/18(水) 01:25
>>424
ネタ?
427132人目の素数さん:2001/04/18(水) 01:54
最小多項式って、なぜ一意に定まるのでしょうか?
証明していただけると嬉しいです。
428132人目の素数さん:2001/04/18(水) 02:06
体K、部分体M、M係数1変数多項式環M[X]
a∈KのM上の最小多項式は、イデアル{f(x)∈M[X] |f(a)=0}
の生成元のうち、次数が最低で、最高次の係数が1であるもの。
別の生成元をとれば、それは最小多項式で割り切れる。
最小多項式と同じ次数の多項式が生成元ならば、それは最小多項式の定数倍。
よって最小多項式は一意に定まる。
429132人目の素数さん:2001/04/18(水) 04:20
>>427
428 氏の説明がわかる人はもともと質問しないと思うので、初心者用の説明。

αの最小多項式とは、f(α)=0 となるもののうち、「次数が最低で最高次の係数が 1 」
であるもの。
αの最小多項式が f(x), g(x) と二つあったとする[f(α)=g(α)=0,f(x)≠g(x)]。

このとき、f(x) と g(x) の次数は等しい。そうでなければ、次数の大きい方が、
「次数が最低で」の条件を満たしていないので最小多項式であることに反する。

次数を n 次として、f(x)=x^n+…, g(x)=x^n+… とする。

h(x)=f(x)-g(x) とおく。x^n は打ち消しあうので、h(x) は高々 n-1 次式。
また、f(x)≠g(x) なので、h(x) は恒等的に 0 とはならない。
よって、h(x)=ax^m+bx^(m-1)+… (a≠0,m<n)とおくことができ、
h(α)=f(α)-g(α)=0 である。
そこで、k(x)=h(x)/a を考えれば、これは、「k(α)=0 で、次数が n より小さく、
最高次の係数が 1 」である。
次数が n より小さいものができてしまったので、f(x), g(x) は最小多項式でなかった
ことになり矛盾。
430132人目の素数さん:2001/04/18(水) 04:49
>>403
穴の数のことを種数といいます。
f(x) を n 次式で、f(x)=0 が重解を持たないとすると、y^2=f(x) の種数は、
n=1, 2 → 種数0
n=3, 4 → 種数1
n=5, 6 → 種数2
n=7, 8 → 種数3
という具合になります。
少なくとも y=±√x を2枚の複素平面を張り合わせて一価関数にする話(リーマン面)
くらいは知らないと・・・。高校程度の知識で説明するのは難しそう。
431427:2001/04/18(水) 12:50
>>428
>>429
ありがとうございました。
432zxc:2001/04/18(水) 12:59
a,b,cを0でない実数として、空間内に3点A(a、0,0)B(0,b、0)
C(0,0、c)をとる。
(1) 空間内の点PがベクトルAP・(ベクトルBP+ベクトル2CP)=0
を満たしながら動くとき、点Pはある定点Qから一定の距離にあることを示せ。

(2) (1)における定点Qは3点A,B,Cを通る平面上にあることを示せ。

(3) (1)における点Pについて、四面体ABCPの体積の最大値を求めよ。

*なるべく高1の範囲で解いてください。よろしくお願いします。
433132人目の素数さん:2001/04/18(水) 16:23
以下、ベクトルPを「Pなどと表します。
「P=(x,y,z)と置く。
「AP=「P−「A=(x-a,y,z)
「BP=「P−「B=(x,y-b,z)
2「CP=2(「P−「C)=(2x,2y,2z-2c)
∴与式の左辺=(x-a,y,z)・(3x,3y-b,3z-2c)
=3(x^2-ax) +3(y^2- (by/3)) +3(z^2- (2cz/3))=0
3で割って平方完成すると、
 (x- (a/2))^2 +(y- (b/6))^2 +(z- (c/3)^2 =(a/2)^2 +(b/6)^2 +(c/3)^2
この式を満たすx,y,zは、点(a/2,b/6,c/3)を頂点とする球上にある。
よってPは定点Q=(a/2,b/6,c/3)から一定の距離にある。

また、Qが平面ABC上にある事は、
『ある点xが平面def上にある条件
「x=r「d +q「e +s「f(r+q+s=1)』
より明らか。

また、「QPがABCに垂直なとき体積は最大で、このとき
最大値=球Pの半径*三角形ABCの面積である。

・・である。けど。出せない。(汗
434433:2001/04/18(水) 16:27
ちょっと待て。
最大値=球Pの半径*三角形ABCの面積/3
なにやってんだか。
435わからん:2001/04/18(水) 18:38
くだらんスレにもお願いしてるんですが・・・
3 2
1

6 2 5
4 3
1
のように三角形に自然数を並べ、差が下の数になるようにする。
5段(n=15)まではいけるけど、そのあとはどうでしょうか?
436y^2=x^3+ax+b:2001/04/18(水) 21:08
(1)条件より
OA・OB=OA・OC=OB・OC=0
よって(1)の式は
AP・(BP+2CP)=0
⇔(OP-OA)・(OP-OB+2OP-2OC)=0
⇔3|OP|^2-OP・(OB+2OC+3OA)=0
⇔| OP-{(1/3)OB+(2/3)OC+OA}/2 |^2=|{(1/3)OB+(2/3)OC+OA|^2 / 4
より、定点{(1/3)OB+(2/3)OC+OA}/2
から等距離|{(1/3)OB+(2/3)OC+OA| / 2
にある点である

(2)(1)より明らか
(ABの内分点とその点とAの内分点)

(3)Qが球の中心で、平面ABCが、そのQを通る平面である事から
(1)で出た距離を使って簡単に計算できる
437xyc:2001/04/18(水) 21:26
(2)をもうちょっと詳しくお願いします。
438xyc:2001/04/18(水) 21:27
(2)をもうちょっと詳しくお願いします。
439xyc:2001/04/18(水) 21:31
Oを原点とする空間内に3点A,B,Cがあり、4点O,A,B,Cは
同一平面上にないものとする。
ベクトルOP=ベクトル2OA+ベクトル3OB+ベクトル4OC と点Pをおくとき
(1) 四面体PABCの体積と四面体OABCの体積比は?

ほかのところに書いたんですが返答がなかったので、これもよろしくお願いします。

440無限等比級数:2001/04/18(水) 21:36
数三の範囲の、無限等比級数と図形の問題です。
学校でやっていないのでさっぱりわかりません、教えてください。

問題

∠Aが頂角である三角形ABCで、CA=1、AB=2とする。
頂点Aから辺BCに垂線を下ろし、その足をA1とする。
次にA1から辺ABに垂線を下ろして、その足をA2とする。
このようにして、図のように垂線A2A3、A3A4、A4A5、・・・
を作っていく。
このとき、L=AA1+A1A2+A2A3+・・・・・・を求めよ。
(図形では、Aが左下で直角です)
最終的にこれは、初項2/√5、公比2/√5の収束する
無限等比級数の和を計算する、ってなるんですが、過程がさっぱりわかりません
よろしくおねがいします。
441132人目の素数さん:2001/04/18(水) 21:41
解けるけど書くのめんどい。方針はOPと平面
ABCの交点をQとでもしてOQ対QPが答えに
なる。OQをベクトルOA,OB、OCで現そう。
442132人目の素数さん:2001/04/18(水) 21:44
>>441>>439の返事ね。
>>441
あんたねえ、参考書みたほうが早いよ?
問題自体は簡単だけど。似た問題チャートにも
あるでしょう?
443132人目の素数さん:2001/04/18(水) 21:45
>>441>>439の返事ね。
>>442
あんたねえ、参考書みたほうが早いよ?
問題自体は簡単だけど。似た問題チャートにも
あるでしょう?
444132人目の素数さん:2001/04/18(水) 21:52
>>440
まずAA1、A1A2、A2A3、……を求めれ
中学でやったろ?
445xyc:2001/04/18(水) 21:56
そこをなんとかおねがいします。
446132人目の素数さん:2001/04/18(水) 22:17
>>439
OQ=2/9OA+3/9OB+4/9OCになる。
よってOQ対QP=1対8だから1対8
OQ=pOA+qOB+rOC,p+q+r=1
p:q:r=2:3:4でだすわかる?
447わからん:2001/04/18(水) 22:21
>440
点A,点B,点Cでできる三角形をΔ(A,B,C)と書くと
Δ(C,A,B)∽Δ(c、A,A1)よりAA1=2/√5
同様に
Δ(C,A,B)∽Δ(A,A2、A1)より
A1A2=AA1*2/√5=(2/√5)^2
初項2/√5、公比2/√5の無限等比級数ですな
俺もけっこういけるやん、30年前に受験したにしては
448xyc:2001/04/18(水) 22:31
>446
もっと詳しくお願いです。
449132人目の素数さん:2001/04/18(水) 23:02
>>448
>>446
の三行目は平面ABC上にQがある条件
4行目はQがOP上にあるという条件
OQ=1/9OPだからOQ対OP=1:9
よってOQ:QP=1:8
PABCとOABCは底面ABCが共通だから高さの比が
体積比になる。図をかくと高さの比はOQ:QPに
なることがわかる。自分で図をかけ!
450y^2=x^3+ax+b:2001/04/18(水) 23:21
>>437 >>438
{(1/3)OB+(2/3)OC+OA}/2

BCを2:1に内分した点をDと呼ぶとき
ADを1:1に内分した点
ちゃんと紙に綺麗に分数を書けば分かるはずです
451132人目の素数さん:2001/04/18(水) 23:54
10進法表示のn桁の正の整数で、
隣り合う桁の数字が互いに相異なるような数の個数をA(n)とする。

(1)A(n)を求めよ
(2)上の数のうちで、1の位の数が0である数の個数を
B(n)とするときlim(n→∞){B(n)/A(n)}を求めよ

東工大の問題ですがよろしくおねがいします
452xyc:2001/04/19(木) 00:03
さっきの(3)も計算式を書いてもらうとありがたいんですが・・・
どうかよろしくお願いします。
453132人目の素数さん:2001/04/19(木) 00:08
>>452
計算くらい自分でやれゴルァ
454留年太郎:2001/04/19(木) 00:22
すいません。
私も一つお願いします

京都府立の問題です

「一回の試行で事象Aが起こる確率はpであって
Aがおこれば2点、起こらなければ一点が与えられる。
この試行を繰り返し行うとき、得点の合計が途中で丁度n点となる確率を
P(n)とする」
(1)
P(n)={(1-p)P(n-1)}+{p*P(n-2)}を示せ
(2)
P(n)をnの式であらわせ
(3)
得点の合計が途中でn点とならないで2n点となる確率を求めよ
455132人目の素数さん:2001/04/19(木) 01:11
代数的数全体は、なぜ体になるのですか?
456tr:2001/04/19(木) 02:04
>>451 さん
(1) { A(1) = 9          | 1,2,3,4,…,8,9
  { A(n+1) = A(n)*9 (n≧1) |------------
なので, これを解いて     | 10,12,13,…,19
  A(n) = 9^n (n≧1)      | 20,21,23,…,29
                  | …     …
(2) B(n) = A(n)/9 (n≧2)    | 90,91,92,…,98
が成り立つから,
  B(n) = 9^(n-1) (n≧2)
  ∴ lim[n->∞] B(n)/A(n) = 1/9

# 東工大の問題がこんな簡単なハズがない..
# めっちゃ勘違いしてるかもしれません(汗)
457tr:2001/04/19(木) 02:06
>>454 さん
(1) ちょうど n点になるのは,
  i) (n-1)点の状態から A が起こらない
  ii) (n-2)点の状態から A が起こる
のどちらかなので
  P(n) = P(n-1)*(1-p) + P(n-2)*p (n≧3)

(2) { P(1) = 1-p, P(2) = p + (1-p)^2……(#1)
  { P(n) = (1-p)*P(n-1) + p*P(n-2) …(#2)
(#2) を変形した
  { P(n) - P(n-1) = -p{P(n-1) - P(n-2)}
  { P(n) + p*P(n-1) = P(n-1) + p*P(n-2)
それぞれから n≧3 に対し, 次式を得る。
  { P(n+1) - P(n) = {P(2)-P(1)}*(-p)^(n-1)
  { P(n+1) + p*P(n) = P(2) + p*P(1)
辺々ひいたのち (#1) を代入, 整理して
  P(n) = -{(p^2)*p^(n-1) - 1}/(p+1) (∵ p+1≠0)
これは n = 1, 2 にも適するので n≧1 で成り立つ。

(3) P(2n) - {P(n)}^2 を計算すればよく
  {(p^3)*(-p)^(2n-2) + 2p^2(-p)^(n-1) + p}/(p+1)^2 (n≧1)
458tr@自己レス:2001/04/19(木) 02:41
やっぱ >>456 は間違ってます。(爆)
459tr@訂正:2001/04/19(木) 02:59
と言うことで >>456 の (2) を訂正します。

(2) B(n+1) = A(n) - B(n) (n≧1)
が成り立つことと, B(1) = 0 から
  B(n+1)/9^(n+1) - 1/10 = (-1/9){B(n)/9^n - 1/10}
  ⇒ B(n)*/9^n = {B(1)/9 - 1/10}*(-1/9)^(n-1) + 1/10
  ⇒ B(n) = (1/10){9^n - 9*(-1)^(n-1)} (n≧1)
したがって
  lim[n->∞] B(n)/A(n) = 1/10

# 今度はあってるかな?(汗)
460132人目の素数さん:2001/04/19(木) 06:03
>>455
a を代数的数として、たとえば、a^4+2a^3+3a^2+4a+5=0 であるとします。
それなら、

(-a)^4-2(-a)^3+3(-a)^2-4(-a)+5=0
1+2(1/a)+3(1/a)^2+4(1/a)^3+5(1/a)^4=0

も成立しますので、-a や 1/a も代数的数であることはすぐわかります。
しかし、a, b が代数的数であるとき、a+b, ab も代数的数であることを示すのは、
少し面倒。代数学の教科書を見てください。
461132人目の素数さん:2001/04/19(木) 06:26
a,bがQ上代数的なら、Q(a,b)/Qは代数拡大。
462名無し:2001/04/19(木) 15:58
偏微分の記号∂の読み方教えてください。
463132人目の素数さん:2001/04/19(木) 16:11
ディー。ラウウンドと言う人もいるのかな?
464ご冗談でしょう?名無しさん:2001/04/19(木) 16:18
「でる」って私は読んでいるが
465132人目の素数さん:2001/04/19(木) 18:17
Tex流に「ぱーしゃる」って読むひとはいないか?
466132人目の素数さん:2001/04/19(木) 22:19
対偶と背理法は同じものなのですか?
467留年太郎:2001/04/19(木) 22:20
trさんご解答ありがとうございます。
自分は確率漸化式が弱いので演習を繰り返しているのですが
もう一問、同じ分野で質問させてください。

2つの袋A,Bがあってそのどちらにも赤玉1つ、白玉1つの計2個ずつの玉が入っている
両方の袋からでたらめに玉を一つずつ取り出していれかえる。
この操作をn回引き続いていったとき
始めのように各袋に赤玉と白玉が1つずつ入ってる確率をP(n)とする。

(1)n≧1としてP(n)とP(n+1)の関係式を求めよ

(2)P(n)を求めよ
468132人目の素数さん:2001/04/19(木) 23:34
裳華房 集合と位相 内田伏一著 の10P問2.5(5)
A=B−(B−A)がA⊂Bと同値であることを示せ。
どのようにしてやればいいのでしょうか? 教えてください。

右辺={x|x∈Bかつx(∈≠)(B−A)}
  ={x|x∈Bかつx(∈≠){x|x∈Bかつx(∈≠)A}
  =?
469132人目の素数さん:2001/04/20(金) 00:18
>>468
A=B-(B-A)⇔A⊆B

(→)
x∈A
⇒x∈B-(B-A)
⇒x∈BかつNOT(x∈B-A)
⇒x∈BかつNOT(x∈BかつNOT(x∈A))
⇒x∈Bかつ(NOT(x∈B)またはx∈A)
⇒(x∈BかつNOTx∈B)または(x∈Bかつx∈A)
⇒x∈B

(←)
x∈A
⇒x∈B
⇒x∈Bかつx∈A
・・・
⇒x∈B-(B-A)
また
x∈B-(B-A)
⇒x∈Bかつx∈A
⇒x∈A
より
x=B-(B-A)■
470tr > 留年太郎さん:2001/04/20(金) 01:24
# 状況ごとに記号をさだめるとわかりやすいかも?

>>467 の問題
n 回いれかえを終わらせた状況について
  A が 赤白 [, B が 赤白] を P(n)
  A が 赤赤 [, B が 白白] を Q(n)
  A が 白白 [, B が 赤赤] を R(n)
とおくと n≧1 に対し, 次の 3式が成り立つ。
  { P(n) + Q(n) + R(n) = 1
  { Q(n) = R(n)
  { P(n+1) = P(n)*(1/2) + Q(n)*1 + R(n)*1
Q(n), R(n) を消去して
  P(n+1) = (-1/2)P(n) + 1 (n≧1)
これを解いて
  P(n) = {P(1) - 2/3}*(-1/2)^(n-1) + 2/3
     = (-1/6)*(-1/2)^(n-1) + 2/3 (n≧1)
471132人目の素数さん:2001/04/20(金) 10:47
2を越える偶数は2つの素数の和で書ける。誰か証明してちょ。
472132人目の素数さん:2001/04/20(金) 11:45
>>471
とんでもを誘導しよ〜っちゅ〜はらやな。ちょっと意地悪いで。
473132人目の素数さん:2001/04/20(金) 11:58
二つの事象AとBが独立であるというのと二つの事象XとYが独立
であるということの定義はどこが違うのでしょうか?それぞれ
の具体例を出していただけるとありがたいのですが。
474132人目の素数さん:2001/04/20(金) 12:16
>>469さん、レスありがとうございました。

もう一つだけ、
(A∪B)∩(A∪C)∩(A∪D)∩(B∪C)∩(B∪D)∩(C∪D)
=(A∩B∩C)∩(A∩B∩D)∩(A∩C∩D)∩(B∩C∩D)
がどうしても示せません。
長いやつで申し訳ありませんが、教えていただけないでしょうか。
475132人目の素数さん:2001/04/20(金) 12:43
初歩的な質問かもしれませんが、わかる方教えてください。

10進数の25.375を2進数に変換すると
25の部分は 11001 に
0.375の部分は .011 となると書いてるんですけど

逆に2進法の11001.011を10進数に変換する場合
どういう計算すればいいんでしょうか?
11001の部分は、右から順に2・4・8・16・32と計算していけばと
わかるんですが、小数の部分の計算がわからないのです。
2進法の.011をどう計算すれば10進法の0.375になるのですか?
476132人目の素数さん:2001/04/20(金) 12:50
>>475
>2進法の.011をどう計算すれば10進法の0.375になるのですか?
0*(1/2)+1*(1/4)+1*(1/8)=0.375
477>:2001/04/20(金) 12:52
10進法とおなじこと 0.123 は
1* 1/10 + 2* (1/10^2) + 3 * (1/10^3) のこと。
同じように2進で 0.011とは
  0* (1/2) + 1 * (1/2^2) +1 * (1/2^3)のこと。
 ( つまり 10進表記で 1/4+1/8 )
  
478132人目の素数さん:2001/04/20(金) 14:25
>>476.477
やっと理解できました。ありがとうございます。
479132人目の素数さん:2001/04/20(金) 15:08
有限個の閉集合の和集合は閉集合である。これの証明はどうやるんですか?
一方、無限個の閉集合の和集合は閉集合であるとは限らない。反例はあるんですか?
480132人目の素数さん:2001/04/20(金) 15:14
これがわかりません。
Fを閉集合、Gを開集合とする。F\G、G\Fについて何が云えるか。
誰か教えてください。
お願いします。
481132人目の素数さん:2001/04/20(金) 15:19
>>479
>>480

 どのレベルで質問してるのかな?一般位相の話?それとも距離空間とかユークリ
ッド空間の話?反例なら、閉区間[1/n,1-1/n]を自然数nについて和集合をとれば、
開区間(0,1)になって、これは閉集合じゃないよな。
482479:2001/04/20(金) 19:35
>481さんへ
お返事ありがとうございます。
一般位相での話です。宜しくお願いします。
483132人目の素数さん:2001/04/20(金) 23:33
>>482
479さんの質問の仕方では何が前提でそれを示そうと
しているのかわかりません。

たとえば仮に開集合の公理が適用された上で閉集合の定義が
開集合の補集合として定義された、としたら開集合の
公理から自明です。
484ご冗談でしょう?名無しさん:2001/04/20(金) 23:33
複素関数の分野に関して質問です。

関数f(x)がZ=Z_oにおいて複素微分可能であるとき、f(x)はZ=Z_oにおいて連続である
事を示せ。

485tr:2001/04/21(土) 04:08
>>474 さん
  x∈(A∪B)∩(A∪C)∩(A∪D)∩(B∪C)∩(B∪D)∩(C∪D)
⇔ x∈(D∪A)∩(A∪B), x∈(A∪B)∩(B∪C),
  x∈(B∪C)∩(C∪D), x∈(C∪D)∩(D∪A)
⇔ x∈A, x∈B, x∈C, x∈D
⇔ x∈A∩B∩C∩D
⇔ x∈A∩B∩C, x∈A∩B∩D, x∈A∩C∩D, x∈B∩C∩D
⇔ x∈(A∩B∩C)∩(A∩B∩D)∩(A∩C∩D)∩(B∩C∩D)
486132人目の素数さん:2001/04/21(土) 04:42
>>484
実数の場合とまったく同様に示せるんじゃないの?
487132人目の素数さん:2001/04/21(土) 05:01
測度論の話で「選択公理を認めるとルベーグ非可測集合が作れる。」
って言ってたんですけど。誰か簡単に説明してもらえません?
488132人目の素数さん:2001/04/21(土) 06:58
>>487
実数の集合に、次のような同値関係を入れる。
x 〜 y ⇔ x-y が有理数
各同値類から、区間 [0,1] に含まれる代表元を選び出し、その代表元の集合を A とする。
A が可測と仮定し矛盾をいう。
以下、m(A) とは、集合 A のルベーグ測度を表す。

B_n=A+1/n とおく(n は自然数。「+」は、A を数直線上 1/n だけ右にずらすという意味)。
このとき次のことは明らか。
(1) m(B_n)=m(A)
(2) n≠m のとき、B_n と B_m は排反。
(3) あらゆる n について、B_n は区間 [0,2] に含まれる。
[0,2] のルベーグ測度は有限(っていうか2)なので、(1),(2),(3) より、m(A)=0 である。

有理数は可付番なので、実際に番号を打ち、r_n (n=1,2,3…)をすべての有理数とする。
C_n=A+r_n とおく。次は明らか。
(1) m(C_n)=m(A)=0
(2) n≠m のとき、C_n と C_m は排反。
(3) すべての C_n の和集合は、実数全体と一致する(つまり、∪C_n=R )。
しかし、これでは、m(R)=m(∪C_n)=Σm(C_n)=0 で、実数全体の測度がゼロになるから矛盾。
489132人目の素数さん:2001/04/21(土) 09:39
>>485さん
お答えありがとうございました。
やっと納得することができました。
感謝、感謝です。
490132人目の素数さん:2001/04/21(土) 14:03
>>484
fがZoで複素微分可能 ⇔ f(Zo+h) - f(Zo) = f'(Zo)h + o(|h|)
h→0として
f(Zo+h)→f(Zo)
491無名の新人:2001/04/21(土) 14:50
まいっています。助けてください。

1+2+3+....+n = n(n+1)/2
1^2+2^2+3^2+....+n^2 = n(n+1)(2n+1)/6
1^3+2^3+3^3+....+n^3 = {n(n+1)/2}^2

ここまではわかりました。

しかし,

1^4+2^4+3^4+....+n^4 = ?
1^5+2^5+3^5+....+n^5 = ?
1^k+2^k+3^k+....+n^k = ?

はちんぷんかんぷんです。
指数がkの時は,ベルヌーイ数を使ってといわれましたが?

よろしくお願いいたします。


492132人目の素数さん:2001/04/21(土) 15:29
(x+B)^n=x^n + nC1*x^(n-1)*B + nC2*x^(n-2)*B^2 + ...
(x+B-1)^n=x^n + nC1*x^(n-1)*(B-1) + nC2*x^(n-2)*(B-1)^2
これより
★(x+B)^n - (x+B-1)^n=nC1*x^(n-1) + nC2*x^(n-2)*{B^2-(B-1)^2}+...

ここで、B^nをBのべきと思わずに、次の条件を満たす未知数とみなす
 B^2-(B-1)^2=0、B^3-(B-1)^3=0、....B^n-(B-1)^n=0
B^nの代わりにBnと書いて、Bernoulli数という。
B1=1/2、B2=1/6、B3=0, .....
すると、★式は
(x+B)^n - (x+B-1)^n=nC1*x^(n-1)  すなわち
 n*x^(n-1)=(x+B)^n - (x+B-1)^n となる。
これに、x=1,2,...kを代入して辺々を足し合わせれば、N=n-1とおいて、
 n{1^N+2^N+...+k^N}=(k+B)^(N+1) - B^(N+1)
1^N+2^N+...+k^N ={(k+B)^(N+1) - B^(N+1)}/(N+1)
右辺を展開して、B^NをBernoulli数BNに置き換えればよい。
たとえば、
1^3+2^3+3^3+....+n^3={(n+B)^4 - B^4}/4
          ={n^4+4B1*n^3+6B2*n^2+4B3*n^3}/4
          ={n^4+2*n^3+*n^2}/4={n(n+1)/2}^2
493132人目の素数さん:2001/04/21(土) 15:43
集合と位相についてです。

1.A⊂f^(‐1)(f(A))
2.f(f^(‐1)(C))⊂C

それぞれに対して、等号が成り立たないのはどのようなときですか?
成り立つときの証明の仕方は本なあったんですが、成り立たない時の具体例がわかりません。

あと、もうひとつ

In=(‐1/n ,1/n)とするとき、∩In、はなんですか
Dn={(x,y)∈R^(2)|x^(2)+y^(2)<1/n^(2)}とするとき、∩Dn、はなんですか

私は共に0ではないかと思うんですがどうなんでしょうか?
494132人目の素数さん:2001/04/21(土) 16:07
>>493
>それぞれに対して、等号が成り立たないのはどのようなときですか?

X={a,b},f:X→X
で反例が作れる.

>私は共に0ではないかと思うんですが

0と{0}の区別はつけてくれ.
495494:2001/04/21(土) 16:09
「反例」というより「例」と言ったほうがいい?
496132人目の素数さん:2001/04/21(土) 16:12
f(x)=x^2
f((-∞,0))=(0,∞)
f^(-1)f((-∞,0))=f^(-1)((0,∞))=(-∞,0)∪(0,∞)≠(-∞,0)

fの定義域をX、C⊂f(X)とすればf(f^(-1)(C))=C
そうでないとき、x∈C\f(X) ならば f(f^(-1)(x))=φ
497無名の新人:2001/04/21(土) 17:02
>>492

ありがとうございました。
こんなにはやく答えがくるとは思わずに,
お礼が遅れて申し訳ありませんでした。

感謝・感激です。
498132人目の素数さん:2001/04/21(土) 17:05
f(z)=(1+z)(1+z^2)(1+z^4)...(1+z^2n)...
このとき,
f(z)=1/1-z
となるということですが,どうしてでしょうか?
499132人目の素数さん:2001/04/21(土) 17:12
非常に基本的な問題ですいません教えてください。

1問目
 0<m<1 をみたす,有理整数は存在しないことを示せ。
2問目
 x>0 とする。このとき,
 「xは正の約数は,1とxのみ」
 または,
 「1<(xの正の約数)≦√x をみたす適当なxの約数が存在する」
3問目
 n変数のd次多項式を,単項式の和として
 無駄のない表示をした時,
 でてくる項数≦(n+d)/n!d!

以上3問たのみます。
500132人目の素数さん:2001/04/21(土) 17:44
(n+d)/n!d! ...???
501132人目の素数さん:2001/04/21(土) 17:55
>>500
間違いでした。
(n+d)/n!d!
でなくて,
(n+d)!/n!d!
でした。
すいません。

502132人目の素数さん:2001/04/21(土) 18:21
>>498
f(z)=(1+z)(1+z^2)(1+z^4)...(1+z^(2^n))...
の間違いと思われ。
(1+z)(1+z^2)(1+z^4)...(1+z^(2^n))=Σ[k=0 to 2^(n+1) -1]z^kの証明
nに関する帰納法。n=0のときは明らか。
一般のn≧1について成り立つと仮定。
(1+z)(1+z^2)(1+z^4)...(1+z^(2^n))(1+z^(2^(n+1)))
={1+z+z^2+...+z^(2^(n+1) -1))}(1+z^(2^(n+1)))
=(1+z+z^2+...+z^(2^(n+2) -1)))
よってn+1のときも成り立つ。
503132人目の素数さん:2001/04/21(土) 18:33
>>502
ご指摘の通りです。指数の表記を間違えていました。
 それでも,正答が出せるところがすごいですね。
504132人目の素数さん:2001/04/21(土) 18:34
>>498
暗算で
(1-z)(1+z)(1+z^2)(1+z^4)...(1+z^(2^n))=(1-z^(2^(n+1)))
を導く。
|z|<1 として n→∞ とすれば (1-z)f(z)=1 を得る。
505502:2001/04/21(土) 18:42
蛇足ですが、無限積が収束することもちゃんと示す必要があります
506132人目の素数さん:2001/04/21(土) 18:43
0<m<1となるm∈Zがあるとする。そのうち最小のものをkとする。
このとき、1-k(>0)∈Z。
0<1-k<1 であり、kの最小性より0<k<1-k これより0<k<1/2
すると、0<1/2-k<1/2<1 であり、1/2-k∈Zであるから、kの最小性より
0<k<1/2-k これよりk<1/4
以下同様にして、任意の正整数Nに対し、0<k<1/N 矛盾
507504:2001/04/21(土) 18:46
>>505
そうですね。蛇足じゃないです。
508●●|●|●:2001/04/21(土) 18:54
>>499
>3問目

ようするに「係数が1の、n変数の高々d次の単項式」の総数が
(n+d)!/(n!d!) 個であることを示せばいい。これは
「係数が1の、n+1変数のd次“同次”単項式」の総数と同じ。
こっちのほうが数えやすいとおもう。
509132人目の素数さん:2001/04/21(土) 19:10
>>502 >>504
 ばっちりわかりました。
 ありがとうございました。
510132人目の素数さん:2001/04/21(土) 19:13
1<d<xをみたす、xの正の約数dがあるとする。x=d*kとおく。
k>1。d>√x and k>√x であると仮定すると、
x=dk>(√x)^2=x 矛盾。よってd≦√x or k≦√x
511493:2001/04/21(土) 21:01
>496
ありがとうございました。
出来れば後半の問題も教えて下さい。直観的に{0}になるのではないかと思うだけで、数学的にそうだと云う根拠はないので教えて下さい。
512132人目の素数さん:2001/04/21(土) 21:04
>>499 3問目

そうだね。次数揃えがいい。

重複組合せ H[n、r] = C[n+r-1、r] は既知としてよいのかな?

x1、x2、…、xn の n 変数による d 次多項式であるとすると、
これを同次式にする為に、形式的に各項に x(n+1) を
必要な個数だけ掛ける(こうする前後で項の個数は
変わらないので問題ない)。

すると、各項は x1、x2、…、x(n+1) の (n+1) 変数から
重複を許して d 個取ってきて作られているわけだから、
項の個数は高々

 H[n+1、d] = C[n+d、d] = (n+d)!/{d!(n+d -d)!} = (n+d)!/(n!d!) .
513蛆虫:2001/04/21(土) 21:14
>>493>>494

今井の実数では (-1/n、1/n) で整数 0 を表します。(w
514132人目の素数さん:2001/04/21(土) 21:43
>>511
任意の n に対して 0 ∈ (-1/n、1/n) であることはいいよね?
これより 0 ∈ ∩In .

さて、任意の実数 r (≠ 0)について、アルキメデスの原理より、
|r|n > 1 なる自然数 n が存在する。即ち |r| > 1/n .
∴ r not ∈ In = (-1/n、1/n) .
∴ r not ∈ ∩In .

以上より、∩In = {0} .
2次元の場合もやることは同じ。

※「∈」「∋」の否定記号って出ないのかな?
  かなり苦しい表記をしてしまいました…。
515511:2001/04/21(土) 21:53
>514
本当にありがとう。また、なにかあったら教えて下さい。
516484:2001/04/21(土) 23:55
>486さん >490さん


どうもありがとうございました
517132人目の素数さん:2001/04/22(日) 00:14
a[1]=1で任意の自然数nに対して
 a[n]>0および
 6Σ[k=1,n]a(k)^2=a[k]a[k+1]*(2a[n+1]-1)
を満たす。
一般項a[n]を求めよ。

高校の問題集でこんな問題があったけど解説が載ってないやつなので
ぜんぜん分かりません。
誰か教えてください。
518132人目の素数さん:2001/04/22(日) 00:16
式をもう一度かくにんして
なんかへんだよ
519132人目の素数さん:2001/04/22(日) 00:29
>>517
正しくは
6Σ[k=1,n]a(k)^2=a[n]a[n+1]*(2a[n+1]-1)
と思われ。2、3項計算してa[n]=nと推定できる。というか左辺と
右辺の“6”あたりからはは〜んとわかる。あとは帰納法で
それをしめせばよい。a[n]=nまでただしいとすると
a[n+1]は漸化式から2a[n+1]^2-a[n+1]-(n+1)(2n+1)=0を満たす。
以下やってみそ!
520519:2001/04/22(日) 00:32
>>519
右辺と左辺をまちがったとおもわれ。
521132人目の素数さん:2001/04/22(日) 00:41
「・・・と思われ」って言い方、
さいきん流行ってんの?
522132人目の素数さん:2001/04/22(日) 00:49
>>521
はやってないと思われ。
523132人目の素数さん:2001/04/22(日) 01:01
写像f:X→Yおよび、Xの部分集合系(Aλ|λ∈Λ)に対して、
f^-1(∪[μ∈M]Bμ)=∪[μ∈M]f^-1(Bμ)
を証明

左辺⇔f^-1(あるμ∈Mに対してx∈Bμ)
  ⇔{x|あるμ∈Mに対してx∈f^-1(Bμ)}
  ⇔左辺
これで証明できていますか? それともどこかおかしいでしょうか?
教えてください。お願いします。
524517:2001/04/22(日) 01:17
>>519
ありがとうゴザイマシタ
なんとかできました。
それでa[n]=nまで正しいとするとってあるけど
そうしないとa[n]よりまえの
例えばa[n-1]=n-1とかが使えないってことですか?
525To:2001/04/22(日) 01:19
Ty=y'は部分空間の線形変換であることを示せ、教えてください
526132人目の素数さん:2001/04/22(日) 01:39
>>525
記号の説明がどっかに載ってない?
Ty=y'だけじゃなんのことやら…
部分空間ってどの空間の部分空間なの?
527132人目の素数さん:2001/04/22(日) 03:21
N、Mが有限集合のとき、全射f:N→M はいくつありますか?
528132人目の素数さん:2001/04/22(日) 03:31
>>527
M>Nのときナシ
M≦NのときM^(M−N)
529132人目の素数さん:2001/04/22(日) 03:32
>>528
符号逆(鬱

M>Nのときナシ
M≦NのときM^(N−M)

530132人目の素数さん:2001/04/22(日) 03:42
>>529
・・・・・・・・・・・・・・・・・・・・・・・・・・・・・・・・・・・・・・・・・・・・・
531132人目の素数さん:2001/04/22(日) 04:48
2円 x^2+y^2-4x-6y+4, x^2+y^2=r^2
が共有点を持たないようなrの値の範囲は何ですか?
ただし,r>0
532132人目の素数さん:2001/04/22(日) 04:56
>>531
2円の中心と半径を求めろ
ヴァカ!!
533531:2001/04/22(日) 05:10
>>532
ありがとう、普通に分かった。
恩にきります。
534132人目の素数さん:2001/04/22(日) 06:28
教えてください。

金利(複利)計算なんですが、
年利xでn年預けるとほぼ2倍になるかという問題で、
x*n=(約)0.72
という式が成り立つみたいです。

どなたか、導いてくれませんか?
要するに、
(1+x)^n = 2 で n → ∞ のとき xn が何に収束するかということだと
思うのですが。

よろしくお願いします。
535132人目の素数さん:2001/04/22(日) 06:46
>>527
それって、簡単な形に求まるの?
次の式以上、どうにもならないように思うけど。
N、Mの要素の数を n, m とすると、

Σ[k=1,m](-1)^(m-k) mCk k^n
536519:2001/04/22(日) 07:29
>>524
>それでa[n]=nまで正しいとするとってあるけど
>そうしないとa[n]よりまえの
>例えばa[n-1]=n-1とかが使えないってことですか?
そうっす。漸化式の右辺にa[1]〜a[n]まで全部でてきてこれを全部1〜nに
おきかえないとしんどいのでa[1]=1〜a[n]=nまで全部仮定すると
a[n+1]=n+1も証明できるのでOKというLogicです。
537名無しさん:2001/04/22(日) 07:50
>>534
log2=0.6931471806
538132人目の素数さん:2001/04/22(日) 08:44
>>537さん
ありがとうございます。
x → 0 のとき、log(1+x)=x
を使えば良いんですよね。

0.69や0.7でなくて0.72と書かれていたので悩んだのですが、
「約数が多い(暗算しやすい?)」という単純な理由のような気がしてきました。
実際に想定されるnの値が、収束値に近づくには
いまいち小さすぎというのもあるのでしょうが。
#だったら、「数学的に導かれる値」とか書かないでくれ.......

世間で使われる数学(数字)って結構いいかげん。
539132人目の素数さん:2001/04/22(日) 09:09
ストルツの定理ってどんな定理なんですか?
540久保 康隆:2001/04/22(日) 13:21
空間に、3点A(1,-1,1)B(1,1,1)C(0,0,1)がある。直線
x=2+at
y=3+bt  (但し、tはパラメータ)
z=2+t

が三角形ABCと共有点を持つためのa、bの条件を求めよ。
また、その条件を満たす点(a,b)の範囲を表せ。但し、三角形ABCは周及び内部を含むものとする。
                                        (大阪府立大)
これがどうも分かりません。空間系は苦手なもので・・


--------------------------------------------------------------------------------
541左右田治:2001/04/22(日) 13:24
三角形ABCにおいて、∠Aの二等分線と辺BCとの交点をDとし、
その外接円の中心をOとする。

AB=2 AC=3 ∠A=θ 1/2ベクトルAB=ベクトルb 1/3ベクトルAC=ベクトルc
とするとき、次の問に答えよ。

(1)ベクトルADをベクトルb、cで表せ。また、ベクトルAOをベクトルb、c、θで表せ。

(2)OC⊥ADとなるとき、cosθを求めよ。
                              (86東大)

542132人目の素数さん:2001/04/22(日) 13:54
>>541

(1)|ベクトルb|=|ベクトルc|=1だからこのベクトルの張る三角形は2と右辺3角形なので
∠Aの2等分線はsをパラメータとしてs(ベクトルb+ベクトルc)/2
DはBC上にあるからtをパラメータとして2(1−t)ベクトルb+3tベクトルc
とも書ける。2式が等しいとおいてsとtを求めればベクトルADが求まる

ベクトルAOは∠Aの二等分線上にあるのでこれもs(ベクトルb+ベクトルc)/2
と書ける。sは外接円の半径だから、BCの長さを求めれば正弦定理よりでる。

(2)
(1)よりOCが求まり、(ベクトルb+ベクトルc)と直交してるので内積とって0より
543132人目の素数さん:2001/04/22(日) 14:37
>>542
×2と右辺3角形
○二等辺三角形

アホ変換
544左右田治:2001/04/22(日) 16:56
545132人目の素数さん:2001/04/22(日) 16:59
>>540
△ABCの内部はベクトルOC+pベクトルCB+qベクトルCA
(0≦p+q≦1,0≦p,q≦1)のように書けるので

(p+q,p-q,1)と表せる。
この点をとおるとしたら、z=1だからt=-1
なので、x,y,tに値を入れてa,bの条件をp,qに関する不等式から出す
546132人目の素数さん:2001/04/22(日) 16:59
微分の分野なんですけど、
Y=sin[tan(sec√(x二乗+5x-3)]
のdy/dxってどうやって求めたらいいんでしょう?
547132人目の素数さん:2001/04/22(日) 17:11
>>546
合成関数の微分
y=sin[tan(sec√(x^2+5x-3)]

dy/dx=cos[tan(sec√(x^2+5x-3)]d[tan(sec√(x^2+5x-3)]/dx
=cos[tan(sec√(x^2+5x-3)]sec^2[sec√(x^2+5x-3)]d[sec√(x^2+5x-3)]/dx
=cos[tan(sec√(x^2+5x-3)]sec^2[sec√(x^2+5x-3)]sin[√(x^2+5x-3)]sec[√(x^2+5x-3)]d[√(x^2+5x-3)]/dx
=cos[tan(sec√(x^2+5x-3)]sec^2[sec√(x^2+5x-3)]sin[√(x^2+5x-3)]sec[√(x^2+5x-3)](2x+5)/{2√(x^2+5x-3)}
548132人目の素数さん:2001/04/22(日) 17:23
ありがとうございました〜、公式みながら頭んなか整理してみます
549気合いをください:2001/04/22(日) 17:36
x,yの多項式f(x,y)=(x+y)^n+x^n+y^nがx,yの多項式
g(x,y)=x^2+xy+y^2を因数にもつという。このとき。自然数n
の満たす条件を求めよ。

非常に悩んでもわかりません、ぜひ教えてください。
550堀松 洋介 ウィークデー:2001/04/22(日) 18:04
三角錐OABC(Oがてっぺんにある)において、PはOAを2:1に内分する点、
QはOBを3:1に内分する点、RはBCの中点とする。
また、この三角錐を3点P,Q,Rを通る平面で斬った時、
この平面が線分ACと交わる点をSとする。

1)
ベクトルa=ベクトルOA、ベクトルb=ベクトルOB、ベクトルc=ベクトルOCとする。
3点P,Q,Rを通る平面の任意の点Xに対して、ベクトルOXを
ベクトルa、b、cで表せ。

2)
点SはACをある比で内分している。ベクトルOSをベクトルa、cで
表すことにより、その比を求めよ。    
                 (大阪大・後期)
これがどうしても分かりません。誰か教えてください。
551132人目の素数さん:2001/04/22(日) 18:16
f(x,y) = g(x, y)P(x,y)とおく。この時、wを1の3乗根とすると-w^2 = 1 + w。
g(1, w) = 0よりf(1,w) = (1 + w)^n + 1^n + w^n = (-w^2)^n + 1 + w^n = 0
これはn = 6n + 2, 6n + 4の時に限る。
この時、f(x, 1) = g(x, 1)P(x)となるP(x)は存在する。
P(x)の各項にyの適当な冪を加えて斉次式にする。
すると、f(x,y) = g(x,y)P(x,y)
となる。
552132人目の素数さん:2001/04/22(日) 18:17
>>549
T[n]:=f(x,y)、p=x+y、q=xyと置く
T[n]=pT[n-1]-qT[n-2]+q p^(n-2)
x^2+xy+y^2=p^2-qだからこれを因数にもつならq=p^2を代入して0になる

q=p^2をT[n]の漸化式に入れればT[n]が0に成るような条件が求まる
553132人目の素数さん:2001/04/22(日) 18:19
>>550
今日日曜だから学校の宿題か?
そろそろ自分で考えろ
どうしてもわからんというのは1週間くらい考えてから言え
554伊藤敦夫の超数学出題:2001/04/22(日) 18:23
・1+2+3+・・・・+200000を√をつかって表せ。
・(100+200+・・・300)+1は素数か、はたまた虚数か
555132人目の素数さん:2001/04/22(日) 18:25
>>554
おっこれ確か大正時代の帝国大学の入試問題じゃん!
見たことあるぜ
556132人目の素数さん:2001/04/22(日) 18:26
>>554
なんだこれ?
557132人目の素数さん:2001/04/22(日) 18:28
>>554=>>555
ジサクジエ-ン
558左右田治:2001/04/22(日) 18:28
>>554
キテレツな問題だな!
世界は広いねえ!!
559隅田川:2001/04/22(日) 18:29
>>554
つーかこれ「フロイトの哲学を数学に応用した問題」だろ?
未だに解けた奴がいないっていう。
560132人目の素数さん:2001/04/22(日) 18:35
554=555=558=559ダロ?
シュクダイオシエテモラエナクナッテ
イジケタノカ?(w
561厨房28号:2001/04/22(日) 19:06
>>554

1 + 2 + 3 + … + 200000 = √(1 + 2 + 3 + … + 200000)^2

(100 + 200 + … + 300) + 1 = 601 is prime.

ドウダ!(w
562132人目の素数さん:2001/04/22(日) 19:07
√(X+[X])+mX=2 (mは正の定数)のとき
異なる3つの実数解を持つような
mの範囲を求めよ
またこの範囲での最小解の取りうる範囲を求めよ
([X]はガウス記号です)
おねがいします

563隅田川:2001/04/22(日) 19:09
>>560
なんで俺がジサクジエンしなきゃなんね-ンだよ!!
ひでえって
564132人目の素数さん:2001/04/22(日) 19:09
560=554だろ?うぜえよ、氏ね
565鶴田:2001/04/22(日) 19:12
ガウスきごうって利用価値高いよな!
566132人目の素数さん:2001/04/22(日) 20:30
>(100+200+・・・300)+1は素数か、はたまた虚数か

本当はどう書きたかったのか予想せよ。(配点50)
567132人目の素数さん:2001/04/22(日) 22:39
>>562
Z会のやつだろおれもやった。
568549 :2001/04/22(日) 23:41
>>551
>>552
どうもありがとうございました。551さんのはとても自分には思いつかないですね。
私には552さんの解法のほうが合ってました。
569でじこ@数学板:2001/04/23(月) 00:00
>>527
異なる n 個の物を m 個の箱(区別しない)に空箱のないように
分配する方法の数は第2種スターリング数と呼ばれるにょ。
第2種スターリング数は

 S(n、m) = 納k=0、m]{(-1)^k *mCk *(m-k)^n}/m!

で与えられるにょ。

f: N → M (全射)の数は、M の m 個の元も区別するので、
それは m!S(n、m) となるにょ。つまり

 納k=0、m](-1)^k *mCk *(m-k)^n .

535さんは微妙に違うにょ。

これ以上は簡単にならないはずにょ…。
570527:2001/04/23(月) 00:17
>>535 >>569
ありがとうございます。
その式をどうやって出したのか聞きたいです・・・
ドキュソな俺に教えてください。

空間の2点A(3,7,1)、B(11、−1,9)の位置ベクトルをそれぞれ
a、bとし、A,Bを通る直線上の点Pの位置ベクトルをpとする。
│p│の値が最小になるときの△OAPの面積を求めよ。
ただし、Oは原点である。

お願いします。
572132人目の素数さん:2001/04/23(月) 03:25
562の問題の締め切りは4月26日です。
573132人目の素数さん:2001/04/23(月) 03:27
添削問題は自分で解かなきゃ意味なんだよ
574132人目の素数さん:2001/04/23(月) 03:40
つまり、>>562の問題の解答は少なくとも4月27日以降にしろってことだよな
575535:2001/04/23(月) 06:20
>>569
>第2種スターリング数と呼ばれるにょ。
知らなかったにゃ。第1種もあるなら教えて欲しいにゃ。

>535さんは微妙に違うにょ。
和の取り方が逆なだけで、同じなんだにゃ。

>>527=570
>その式をどうやって出したのか聞きたいです・・・

inclusion-exclusion principle を利用するのがひとつの方法。

母関数を利用する方法を以下に示す。
n個の集合から、m 個の集合への全射の数をs(m)とする(s(0)=0 としておく)。
以下、n は固定するが、m はあらゆるゼロ以上の整数をとるものとする。
次の式は、N→M の(全射とは限らない)すべての写像の総数を二通りに表したもの。
Σ[k=0,m]mCk*s(k)=m^n

これを変形して、
Σ[k=0,m]{1/(m-k)!}*{s(k)/k!}=(m^n)/m! ・・・・・・(1)

f(x), g(x) を次のように定める。
f(x)=s(0)+s(1)/1!+s(2)/2!+s(3)/3!+s(4)/4!+…
g(x)=0^n+(1^n)/1!+(2^n)/2!+(3^n)/3!+(4^n)/4!+…

また、
e^x=1+x/1!+(x^2)/2!+(x^3)/3!+(x^4)/4!+…
なので、(1)は、(e^x)f(x)=g(x) を表している。

よって、f(x)={e^(-x)}g(x) となるが、この式を展開し、x^m の係数を比較することで、
s(m)/m!=Σ[k=0,m]{(-1)^(m-k)}/(m-k)!*(k^n)/k!
を得る。
576132人目の素数さん:2001/04/23(月) 06:30
224=562 ?
577132人目の素数さん:2001/04/23(月) 06:31
添加した体って、どんな代物ですか?
578132人目の素数さん:2001/04/23(月) 08:07
たとえば、EがFの拡大体として、α∈Eに対して、
「体Fにαを添加した体」とは
「Fとαを含む最小の体」のことだ。
579577:2001/04/23(月) 10:18
>>578
明快な記述どうもです。
580132人目の素数さん:2001/04/23(月) 12:47
線形常微分方程式です。

1問目:y'(t)=a(t)*yの任意に解y1とし、y2を0でない解とするとき、任意のtについて
    y1(t)=C*y0(t) となる定数Cが存在することを示せ。

2問目:y'(t)=a(t)+bの任意の解をy0、y1、y2とするとき、y1−y0≠0ならば、tの関数
    F(t)=y2(t)−y0(t)/y1(t)−y0(t) は定数関数となることを示せ。

3問目:リッカチ型
    y'=ay^2+by+c の1つの解をy0とするとき、u=y−y0と置くと
    uについての方程式が求積可能なものになることを示せ

の3問です。
基本的な問題なのかもしれませんが、マジまいってます。誰か助けてください。お願いします。 
581132人目の素数さん:2001/04/23(月) 15:36
2つの確率変数pとqが独立であることの具体例と2つの確立変数rとsが
独立でないことの具体例をそれぞれ教えてください。お願いします。
582132人目の素数さん:2001/04/23(月) 18:44
y=sin[tan{sec√(x^2+5x-3)}]はどうやって解いたら良いんでしょう?
583ももんが:2001/04/23(月) 19:49
∫1/X4+1dx
これの答えを教えてくだしぇ
584132人目の素数さん:2001/04/23(月) 20:00
>>575
Leibniz rule

D^n(f_1・f_2・...・f_m)
= 農{t_1+t_2+...+t_m=n}D^{t_1}f_1・D^{t_2}f_2・...・D^{t_m}f_m

(D は任意の微分作用素)

を D = d/dt, f_i = (e^t-1) に適用して t=0 を代入したほがはやいかも。
585132人目の素数さん:2001/04/23(月) 20:23
>>582
何言ってるのかサッパリわかりません。
586でじこ@数学板:2001/04/23(月) 21:45
>>569
連続する m 個の自然数 n(n+1)(n+2)・…・(n+m-1) を展開したときの
n^k の係数を第1種スターリング数(T(m、k) と書こう)と呼ぶにょ。

 n(n+1)(n+2)(n+3) = 6n + 11n^2 + 6n^3 + n^4

より、T(4、1) = 6、T(4、2) = 11、T(4、3) = 6、T(4、4) = 1 にょ。

…で、実は(煤AnCr まで許して)T(m、k) の一般形を知らないにょ
(一般に知られていない、ではなく、私個人が知らないという意味)。
誰かご存知なら教えて欲しいにょ。

> >535さんは微妙に違うにょ。
> 和の取り方が逆なだけで、同じなんだにゃ。

535 の式では k = 0 の場合が欠落していると思ったんだけど、
よく考えると k = 0 だと k^n の部分が 0 となって、総和には
関わってこないにょ。というわけで 535 さんの式は正しかったにょ
(情けない…)。ゴメン。
587でじこ@数学板:2001/04/23(月) 21:50
上の「>>569」は「>>575」の誤りです。
重ね重ね申し訳ございません…。
588132人目の素数さん:2001/04/23(月) 22:13
>>587
でじこは謝らないぞ!(笑
それはともかく気にせず行こう。
589132人目の素数さん:2001/04/23(月) 23:26
>>587

でじこ@数学板 様

「目からビーム」はどうすればできるようになるのでしょうか?
590tr:2001/04/24(火) 02:48
>>550 さん
(1) OX = l*OP + m*OQ + n*OR (l+m+n=1)
     = (2/3)l*OA + {(3/4)m + (1/2)n}OB + (1/2)n*OC
(2) OS = (1-k)OA + k*OC と表せるから (1) を用いて
  { (2/3)l = 1-k
  { (3/4)m + (1/2)n = 0
  { (1/2)n = k
この 3式を連立させ k = 3/5 を得る。
  ∴ OS = (2/5)OA + (3/5)OC
  ⇒ AS : SC = 3 : 2
591tr:2001/04/24(火) 02:50
>>571 さん
  AB = (8,-8,8) = 8*(1,-1,1)
  ⇒ OP = OA + k*(1,-1,1)
と表せる。このとき,
  |OP|^2 = (3+k)^2 + (7-k)^2 + (1+k)^2
       = 3(k-1)^2 + 56
から |OP|^2 の min は 56 (k=1) とわかる。

△OAP = (1/2)√{|OA|^2|OP|^2 - (OA・OP)^2}
     = (1/2)√(59*56 -56^2)
     = (1/2)√(56*3) = √(42)
592575:2001/04/24(火) 06:33
>>584
すげぇ。圧倒的な早さだ。

>>586
第1種スターリング数、さんくすです。
593132人目の素数さん:2001/04/24(火) 06:59
>>580
[1問目] G(t)=y1/y2 とおいて、G'(t)=0 を計算で示すだけ。
(y1(t)=C*y0(t) は y1(t)=C*y2(t) のミスだよね)

[2問目] F'(t)=0 を計算で示すだけ。もしくは、u1=y1-y0, u2=y2-y0 とすると、
u'=a(t)*u なのでこれに [1問目] の結果を適用する。
(y'(t)=a(t)+b→y'(t)=a(t)*y(t)+b)

[3問目] u についての式を立ててから、z=1/u とおく。
594132人目の素数さん:2001/04/24(火) 14:05
y=√(3)x+2cosx O<=X<2π
これでタンジェントのラインが平行なポイントっていうのは
いくつなんでしょう???

y=x+sinx 0<=X<2π ってのが(π、π)っていうのはわかったんですけど・・・
595132人目の素数さん:2001/04/24(火) 14:12
>>594
なんかへんなカナ英語まじり。接線がx軸と平行の意か?
なら y'=√3 -2sinx = 0 を解くだけだけど。
596132人目の素数さん:2001/04/24(火) 14:18
すんません、呼び方がわからないもんで。
そのX軸と並行ってことです。
おれがやった限りではcosx=(√3)/2で
x=π/6 なんですけど・・・
597132人目の素数さん:2001/04/24(火) 14:30
>>596
どうやったんでしょう?わかんないけど次はしってる?

公式 曲線 y=f(x) の点 (t,f(t)) における接線は y-f(t)=f'(t)(x-t)
   特にその傾きは f'(t) である。

よって本問は傾き f'(t)=√3 -2sint が 0 になる t をさがせばよい。
つまり sint = √3/2 こうなる t を 0<=t<2π の範囲でさがします。
2つあるね。
598132人目の素数さん:2001/04/24(火) 14:38
あ、そっか、どうもです
599132人目の素数さん:2001/04/24(火) 15:22
y=x^2/√(x^9+9)のdy/dxは?
600132人目の素数さん :2001/04/24(火) 16:14
x,yが(log_2x)^2*(log_2y)^2=log_2x^3+log_2y^3を満たすとき
x/yの最大値と最小値を求めよ。

高校の問題です、誰か教えてください。
601:2001/04/24(火) 16:37
>>600
X = log_2x, Y = log_2y とおいて、、
とやるんでしょ。
602132人目の素数さん:2001/04/24(火) 16:40
>>599

y'=2x(x^9+9)^(-1/2)-(9/2)*x^10*(x^9+9)^(-3/2)
603132人目の素数さん:2001/04/24(火) 16:44
>>600

X=log_2x, Y=log_2y とおいて X^2*Y^2=3(X+Y) として、X-Y の最大値と最小値を求めよ
604132人目の素数さん:2001/04/24(火) 16:45
かぶった・・・・
605580です:2001/04/24(火) 17:15
>593
本当にどうもありがとう。ミスちゃてごめんなさい。
606132人目の素数さん:2001/04/24(火) 19:00
統計の問題なんですけど、

「棄却検定法について、“標準偏差”“正規分布”などの概念を
 ふまえて説明しなさい」

ってのがよくわかりません。どなたか教えていただけないでしょうか。
607132人目の素数さん:2001/04/24(火) 19:19
>>606

【解答例】
棄却検定法は、標準偏差や正規分布を利用します。
(おわり)
608132人目の素数さん:2001/04/24(火) 20:36
有理係数のn次多項式f(x)∈Q[x]はQ上で既約であるとする。
f(x)の根の一つをαとすると、
Q[α]={a_{0} + a_{1}α + … + a_{n-1}α^{n-1} ; a_{j}∈Q}
はCの部分体である。

このことを用いて
「代数的数全体の集合は体である」
ことを示せないものでしょうか??
ご意見ください。
609名無しゲノムのクローンさん:2001/04/24(火) 22:06
lim[n->∞]x_n = a
lim[n->∞]y_n = b
lim[n->∞]x_n*y_n=abを証明するとき、
lim[n->∞]x_n*y_n=1/4*lim[n->∞]{(x_n+y_n)^2-(x_n-y_n)^2}
=1/4*{(a+b)^2-(a-b)^2}=...
としてよいのは何故ですか?
610132人目の素数さん:2001/04/24(火) 22:26
>>609
それって、だめじゃねぇ?
611名無しゲノムのクローンさん:2001/04/24(火) 23:07
>>610
途中の計算省略しているだけ?
612132人目の素数さん:2001/04/24(火) 23:09
>>609
lim[n->∞]x_n = a
lim[n->∞]y_n = b
⇒lim[n->∞]x_n*y_n=a*b
だから良いよ。
613132人目の素数さん:2001/04/24(火) 23:41
a, bが有限の値に収束するならな。
614132人目の素数さん:2001/04/25(水) 00:30
>>609
>としてよいのは何故ですか?

だめですよ。
(x_n+y_n)^2 → (a+b)^2 を示すのに
まさにこれから示したい積の極限を使ってしまってます。
証明はε−δで。

(x_n)±(y_n) → a±b
(x_n)(y_n) → ab
1/(x_n) → 1/a (a≠0)

高校生は↑を証明なしに使ってもいいのかな。
って優香、証明の術を教わらないような気がする。(^_^;
615132人目の素数さん:2001/04/25(水) 00:54
>>614
それは、一般的な命題として示すなら必要だけど
多分>609のは何か他の計算の途中じゃないかな?
616132人目の素数さん:2001/04/25(水) 01:03
「…としてよい」ってどこで誰が言ったんだ?>>609
617名無しゲノムのクローンさん:2001/04/25(水) 01:22
εーδの本の問題なのに
いきなりこういうのが載っていたので...
>>610,611,612,613,614,615,616
618614:2001/04/25(水) 01:23
>>615
609の3行目に、
>lim[n->∞]x_n*y_n=abを証明するとき、
とあるから、まさにこの命題を示す場合だと思って。
619614:2001/04/25(水) 01:41
お、617=609ですよね?

解析概論を丸写しすると、
lim[n->∞]x_n = a
lim[n->∞]y_n = b のとき
|(x_n)| , |(y_n)| < M を満たすMが存在して
|a| , |b| < M

(x_n)(y_n) - ab = ((x_n)-a)b + (x_n)((y_n) - b) より
|(x_n)(y_n) - ab| <= M(|(x_n) - a| + |(y_n) - b|)
nを十分大きくすれば↑の右辺はいくらでも小さくなるので
左辺 → 0 , すなわち(x_n)(y_n) → ab
620カンケイ無いけど:2001/04/25(水) 01:48
>>610-616」と書けばいい>>617
621名無しゲノムのクローンさん:2001/04/25(水) 02:17
>>619さんへ
元の問題は

x_n*y_n=1/4{(x_n+y_n)^2-(x_n-y_n)^2}を利用して

とあります。
これを必ず利用して解いてみてください。

622132人目の素数さん:2001/04/25(水) 02:23
>>621
問題の全文をここに書いて下さい。
623132人目の素数さん:2001/04/25(水) 06:53
>>581
[1] サイコロを2回振った場合の例。
(a) 次の2つは独立。(関係ないから)。
p:1回目の数。 q:2回目の数。
(b) 次の2つは独立でない。(r が大きいと s も大きくなりやすいから)。
r:1回目の数。 s:1回目の数と2回目の数の合計。

[2] 経済の例。
(a) 次の2つは独立。(たぶん関係ないと思う)。
p:日経平均株価。 q:公務員の給料。
(b) 次の2つは独立ではない。(世界経済はリンクしているから)。
r:日経平均株価。 s:アメリカのナスダック総合指数。

[3] 野球の例。
(a)次の2つは独立。(あっちはあっち、こっちはこっち)。
p:巨人の成績。 q:イチローの打率。
(b)次の2つは独立でない。(松井が打てば、巨人は勝ちやすいから)。
r:巨人の成績。 r:松井の本塁打数。
624132人目の素数さん:2001/04/25(水) 07:12
>(a) 次の2つは独立。(たぶん関係ないと思う)。
>p:日経平均株価。 q:公務員の給料。

まったく「独立」だとは言えなくないか?
625580です:2001/04/25(水) 11:44
線形常微分方程式です。

1問目:y'(t)=a(t)*yの任意に解y1とし、y2を0でない解とするとき、任意のtについて
    y1(t)=C*y2(t) となる定数Cが存在することを示せ。

2問目:y'(t)=a(t)y+bの任意の解をy0、y1、y2とするとき、y1−y0≠0ならば、tの関数
    F(t)=y2(t)−y0(t)/y1(t)−y0(t) は定数関数となることを示せ。

3問目:リッカチ型
    y'=ay^2+by+c の1つの解をy0とするとき、u=y−y0と置くと
    uについての方程式が求積可能なものになることを示せ。
>580

[1問目] G(t)=y1/y2 とおいて、G'(t)=0 を計算で示すだけ。

[2問目] F'(t)=0 を計算で示すだけ。もしくは、u1=y1-y0, u2=y2-y0 とすると、
u'=a(t)*u なのでこれに [1問目] の結果を適用する。
(y'(t)=a(t)+b→y'(t)=a(t)*y(t)+b)

[3問目] u についての式を立ててから、z=1/u とおく。
>593
1問目がやっぱりわかりません。
G(t)=y1/y2をtでどう微分するんですか?ほんと、ドキュンな質問してごめんなさい。

2問目:F(t)=y2(t)−y0(t)/y1(t)−y0(t)を微分して
F'(t)=y2'(t)*(y1(t)−y0(t))−y1'(t)*(y2(t)−y0(t))+y0'(t)*(y2(t)−y1(t))/{y1(t)−y0(t)}^2
となり、
u1(t)=y1(t)−y0(t) u2(t)=y2(t)−y0(t) u2(t)−u1(t)=y2(t)−y1(t) とし代入して
F'(t)={y2'(t)−y0'(t)}*u1(t)−{y1'(t)−y0'(t)}*u2(t)/分母
y'(t)=a(t)y+bここで、y'(t)=a(t)y+bより
y2'(t)=a(t)y2+bと考えてもよろしんですか?もし、許されるなら
F'(t)=a(t)*{y2(t)−y0(t)}*u1(t)−a(t)*{y1(t)−y0(t)}*u2(t)/分母 となり
u1(t)=y1(t)−y0(t) u2(t)=y2(t)−y0(t) を代入して
F'(t)=a(t)*u2(t)*u1(t)−a(t)*u1(t)*u2(t)/分母  となり
分子=0より(分母≠0)  F'(t)=0 で宜しいですか?

3問目:y=u+y0として、y'=ay^2+by+c に代入して
y'=au^2+(2ay+b)u+y0'⇔y'−y0'=au^2+(2ay+b)u とし
u'=au^2+(2ay+b)u で、z=1/uを代入するんですか?

誰か助けてください。お願いします。
626名無しゲノムのクローンさん:2001/04/25(水) 12:56
>>622
lim[n->∞]x_n = a
lim[n->∞]y_n = b
であるとき、
lim[n->∞]x_n*y_n=ab
を証明せよ。
x_n*y_n=1/4*{(x_n+y_n)^2-(x_n-y_n)^2}
を利用して証明してみよう。
627132人目の素数さん:2001/04/25(水) 13:03
簡単なことだと思いますが複素数の問題で
   c=p+qi,z=x+yi(p,q,x,y,aは実数)とするとき
   2(px+qy)=aの直線はなぜベクトル(p,q)
   に垂直ということがわかるんでしょうか。
わからないのでお願いします。
628132人目の素数さん:2001/04/25(水) 14:17
>>627
直線が原点を通るように平行移動すりゃ
px+qy=0だろ?コレ(p,q)と(x,y)の内積
629132人目の素数さん:2001/04/25(水) 14:24
>>625
1問目:y1=G(t) y2をy1'(t)=a(t)*y1に代入してy2'(t)=a(t)*y2を使いたまへ
2問目:いきなりF’を計算するのではなく、y1−y0≠0なんだからこれは1問目のy2のことで
任意の解は適当な定数Cが存在してC y2に等しいという1問目の結果を使いたまへ
3問目:とりあえず変数変換してみたまへ
630132人目の素数さん:2001/04/25(水) 15:19
y=x^2×e^(-x)のdy/dxって2xe^(-x)でいいんですか?
631132人目の素数さん:2001/04/25(水) 15:22
>>623,>>624さんレスどうもありがとうございました。
632132人目の素数さん:2001/04/25(水) 15:31
>>630
ダメ。
f(x)=x^2、g(x)=e^(-x)としてy=f(x)g(x)だと思って
積の微分(fg)'=f' g+f g'を使いたまへ
633132人目の素数さん:2001/04/25(水) 15:38
じゃーy'=(x^2)'(e^(-x))+(x^2)(e^(-x)'ですか?
でもeの方の展開の仕方がわかんないです・・・

y'=2x*e^(-x)+-x^2*e^(-x)であってます?
634132人目の素数さん:2001/04/25(水) 15:47
>>633
ダメ。
(e^(-x))'
これはf(x)=e^x、g(x)=-xと思って
合成関数の微分(f(g))'=f'(g) g' を使いたまへ
635132人目の素数さん:2001/04/25(水) 15:56
そこんところがよくわかってないんですよね・・・
んじゃ 2xe^(-x)-(x^2)(-e^(-x))
=2xe^(-x)+(x^2)(e^(-x))
ですか?
636132人目の素数さん:2001/04/25(水) 16:00
>>635

>y'=2x*e^(-x)+-x^2*e^(-x)

良くみたら+の後に−入れてたねコレでいいよ
気付かなかった
637132人目の素数さん:2001/04/25(水) 16:05
ありがとうございました〜、明日までに提出の問題たくさんなもんであせってます。
全部終わったらもう一回参考書見直します。
638132人目の素数さん:2001/04/25(水) 16:54
x[n]=-a*n^2+b*n+c n=1,2,3・・・とする
このとき、次の2つの条件A,Bを満たす自然数a,b,cを求めよ
A:4,x[1],x[2]はこの順で等差数列である。
B:すべての自然数nに対して
{(x[n]+x[n+1])/2}^2 >= x[n]*x[n+1]+1 が成り立つ

お願いします
639RAYNE:2001/04/25(水) 17:02
わたしは大学一回生です。 授業で離散数学を取っているのですがどうしてもでき
ない問題があります。ともだちも分らないらしいです。
問題は・・・
A,B及びCが有限集合ならば、A∩B∩Cも有限であり、等式
n(AUBUC)=n(A)+n(B)+n(C)−n(A∩B)−n(B∩C)−n(A∩C)+n(A∩B∩C)
が成立する。この結果を任意の有限個の集合の場合に一般化せよ
です。よろしくお願いします。
640名無しさん:2001/04/25(水) 18:35
52枚のトランプから3枚のカードを同時にひいたとき、それらのカード
全てが絵札である確率は?

即急にお願いしますね
641BAKA:2001/04/25(水) 19:00
絵札って何?
何枚かあるの?
642625です。:2001/04/25(水) 19:08
>629
がんばってみます。分からなくなったら、またアドバイスお願いします。
643:2001/04/25(水) 20:35
トランプで絵札といえばおそらく J,Q,Kのことと思われる。
Aや10がはいるかどうかは断った方がマギレはないが。。。
644 :2001/04/25(水) 20:49
http://saki.2ch.net/test/read.cgi?bbs=lobby&key=988189685
ここの態度のデカイドキュン文系コテハンを徹底的に叩いてください
645↑「電撃」のことか?:2001/04/25(水) 20:55
646名無しさん:2001/04/25(水) 21:04
>>643
絵札は全部で12枚です
64712C3/52C3:2001/04/25(水) 21:12
 
648132人目の素数さん:2001/04/25(水) 21:21
標準偏差の取り得る範囲って何?
649132人目の素数さん:2001/04/25(水) 21:23
早稲田大学の学籍番号のCDってどうやって出すの?
650132人目の素数さん:2001/04/25(水) 22:50
>>639
Cのところを(CUD)で置き換えた式
n(AUBUCUD)=n(A)+n(B)+n(CUD)−n(A∩B)−n(B∩(CUD))−n(A∩(CUD))+n(A∩B∩(CUD))

まずこの右辺を全てUを使わない形にばらしてみたまへ
どういう式が一般式なのか予想がつくと思われ
651お願いしますぅ:2001/04/25(水) 23:00
『ある部族の長老に、年齢を聞きました。
長老は、「いくらワシでも、500歳はいっとらん。」と答えました。
そして、「ワシの歳に5を足すと13で割り切れる数になる。
そして、ワシの歳から5を引くと31で割り切れる数になる。」
さて。この長老はいくつでしょう?』

数字を並べるんじゃない方法であります?
652132人目の素数さん:2001/04/25(水) 23:02

既出。
653132人目の素数さん:2001/04/26(木) 00:31
654593:2001/04/26(木) 06:42
>>580=625
すでに解決済みかも知れないけれど、念のため。

[1問目] G(t)=y1(t)/y2(t) を t で微分するわけ。
G'(t)={y1'(t)y2(t)-y1(t)y2'(t)}/y2(t)^2
となる。この式に、y1'(t)=a(t)y1(t), y2'(t)=a(t)y2(t) を代入するとゼロになる。

[2問目] 593 に書いたヒントは、次の2つの解法があるという意味。
(a) F'(t)=0 を直接計算して示す。
(b) u1=y1-y0, u2=y2-y0 とおいて、[1問目] の結果を利用する。

(a)の方針でやるなら、u1, u2 なんか使わなくてもいいですよ。(模範解答は (b) の方だと思う)。

>y'(t)=a(t)y+bここで、y'(t)=a(t)y+bより
>y2'(t)=a(t)y2+bと考えてもよろしんですか?

y'(t)=a(t)y(t)+b の解が、y0(t), y1(t), y2(t) なんだから、次の三つの式が成立します。
y0'(t)=a(t)y0(t)+b, y1'(t)=a(t)y1(t)+b, y2'(t)=a(t)y2(t)+b

>で宜しいですか?
計算のやり方が回りくどいんですが、正解です。

[3問目] 最後の式を u^2 で割って(本当は u=0 についての吟味が必要ですが)、
u'/u^2=a+(2ay0+b)*(1/u)
これより、z についての微分方程式に直せるでしょう。(あなたの式にはミスがあり、y → y0 です)。
その「 z についての微分方程式」は、よくあるタイプで、求積可能です。
655132人目の素数さん:2001/04/26(木) 06:59
>>608
どういう意味ですか。

「体であることの証明は知っているけれど、上記のことのみを使った証明法はないものか」
ということでしょうか?

それとも、証明を知らなくて、「あまり予備知識のいらない証明法を教えて欲しい」という
ことでしょうか?
656恐縮です。1です。:2001/04/26(木) 12:52
>654
657580です:2001/04/26(木) 12:54
>654
マジありがとう。
658TKB:2001/04/26(木) 14:48
>>329 いまさら掘り起こし。
659高校受験:2001/04/26(木) 15:09
たぶんものすごく簡単な問題だと思うんですど、解けないっス。
どうか教えてくださいまし。m(..)m

1)ある仕事をするのにAは16日、Bは4日かかる。
  この仕事を初めはBがやっていたが、わけあって途中でAと交代した。
  Aは残りを7日で仕上げた。。。
  このとき、交代するまでにBは何日間仕事をしていたでしょうか?

2)下のような4桁と5桁の足し算において、A〜Iには1〜9の
  異なる数字が入る。E=9 B=2 F=6 とすれば、
  D+G+Hの値は あ〜お のうちどれか?

    ABCD
  +AEFGH      あ/10 い/11
 ────────     う/12 え/13
  =IGEHC      お/14

3)任意の三角形ABCにおいて、
  辺ABを三等分した時にAに近いほうの点を<あ>
  辺BCを四等分した時にBに一番近い点を<い>
  辺CAを五等分した時にBに一番近い点を<う> とする。
  このとき、三角形あいうの面積は三角形ABCの約何%になるか?

以上、お願いします。

660132人目の素数さん:2001/04/26(木) 15:11
http://saki.2ch.net/lobby/index2.html#1
ロビーでの確率論
痛い女が一人いるよ・・・「電撃少女」
文系らしいです
661TKB:2001/04/26(木) 15:22
>>329 いまさら掘り起こし。

前提は以下の3点。
・3本のくじ引きを3人で順番に引く。
・当たりは1本しかない。
・あなたは2番目に引く。

何の気なしに引けば、当たりを引く確率は(2/3)*(1/2)で1/3になる。
しかし

・1人目が引く前にあなたは1本のくじに(心の中で)しるしをつけておく。
・1人目がハズレを引いてあなたの番が回ってきた時に、しるしをつけたくじが残っていたとする。

この条件を追加すると>326と同じ要領で
しるしをつけたくじではなく、
逆を選んだ方が当たる確率は高くなる。

このことから、
心の中でしるしをつけるという行為が確率の変動に関わる、
という結論が得られる。

・・・・・・

以上、間違いを指摘してください。
−−−−−−−−−−−−−−−−−−−−−−−−−−−−−−
さて、ABC3本のうち
仮マークしたくじをAとすると
当たりがAのとき(1/3):そのままなら当たり
当たりがBかCのとき(2/3):
そのうち1/2(通算1/3):結局同じ(当たりを抜かれてしまうので)
のこりの1/2(通算1/3):かえたら当たり
つまりは残りの2枚の勝算は1/3:1/3=1:1。

では>>326のケースの場合、
当たりがC(1/3):Bがオ−プンされる。
当たりがB(1/3):Cがオープンされる。
当たりがA(1/3):どちらでもオープンされる可能性がある。
相手の癖を正確につかんでいないなら確率は五分五分と考えるべき。
つまり Bがオープン=1/6 Cがオープン=1/6

そして結局Bがオープンしたとして(仮にCでも”同様に、”で済むのだが。)
Aが当たり:Cが当たり=1/6:1/3=1:2となる。
662名無しゲノムのクローンさん:2001/04/26(木) 16:18
663ご冗談でしょう?名無しさん:2001/04/26(木) 17:37
ファジィは数学から見てどんな物なんでしょ
664名無しさん@お腹いっぱい。:2001/04/26(木) 17:52
数学ではないのですが、
1+1+1=555に線を一本引いて
式が合うようにする問題があるのですが、
どうすればよいのでしょうか?
665132人目の素数さん:2001/04/26(木) 18:08
>>664

 何度も何度もがいしゅつだけど・・・・。
 「式が合うように」じゃなくて「正しい等式に」じゃないと
 「1+1+1≠555」って答えるやつがいるから問題文訂正しようね。
666132人目の素数さん:2001/04/26(木) 18:09
ごめそ。新作か
667132人目の素数さん:2001/04/26(木) 18:13
>>664
5+5+5 = 550 じゃネーの?
Ans.5+545 = 550
668132人目の素数さん:2001/04/26(木) 19:19
>>665
正しい等式ってなに?
669高校受験:2001/04/26(木) 21:04
だれか659よろ。。。
670668:2001/04/26(木) 22:46
>>665
つまらんこといってスマソ
671132人目の素数さん:2001/04/26(木) 22:49
1+1+1=555 無理っぽい。無理矢理やるとすれば、
1+11+1=555  右辺は十進数、左辺は552進数
672132人目の素数さん:2001/04/27(金) 00:14
ξw_{1} = c_{i1}w_{1} + c_{i2}w_{2} + … + c_{il}w{l} (c_{ij}∈Q)

が 1≦i≦l で成り立つ。従って

|c_{11} - ξ c_{12} … c_{1l} |
| c_{21} c_{11} - ξ … c_{2l} | = 0
|・・・・・・・・・・・・・・・・・・・・・・ |
| c_{l1} c_{l2} … c_{1l} - ξ|

となる。(↑行列式のつもりです↑)左辺を展開して、

ξ^{l} + c_{1}ξ^{l-1} + … +c_{l} = 0

と、代数的整数論(高木貞治著)にあるのですが、
この行列式が何物なのかわかりません。(展開の仕方もアヤしいです)
初歩的な質問でしょうが、どうか教えてください。
673132人目の素数さん:2001/04/27(金) 00:34
原点O(0.0)と点P(pr/p^2+q^2,qr/p^2+q^2)との距離を求めよ。

↑何方かこの問題の解き方と答えを教えていただけませんか?
厨房のうえに数学苦手なんです...お願いします。
674132人目の素数さん:2001/04/27(金) 00:39
c_{1} = -tr {c_{ij}}, ....., c_l = (-1)^l det {{c}_{ij}}
じゃないの?単にC - \xi Idの行列式を取っているように
見えるのだけれど。
675132人目の素数さん:2001/04/27(金) 02:29
>>673
距離の定義を教科書で確かめてください。
676tr:2001/04/27(金) 03:37
>>638 さん
  条件A ⇔ 2*x[1] = 4 + x[2] ………(A')
  条件B ⇔ (x[n] - x[n+1])^2 ≧ 4 …(B')
(A') に
  x[1] = -a + b + c
  x[2] = -4a + 2b + c
を代入して
  2a + c = 4 ⇒ (a,c) = (1,2) (∵ 自然数)
また (B') に
  x[n] = -n^2 + bn + 2
  x[n+1] = -(n+1)^2 + b(n+1) + 2
       = -n^2 + (b-2)n + (b+1)
を代入して
  {n - (b-1)/2}^2 ≧ 1

上式の左辺を f(n) (n は自然数) とおくと
min{f(n)} は, 以下の 3パタンにわかれるから
  i) (b-1)/2 < 1 ⇒ f(1) = {(3-b)/2}^2
  ii) (b-1)/2≧1, b:奇数 ⇒ f((b-1)/2) = 0
  iii) (b-1)/2≧1, b:偶数 ⇒ f((b-2)/2) = f(b/2) = 1/4
適するのは i) で, b = 1 を得る。

以上まとめて (a,b,c) = (1,1,2)
677tr:2001/04/27(金) 03:41
>>659 さん
1) B が x日間仕事をしたとして
  (1/4)x + (1/16)*6 ≦ 1 < (1/4)x + (1/16)*7
  ⇒ 9/4 ≦ x < 10/4
# で, どう答えればよいのでしょうか?(汗)

2) I-1 = A = G+1 で 2, 6, 9 が使えないから
  (A, I, G) = (4, 5, 3)
繰り上がりの存在により C + G ≧ 11 で
  C + G = 8 + 3 ⇒ C = 8
  ⇒ (D, H) = (7, 1)
∴ D + G + H = 11 で 「い」

3) 問題文を訂正し
辺CA を 5等分した時に 「C」 に一番近い点を <う> として
  △あBい = (2/3)*(1/4)*△ABC
  △いCう = (3/4)*(1/5)*△ABC
  △うAあ = (4/5)*(1/3)*△ABC
であるから
  △あいう = △ABC - △あBい - △いCう - △うAあ
       = (1 - 35/60)△ABC
∴ (25/60)*100 = 125/3 = 41.66… ≒ 42 [%]
678tr@訂正:2001/04/27(金) 03:46
>>677 問1) の 「≦」 と 「<」 が逆!訂正願います。<(_ _)>

これ以外にも間違いがあったら, 適宜修正願います。(爆)
679132人目の素数さん:2001/04/27(金) 03:47
あの〜、くだらない質問で恐縮ですが、
数の位ありますよね、千とか万とか。
でね、最後の5つのうちどうしても1つだけ
思いだせないのです。

漢字分かりませんが、
なゆた、あそうぎ、不可思議、無量大数

あと一個何でしたっけ?
680132人目の素数さん:2001/04/27(金) 03:49
こうがしゃとか?
681132人目の素数さん:2001/04/27(金) 03:56
>680
おお!
そう言われればそんな感じですよね。
『こうかしゃ』か『こうがしゃ』の
どっちかですよね、多分。
ありがとうございましたー。
11年間もやもやのがす〜っと取れました。
くだらない質問してすいませんでした。
では。
682132人目の素数さん:2001/04/27(金) 05:04
683132人目の素数さん:2001/04/27(金) 12:01
コーシの収束条件わけわかりまへん。
教えてくださいな
684132人目の素数さん:2001/04/27(金) 12:31
微分積分の分野の話です。


区間[a,b]における連続関数f(x),g(x)に対しても、シュヴァルツの不等式

|∫f(x)*g(x)dx|≦(∫|f(x)^2|dx)^1/2*(∫|g(x)|^2dx)^1/2 @

3角不等式

(∫|f(x)+g(x)|^2dx)^1/2≦(∫|f(x)^2|dx)^1/2+(∫|g(x)|^2dx)^1/2 A

@,Aそれぞれが成立することはどう証明すればよいのですか?
誰か証明の仕方を教えてください。お願いします。
685132人目の素数さん:2001/04/27(金) 12:37
y"-5y'+6y=0を線形代数の範囲で微分方程式として解く方法を教えてください。
686132人目の素数さん:2001/04/27(金) 12:43
>>684

なぁ、こういう質問する前に微分積分じゃなくて、ふつーの
こーしー・しゅヴぁるつの不等式と三角不等式の証明を見て理解したら?

積分が単なる和であることさえわかれば、こんなヴァカな質問は
イマイと同レベルだぜ
687132人目の素数さん:2001/04/27(金) 12:49
>>685
>線形代数の範囲で・・・

2次方程式の知識で十分だよ
線形微分方程式なんだから特解の重ねあわせで一般解はでる。
特解をy=e^(λx)とおくと
λ^2-5λ+6=0 → λ=2,3 → y=C1*e^(2x)+C2*e^(3x)
688132人目の素数さん:2001/04/27(金) 12:50
>>683

教科書を読めヴァカ!
689132人目の素数さん:2001/04/27(金) 12:58
age
690132人目の素数さん:2001/04/27(金) 13:04
691132人目の素数さん:2001/04/27(金) 13:04
692132人目の素数さん:2001/04/27(金) 13:05
693132人目の素数さん:2001/04/27(金) 13:06
694672:2001/04/27(金) 13:18
>>674
\xiってなんでしょう?
695132人目の素数さん:2001/04/27(金) 13:39
>>694

ξ
696高校受験:2001/04/27(金) 13:45
>>677さん
うぉーすげぇー! 賢い〜
1)、2)は納得できました。ありがとうございます。
3)がまだよくわからないんスけど、
 △あBい=(2/3)*(1/4)*△ABC
これなんでですか〜 σ(´へ`)??
697132人目の素数さん:2001/04/27(金) 16:07
誰か助けてください。この2問どうやって、解くんですか?

DをR^nの開部分集合とする。
1問目:f or ∀x∈D、∀v∈R^n、∃r∈R(r>0)
s.t 0≦|t|<r(t∈R)⇒x+tv∈D を示せ。

2問目:x∈D、v∈R^nを一組固定する。
g(t)=f(x+tv)が開区間(-r、r)で微分可能のとき次を示せ。
g'(t)=f'(x+tv;v) (∀t∈(-r、r))

お願いします。
698132人目の素数さん:2001/04/27(金) 17:02
n次正方行列A、n次列ベクトルBに対し
A*x = b (det A = 0)
となるとき、どのようなことがわかるのでしょうか?
699132人目の素数さん:2001/04/27(金) 17:21
Aが逆行列を持たないことがわかる。
700132人目の素数さん:2001/04/27(金) 17:38
>>697
Dが開集合じゃから、テキトーな正数 ε をとると
点 x の ε-近傍がDに含まれるワイのう。さすれば
r = ε/|v| とでもとれば x+tv はDに含まれるじゃろうて。
701高校受験:2001/04/27(金) 17:48
もう一問いいでしょうか?
 一辺の長さが10cmの正方形がある。
 これの四隅からXcmの正方形を切り取り、
 残った部分を折り曲げて直方体の容器を作る。
 Xが何cmのとき容積は最大になるかσ(´へ`)??
なんか回答見ると、「容積=f(X)を微分して0とすると」
とか書いてるんですけど、なんで微分するのかがわかんないっス。
702名無しゲノムのクローンさん:2001/04/27(金) 17:51
極大値
703132人目の素数さん:2001/04/27(金) 18:54
>>701
Xを動かしたとき容積がどのようになるかを
調べる為です
要はグラフを書くため
っつーかあなた勉強まじめにしたことありますか?
704φ人:2001/04/27(金) 19:02
微分しなくても平方完成すればよろし。
どっちでも一二^H^Hいいあるよ。703が言うようにグラフ書くあるよろし。
705132人目の素数さん:2001/04/27(金) 19:34
集合と位相の問題なんですけど、
|A|≦|B|,|B|<|C|⇒|A|<|C|
の証明を教えてください。お願いします。
706>705:2001/04/27(金) 20:37
この問題の
大小関係(順序)って何をもって定義してるの?
707664:2001/04/27(金) 20:59
1+1+1=555に線を一本引いて正しい等式にする。
やっぱり無理ですかねえ。
708697です。:2001/04/27(金) 21:04
>700
ヒントありがとう。がんばってみます。わかんなくなったらまた、助けてください。
709132人目の素数さん:2001/04/27(金) 23:00
>>706さん
|A|≦|B|:=AからBへ単射が存在
|A|<|B|:=AからBへ単射が存在するが、BからAへの単射は存在しない
です。
710608:2001/04/27(金) 23:16
>>655
遅れて申しわけありません。後者の方であります。
711698:2001/04/27(金) 23:20
>>699
xについては何か言えないものでしょうか??
712132人目の素数さん:2001/04/27(金) 23:41
―――――――――――――――――――――――――――――――
円C: x^2 + y^2 = r^2、楕円D: x^2/a^2 + y^2/b^2 = 1
(ただし、a > b > r)を考える。
「円C上の点PにおけるCの接線が楕円Dによって切り取られる
線分の長さ」を最大にする点Pの座標を決定せよ。
―――――――――――――――――――――――――――――――

問題文にヘンなところないですよね??(ちょっと自信なし…)
713132人目の素数さん:2001/04/28(土) 00:20
>>705=709
706 じゃないけど、それってものすごく簡単じゃないのか。

A から B への単射が存在して、B から C への単射も存在すれば、それらを合成することで、
A から C へ単射になるぞ。

もし、C から A への単射が存在するなら、A から B への単射と合成することで、
C から B への単射ができてしまうぞ。
714132人目の素数さん:2001/04/28(土) 00:32
>>713さん
|A|≦|B|,|B|≦|C|⇒|A|≦|C|
これだったら、そういう考えで簡単にできると思うのですが、
<←これの定義が何となくしっくりきません。

それとも、証明はそれでOKなのでしょうか?
715713:2001/04/28(土) 01:03
>>714
しっくりくるかどうかはともかくとして、
次の2つを示せば証明完了ということはいいんでしょう?

(1) A から C へ単射が存在すること。
(2) C から A への単射が存在しないこと。

なら、粛々とその2つを示せばいいと思うんだけど、違う?
716132人目の素数さん:2001/04/28(土) 01:13
あってるでしょ。何処の点が納得できないんだろう。
717132人目の素数さん:2001/04/28(土) 01:44
>>608 さんと >>672 さんは同じ人なの?
記号の使い方が似ている・・・。

代数的整数が環をなすことがわかるなら、代数的数が体をなすことも
わかりそうなものだけど?
718608=672:2001/04/28(土) 03:19
>>717
大学の講義では、最近環に入ったばかりでして…。(今日、準同型しました)
しかし、私は体が必要なので…ハァハァ逝っています。
けど、おかげさまでなんとか解決できました。
やっぱり、自分で予習しておけってことですかねぇ?
719132人目の素数さん:2001/04/28(土) 03:44
>>718
授業なんか出ずに一人で勉強しろということだ
馬鹿モノ!
720132人目の素数さん:2001/04/28(土) 03:47
2chからロリコン犯罪者がでました。
小学2年の女の子を誘拐した罪で逮捕されたもよう。

・逮捕された中川泰利くん(22歳)
http://mentai.2ch.net/test/read.cgi?bbs=otaku&key=987849405&ls=100
721132人目の素数さん:2001/04/28(土) 11:52
>>715-716さん
お答えありがとうございます。
(2) C から A への単射が存在しないこと。
これをどう示すのかがよく分かりませんでした。
↓これですね。
>もし、C から A への単射が存在するなら、A から B への単射と合成することで、
>C から B への単射ができてしまうぞ。
やっと納得できました。

ところで、
CからBへの単射が存在しないから、CからAへの単射も存在しない。
これでは、省略しすぎですか?
722132人目の素数さん :2001/04/28(土) 15:02
α_{i} (1≦i≦m) が代数的独立ならば、各α_{i}はいずれも超越数
ってのは自明なことでしょうか?
723132人目の素数さん:2001/04/28(土) 15:16
>>722
だって、超越数じゃなかったら代数的数だから
適当な代数方程式の根になっちゃって、代数的独立にならないじゃん
724132人目の素数さん:2001/04/28(土) 17:24
質問です。
 天秤の左右どちらでもいいので,分銅をのせて重さをはかる。
 分銅は,1,3,3~2....3~n-1 (重さはすべてg)が1個ずつある。
 このとき,Xgのものは,n個の分銅を用いて計測できる。
 しかもこの時分銅の乗せ方はただ一通りに決まる。
どうやって解いたらよいのでしょう?


725132人目の素数さん:2001/04/28(土) 17:25
>>724
分銅は,1,3,3^2....3^n-1
が正しいです。


726132人目の素数さん:2001/04/28(土) 18:02
最大公約数に関する問題
@ ad-bc=±1の時に,
  GCD(x,y)=GCD(ax+by,cx+dy)
なることを示せ
A ax+by=GCD(a,b)
を満たす整数x,yの求める方法をのべよ
B α∈Q α≠0 α=n/m=u/v
  n,m,u,v∈Q m>0 u>0 GCD(m,n)=1 GCD(u,v)=1
  ならば,m=u かつ n=v なることを示せ

よろしく頼みます  
  
727132人目の素数さん:2001/04/28(土) 18:57
>>724
Xを3進法で表記する。ただし各位で使う数字は0、1、2でなくて-1、0、1を使えば
左右どちらの皿に乗せたらいいかも一意に決まる。
728最初のほうだけ:2001/04/28(土) 19:02
>>726
以下,文字は全て整数を表わすものとする.
s=ax+by,
t=cx+dy
とおく.すると
x,yの公約数は明らかにs,tの公約数である.
ところでad−bc=1とすると
x= ds−bt,
y=−cs+at
だから
s,tの公約数は明らかにx,yの公約数である.
よってad−bc=1のとき
{x,yの公約数}={s,tの公約数}
である.
729132人目の素数さん:2001/04/28(土) 19:19
すいません、金沢大学のこの問題と類題を質問させてください
【金沢大学】
(1)
整数n≧3に対して(n)C(3)=Σ(k=3→n){(k-1)C(2)}を示せ
また整数k≧3に対してx+y+z=kを満たす自然数x.y.zの組
(x.y.z)の個数は{1/2(k-1)(k-2)}であることを示せ

(2)整数m≧0に対して、x+y+z≦mを満たす負でない整数x.y.zの
組(x.y.z)の個数を(1)を用いて求めよ

【類題】
次の式を満たす整数x.y.zの組(x.y.z)の総数を求めよ
x+y+z=10
x≧3、y≧2、z≧-1
730132人目の素数さん:2001/04/28(土) 22:21
(1)nCk=n-1Ck+n-1Ck-1 を繰り返し使う
(2)3Hm
(3)適当に変数変換して(2)に帰着させる
731132人目の素数さん:2001/04/28(土) 22:59
>>729,>>730
でもなんだろこの問題。(1)は(2)の誘導みたいだけどこの方針で
やると>>730の方針より大変だよね。miss leadってかんじ?
(1)も思い切って iCi+(i+1)Ci+...+nCi=(n+1)C(i+1) を示せ。
のほうが簡単っぽくかんじるゾ。
732132人目の素数さん:2001/04/28(土) 23:58
>>726

(3)

n/m=u/vより、n*v=m*u

m*uはnで割り切れる(n*v=m*uだから)。

nとmは互いに素だから、uがnで割り切れる。・・・(1)

n*v=m*uだから、n*vはuで割り切れる。

uとvは互いに素だから、nがuで割り切れる。・・・(2)

(1)と(2)より、n=uが言える。

また、n*v=m*uとn=uより、m=vも言える。
733132人目の素数さん:2001/04/29(日) 00:37
>>729-731
誘導に従うならこんな感じか?

(1)前者
 >>729 あるいは C(k-1, 2) = (k-1)(k-2)/2 として右辺を計算
(1)後者
 k個の物を一列に並べると、物と物の間は k-1 個
 k-1 個の間から2つ選んで左側の物の個数を x、間を y、右を z とする
 よって C(k-1, 2)
(2)
 a = x+1, b = y+1, c = z+1 とおくと a, b, c ∈ N で a+b+c <= m+3
 求める個数は(1)後者において k = 3 〜 m+3 の和より
 Σ[k=3 -> m+3]C(k-1, 2) = C(m+3, 3)

類題
 a = x-2, b = y-1, c = z+2 とおく
734132人目の素数さん:2001/04/29(日) 00:49
>>733(1)
うん。この方法なら納得いくんだけどどうも誘導はnC2=n(n-1)/2,
nC3=n(n-1)(n-2)と級数の公式婆^2=n(n+1)(2n+1)/6などを
使わせようとしていると思われ。まあ、重複順列とかパスカル三角形
とか知らないと思いつきにくいからこうゆう誘導にしたのかな?
735132人目の素数さん:2001/04/29(日) 01:31
>>723
どうもです。逝ってきます。
736132人目の素数さん:2001/04/29(日) 07:49
質問です。お願します。
x,yに関する連立一次方程式
ax+by=0
cx+dy=0
において、解(x,y)は(0,0)と異なるなら、行列式
|a b|
|c d|
は0ではないみたいですけど、これってどうしてですか?
737132人目の素数さん:2001/04/29(日) 07:58
>>736
0=d(ax+by)-b(cx+dy)=(ad-bc)x

0=c(ax+by)-a(cx+dy)=-(ad-bc)y

xとyは同時に0ではないからad-bcは0でない
738132人目の素数さん:2001/04/29(日) 08:00
???>>736-737
739132人目の素数さん:2001/04/29(日) 08:17
>>737
× xとyは同時に0ではないからad-bcは0でない
○ xとyは同時に0ではないからad-bcは0である

ですね…
740ななし:2001/04/29(日) 09:49
誰かMarcov Chain Monte Carloって中のハイブリッドモンテカルロという方法を調べてるんですけど知りませんか?日本語の参考文献やほむぺがあったら紹介して下さい。
741ななし:2001/04/29(日) 09:49
誰かMarcov Chain Monte Carloって中のハイブリッドモンテカルロという方法を調べてるんですけど知りませんか?日本語の参考文献やほむぺがあったら紹介して下さい。
742ななし:2001/04/29(日) 09:50
誰かMarcov Chain Monte Carloって中のハイブリッドモンテカルロという方法を調べてるんですけど知りませんか?日本語の参考文献やほむぺがあったら紹介して下さい。
743729:2001/04/29(日) 19:22
助言ありがとうございます。
やっと理解できました。

あと一題、東北大学の問題を質問させてください

【東北大】
(1)白い玉を2個、黒い玉2個、全部で4個の玉を円周上に並べる。
このとき、同じ色の玉が隣り合わない確率を求めよ

(2)赤い玉を2個、青い玉2個、黄色い玉を2個、全部で6個の玉を
円周上に並べる。このとき、同じ色の玉が隣り合わない確率を求めよ
744あってるかどうかは知らんよ:2001/04/29(日) 21:24
(1)は、同色の玉を区別して考えると(4-1)!=6通りしかないから、
全部書いてみるのがテットリファースト。
白1白2黒1黒2 白1白2黒2黒1 白2白1黒1黒2 白2白1黒2黒1
白1黒1白2黒2 白1黒2白2黒1
2/6=1/3
(2)上の6通りのそれぞれに、黄色2個を追加した並びを考える。
それぞれの場合に20通り、計120通り。上の4つについては、
白と黒を分断しなくてはならないので。各々2通りの計8通り。
下の2つについては、黄色を2つくっつけて追加するやり方以外を考え
ればよい。各々20-8=12通り、計24通り。32/210=16/105
745132人目の素数さん:2001/04/29(日) 21:26
>>743
余計なことを考えずに数えちゃった方が早いよコレ
(1)は白い玉が並んでるか離れてるかで2通りしかない
(2)は黄色い玉は置いておいて赤と青は(1)の通りしかなくそのどこかに黄色い玉を並べて入れるか
別々に離していれるかを絵を書いて数えて終わり
746132人目の素数さん:2001/04/29(日) 23:56
>>712
普通に計算していけばできるよ。
接点の座標を (c,s) とおく(c^2+s^2=r^2)。
接線は cx+sy=r^2 になり、これと楕円の式から y を消去。
得られた2次方程式の解を α,β とする。接線の傾きを m=-c/s とすると、

[問題の線分の長さ]^2=(1+m^2)*(α-β)^2

これを解と係数の関係を使って計算・整理する(少しだけ面倒)。
t=(ac)^2+(bs)^2 とおくと、次のようになる。

[問題の線分の長さ]^2=(2abr)^2*(t-r^4)/t^2

t で微分。t=2r^4 で極大となるが、もともと t は (br)^2≦t≦(ar)^2 の範囲しか
動けないので場合分けが必要。あとは易しい。
747132人目の素数さん:2001/04/30(月) 00:26
zの共役複素数、Re(z)、|z|は全ての点で複素微分不可能
であることを示せ、という問題なのですけど、zの共役
複素数は簡単なのですが、残りが微分の定義式書いた
だけで止まってしまいます。どなたか教えていただけませんか?
748132人目の素数さん:2001/04/30(月) 00:35
          _
f(z)が正則 ⇔ ∂/∂z(f(z))=0
これ使うと一瞬で終わる
749132人目の素数さん:2001/04/30(月) 00:43
>>747
きまじめにやるならコーシー・リーマンの微分方程式を
満たしていないことをいう。
750132人目の素数さん:2001/04/30(月) 01:15
>>747
>微分の定義式書いただけで止まってしまいます。

止まったところまで書いて
751132人目の素数さん:2001/04/30(月) 01:39
>>726

Aの解答
簡単にする為に、GCD(a,b)=dと置く。

a=d*a' b=d*b' a'とb'は互いに素

d=1のとき、

a個の数、1*b,2*b,・・・,a*bのなかには、aで割った余りが、1になる物が必ず存在する。

証明
もしそうでなければ、 1*b,2*b,・・・,a*bをaで割った余りは、1、・・・、a-1のa-1通り
しかない事になる。

よって、1*b,2*b,・・・,a*bの中に、aで割った余りが等しくなるような

2数i*b、j*bが必ず存在する(ただし、i、jは0<i<j<aを満たす自然数)。

j*b-i*b=(j-i)*bはaで割り切れる(j*bとi*bをaで割った余りは等しいので)。

aとbは互いに素だから、j-iはaで割り切れることになるが、

これは、0<j-i<aに反する。
証明終

1*b,2*b,・・・,a*bのなかで、aで割った余りが1になるものをk*bとする。

k*bをaで割った商をlとする。

k*b=l*a+1 つまり、a*(-l)+b*k=1

よって(-l)、kが求めるx、yである。

d>1のときも、a'とb'で同様に、a'*x'+b'*y'=1を満たす整数x'、y'が見つかるので、

両辺にdを掛ければ、a*x'+b*y'=d

つまり、x'とy'が求める解である。

752132人目の素数さん:2001/04/30(月) 02:09
>>737-739
Thank you!です。
753悩める受験生:2001/04/30(月) 12:12
1/nの和が正の無限大に発散することは証明できたのですが1/nの2乗の和が
収束するということがどうしても証明できません。
ご迷惑をおかけしますが教えてください。よろしくお願いします。
754132人目の素数さん:2001/04/30(月) 13:02
すみません。どうしても (3)の期待値の問題で確率を足しても1にならないのですが、
教えていただけないでしょうか。


@、A、B、Cのカードが各3枚、計12枚のカードが入っている袋の中から3枚のカードを取り出すという試行を考える。
取り出した3枚のカードの数字がすべて異なるか、3枚とも同じであるとき、当たりとする。

(1)3枚のカードの数字がすべて同じで、当たりとなる確率を求めよ。
これは11分の2*10分の1で、55分の2になりました。    

(2)当たりとなる確率を求めよ。
これも、11分の9*10分の6で、55分の27
そして、55分の27+55分の2より、55分の29となりました。

(3)当たりの場合、カードの数字の和を得点とする。はずれの場合、得点は0とする。このとき、この試行で得られる得点の期待値を求めよ。
(1)の解き方と、 (2)の解き方も間違っているのかもしれませんが、この問題でどうしても
すべての確率を足して1になりません。どなたかおしえてください。どうかよろしくお願いします。
755132人目の素数さん:2001/04/30(月) 13:13
この問題で、

  y=sin(30°−2x)+cos2xについて、
(1)y=r sin(2x+α) (r>0、−180°<α≦180°)と表すとき、r、αを求めよ。
この (1)で、式を−2sinx+sin(30°−2x)+1 と変形したのですが、
この先の求め方が分かりません。大変簡単な問題だったらすみませんが、解き方を
教えてくれませんか? お願いします。
756132人目の素数さん:2001/04/30(月) 13:14
>>753
1/xの2乗の積分で上から押さえたまへ
絵を書けばわかる
757132人目の素数さん:2001/04/30(月) 13:24
>>754
(1)は 1/55 だと思われ
758132人目の素数さん:2001/04/30(月) 13:30
こういうやつが分数の掛け算もできない大学生になって逝くのか…
759132人目の素数さん:2001/04/30(月) 13:30
>>755
sin(30゚-2x) の方に加法定理を使うべきだと思われ
760132人目の素数さん:2001/04/30(月) 13:31
数列に関する質問です。

数列{A(n)}は0<A(1)<3、A(n+1)=1+√1+A(n) (n=1,2,3、…)をみたす。

0<A(n)<3、3−A(n+1)<(3−A(n))×1/3であるとき、

なぜ、0<3−A(n)<(3−A(1))×((1/3))^(n−1) となるのですか? 
761132人目の素数さん:2001/04/30(月) 13:33
>>754
(1)が間違ってるのは置いておいても
全ての確率ってどうして計算したの?
得点0なんだからはずれの確率は計算する必要がないし
そうでなくても1−当たりの確率だよね
762132人目の素数さん:2001/04/30(月) 13:45
>>760
0 < A[n] < 3 より 0 < 3-A[n]

また、3-A[n+1] < (1/3) (3-A[n]) より
3-A[n] < (1/3) (3-A[n-1]) < (1/3)^2 (3-A[n-2]) < … < (1/3)^(n-1) (3-A[1])
763132人目の素数さん:2001/04/30(月) 13:48
>>753
756にかぶり負け(打つ
有界なことだけなら・・・

k > 0のとき
k^(-2) < ∫[(k-1),k]dx/(x^2)
Σ[k=2,n]k^(-2) < ∫[2,k]dx/(x^2)
S(n) = Σ[k=1,n]k^(-2) < 1 + ∫[2,k]dx/(x^2) = 2 - 1/n < 2

(π^2)/6=1.5(笑)に収束することは範囲外ですかね?
764高校一年生:2001/04/30(月) 14:07
(a+b)(b+c)(c+a)+abc(因数分解をする)の解き方のコツを教えてください。
765132人目の素数さん:2001/04/30(月) 14:08
>>761
だから、はずれを計算しなかったとしても、当たりの確率
55分の28 に、全ての場合の確率を足してもならないんです。
得点が3になる場合、6になる場合、7になる場合、8になる場合、9になる場合、12になる場合
の確率の和が、55分の28にほど遠いんです。

たとえば、得点が3になる場合の確率は220分の1になりました。どこが間違っているのかわかりませんか?
766132人目の素数さん:2001/04/30(月) 14:16
>>765
全部書いてみれ
767132人目の素数さん:2001/04/30(月) 14:21
>>764
バラしてaでまとめてaの2次方程式として解いてみたまへ
768132人目の素数さん:2001/04/30(月) 14:35
>>765
28/55になったけど、また分数の計算がおかしいんじゃないの?
どこが間違ってるのかはそれだけじゃわからんので
全ての場合の確率を書いてみないと何が間違ってるのかは誰にもわからない
769132人目の素数さん:2001/04/30(月) 14:39
>>753
1/(n^2) < 1/(n(n-1)) = 1/(n-1) - 1/n
使って上から押さえる
770765:2001/04/30(月) 15:15
得点3が220分の1
   6が440分の11
   7が440分の9
   8が440分の9
   9が440分の11
  12が220分の1

ちなみに0は55分の27になりました
771132人目の素数さん:2001/04/30(月) 15:20
>>770
C(12, 3) = 220 だから n/440 (nと440は素) とはならない
計算やり直したまえ
772132人目の素数さん:2001/04/30(月) 20:32
はさみうちの原理を証明してください
773shiketa:2001/04/30(月) 20:59
abc=1のとき
a    b c
------+-------+-------=1の証明の仕方を教えて下さい
ab+a+1 bc+b+1 ca+c+1
774132人目の素数さん:2001/04/30(月) 21:08
すべてのnについて、An≦Cn≦Bn  An、Bnが同一の極限値αに収束する
このとき任意のε>0に対してあるNがあって、n≧Nならば
|α−An|<ε、|Bn−α|<ε
α−ε<An≦Cn≦Bn<α+ε  よって|Cn−α|<ε
775132人目の素数さん:2001/04/30(月) 21:45
>>773

もともとの左辺を A とすると
a/(ab+a+1)=a/(ab+a+abc)=1/(bc+b+1) などから

A=1/(ab+a+1)+1/(bc+b+1)+1/(ca+c+1) …B

これと、 1/(ab+a+1)=abc/(ab+a+abc)=bc/(bc+b+1) などから

B=ab/(ab+a+1)+bc/(bc+b+1)+ca/(ca+c+1) …C

ゆえに A=B=C だが、一方

A+B+C=(a+1+ab)/(ab+a+1)+…=3

よって A=1
776悩める受験生:2001/04/30(月) 22:41
できました。どうもありがとうございました。
777小泉純二郎:2001/04/30(月) 23:49
教えてください。
1,2,4,1,3,5,2,4,6,8,・・20番目に来る数字は何かという問題です。
答えは10ですが、なぜそうなるのか分かりません。ちなみに小学五年生の算数の
問題集に載っていたものです。
778132人目の素数さん:2001/05/01(火) 00:16

2,4
1,3,5
2,4,6,8
1,3,5,7,9
2,4,6,8,10,12
1,3,5,7,9,11,13
・・・・・・・・・・

ってか?(笑)
779132人目の素数さん:2001/05/01(火) 00:16
奇数偶数を小さい物順に並べた群が交互に一個ずつ個数を増やしてならんでるんです。
780132人目の素数さん:2001/05/01(火) 00:20
よこに
781小泉純二郎:2001/05/01(火) 00:23
ありがとうございました。>132人目の素数さん
782132人目の素数さん:2001/05/01(火) 01:26
「132人目の素数さん」だったのか。
「132人目の素敵さん」だとばかり思ってた。
783132人目の素敵さん:2001/05/01(火) 03:26
L⊃K:体、a,b∈L、aとbはK上共役なとき、
f(a)=bとなるK-同型写像f:K(a)→K(b)が存在する。

これを示したいのですが、フツーに準同型と全単射を示せばいいのでしょうか?
もし、もっとよい方法があったら教えてください。お願します。

784726人目の素数さん:2001/05/01(火) 08:31
>>728 >>732 >>751
ありがとうございました。
お礼が遅くなってすいません。
785132人目の素数さん:2001/05/01(火) 10:45
>>783
あなたの“共役”の定義がわからない(“共役”の定義はいっぱいある)
のでなんともいえんけど、かりに

a,b∈L がK上共役:⇔a,b の(K上の)最小多項式 P(T) ∈ K[T]が等しい。

が定義なら次をしめせばよい。

補題
L⊃K:体、a,b∈L、aの(K上の)最小多項式を P(T) とするとき
体同型写像 φ:K[T]/<P(T)> →K(a) でφ(t)=aなるものが存在する。
ただし<P(T)>はP(T)で生成されるK[T]のイデアルで t はその剰余類。

Point はつぎの普遍性(Universality)とよばれる重要な性質をおさえておくこと。

定理(多項式環のUniversality)
R,S を可換環、s∈S を任意の元とするとき準同型写像φ:R[T]→Sでφ(T)=s
なるものが存在する。

定理(剰余環のUniversality)
R,S を可換環、I⊂R をイデアル、φ:R→Sを環準同型でφ(r)=0(∀r∈I)
をみたすものとし、π:R→R/I を自然な射影とするとき、準同型ψ:R/I→S
でφ=ψπとなるものが存在する。

こーゆー補題をおさえとくとあとあとよろしい。(と思う。)
同型性=全単射性はやさしい...ハズ
786 :2001/05/01(火) 12:49
787132人目の素数さん:2001/05/01(火) 13:40
>>785
>定理(多項式環のUniversality)
>R,S を可換環、s∈S を任意の元とするとき準同型写像φ:R[T]→Sでφ(T)=s
>なるものが存在する。

言葉のモンダイなんだけどUniversalityと言ったときには
「一意的に存在する」までは要求しないんですか?
上の定理では「一意的に」までは主張してないですけど。
#上の定理の主張が間違いだと言ってるわけじゃないです。
788132人目の素数さん:2001/05/01(火) 13:50
>>785
すまソまちがえた。

定理(多項式環のUniversality)
R,S を可換環、s∈S を任意の元とするとき準同型写像φ:R[T]→Sでφ(T)=s
なるものが存在する。



定理(多項式環のUniversality)
R,S を可換環、s∈S を任意の元、φ:R→Sを任意の順同型とするとき
準同型写像ψ:R[T]→Sでψ(T)=s、ψ(r)=φ(r)(∀r∈R)なるものが
存在する。

のまちがい。
789132人目の素数さん:2001/05/01(火) 14:15
>>787
ご指摘のトウリ。一意性が必要。すまソ。
790うぎゃ:2001/05/01(火) 21:53
双対空間って?
線形代数の体K上のベクトル空間Vの双対空間V^*って、
Vに計量入れるのとどう違うんでしょうか?
V^* 上の双対基底f_iと V上の基底e_jで f_i(e_j)=\delta_{ij}
ならばVに特別な計量を入れているのと同値な気がするのですが。
V^{**}->Vへは自然な写像とかいうけど、Vの基底からV^*の基底
への写像を適当に定義すれば、結構自然な気がします。
もしかしてdimV\neq dimV^* なんですか?
わかりません。
791132人目の素数さん:2001/05/01(火) 22:09
>>790
有限次元なら適当な同一視の下に同じ。例えば無限数列の空間
のdualを考えてみそ。双対空間の定義はpairingを取って常に
有限確定値を取る事を考えると、V \neq V^*が分かるはず。
792132人目の素数さん:2001/05/01(火) 22:45

[問]直線y=3x+1/2上の点P(p,q)から放物線y=x^2の法線は何本ひけるか調べよ。

おねがいします

793132人目の素数さん:2001/05/01(火) 22:52
すいません、一つ質問させてください

1<|3x-1|<7という不等式を解けという問題です。
学校で当てられ黒板で解答を書いたのですが
y=|3x-1|のグラフとy=1のグラフ、y=7のグラフを書いて
範囲はy=1のグラフとy=7の間なのでグラフより・・・って書いたら先生に駄目といわれました。

絶対値キゴウをはずして不等式として解くか、二乗しろといわれたのですが
グラフからの判断ではなぜいけないのでしょうか?

ちなみに解答はあってました。
よろしくおねがいします
794132人目の素数さん:2001/05/01(火) 22:57
>>790
>Vの基底からV^*の基底
>への写像を適当に定義すれば、結構自然な気がします。

Vに内積が入っていない場合の話?
写像はどう定義するの?
795りりむ:2001/05/01(火) 23:00
>>793
・グラフを書き間違えた
・先生がアホ

のどちらかです。
796132人目の素数さん:2001/05/01(火) 23:01
>>793
>グラフからの判断ではなぜいけないのでしょうか?

いいとおもうけどね。
何故いけないのかはその先生に訊いたほうがいい。
797793:2001/05/01(火) 23:04
先生にその理由を聞いたら
普通、絶対値がついていてグラフなんかかかないから
そんなことしては駄目だ。といわれました

798りりむ:2001/05/01(火) 23:08
>>797
・先生がアホ

に決定しました。
799132人目の素数さん:2001/05/01(火) 23:26
>>797
どうみてもただの馬鹿だなソレ
800793:2001/05/01(火) 23:28
みなさんご意見ありがとうございました。
金輪際、その先生の授業は切ってチャートでもやることにします
801132人目の素数さん:2001/05/01(火) 23:31
>>793
計算式を並べるだけよりグラフを描いて視覚的にとらえたほうが
イメージが湧きやすいし良いと思うよ
先生の言うことなんて気にせず頑張って
ただ、グラフを描くだけじゃなくて
1度ちゃんと計算してみたほうがいいかも
802796:2001/05/01(火) 23:34
>>797
>普通、絶対値がついていてグラフなんかかかないから
>そんなことしては駄目だ。

説明という行為を何か型にハメ過ぎのような気が…

>>800
>金輪際、その先生の授業は切って

そこまでせんでも…
803132人目の素数さん:2001/05/01(火) 23:44
っつーか、高校の授業は受けない方がいいことも多い
804132人目の素数さん:2001/05/01(火) 23:51
授業切らないでいいから、ボケ教師を斬り捨ててください。
805132人目の素数さん:2001/05/02(水) 00:00
>>792
少しは考えたのか?
こういうふうにやってみたけど、ここで詰まってしまった・・・
という感じで質問した方がよさそう。
806132人目の素数さん:2001/05/02(水) 00:04
>>797
まあ、おそらく単元に合った解き方ではないということでしょう。
むろんそれでダメってのはどうかと思うけど。

先生がそういうなら君が大人になって合わせてやれば?
別に反抗するほどのことじゃなし。
807おさかなくわえた名無しさん:2001/05/02(水) 00:15
なんだかもめてます。
どなたか一言お答えを書き込んでやってください。

http://mentai.2ch.net/test/read.cgi?bbs=tax&key=988559106&ls=50
808132人目の素数さん:2001/05/02(水) 00:36
>>807
『試験をパスできる確率が3/100
 その試験を5回受けたとき、
 少なくとも1回パスできる確率はいくらか?』
を揉めてるってこと?
もし、そうだったらこんなのもわからんヤシ等が税理士とかやってるのか。
猫でもできる仕事だな、税理士って(ワラ
809132人目の素数さん:2001/05/02(水) 01:04
よく
「〜プラスアルファ」って言いますが、
このアルファとは何のことですか?
810132人目の素数さん:2001/05/02(水) 01:13
x
811Tree:2001/05/02(水) 01:55
f(x)=x^2 (xが無理数)
   0 (xが有理数)
であるとき、x=0で微分可能か?
という問題の解を教えてください
関数が連続でないんで、微分不可だと思うんですが…
お願いします
812132人目の素数さん:2001/05/02(水) 02:06
>>811
x=0で微分可能です。定義通りに考えればいい。
当然x=0で連続です。
813hint:2001/05/02(水) 02:26
>>811
任意の h(≠0)に対して |f(h)-f(0)|=|f(h)|≦|h^2| が成り立つ
814Tree:2001/05/02(水) 02:49
なる…
わかりました…
どもでした
815132人目の素数さん:2001/05/02(水) 05:51
>>809
確か野球から来てるはず
816うぎゃ:2001/05/02(水) 08:04
>>790
ありがとうございます。有限次元なら同一視できますよね。
無限次元では違うんですか。なるほど。もう少し考えます。
とりあえずお礼まで。

>>794
たとえば Vの基底 e_i を V^*の基底 f_i へマップするようにすれば
V->V^*への線形写像が定義できますよね。その際VにもV^*にも
内積は入ってないです。
817でじこ@数学板:2001/05/02(水) 08:23
>>809
野球で、9回の表を終了した時点で後攻チームがリードしていた場合、
9回の裏の攻撃は行われず、スコアボードの当該欄に「X」が入るにょ。
この文字「X」は、手書きだとつながって「α」に見えなくもないにょ。

 日本に野球が輸入されて間もない頃、無知な人(審判?)が
 試合終了後にスコアボードの「X」を確認、意味が分からず、
 これをαと考え、後攻チームの得点を「○点+α(アルファ)」
 と読み上げた…

というのが「プラスアルファ」の起源だと聞いた覚えがあるにょ。
(古い記憶なので、細かい所で微妙に違うかも。)
818132人目の素数さん:2001/05/02(水) 09:06
>>816
でもVとV^*の同型を与える事とVに内積を入れる事は同値だよ。
819818:2001/05/02(水) 09:12
ここでいう内積とは非退化な双線形形式のことだけど。
820132人目の素数さん:2001/05/02(水) 09:55
>>790
“自然な写像”というのは“感覚的に自然”という意味でなく“自然変換”
(=natural transformation)という意味です。“圏”(category)というものを
勉強すればのってます。
821132人目の素数さん:2001/05/02(水) 10:10
>>810 >>815 >>817
へぇ〜、そおなんですかぁ?
知りませんでした。
ありがとおございます!
822:2001/05/02(水) 10:56
cartesian product (AxB) のboundedness はどうやって証明すればいいのでしょうか?
823三角比:2001/05/02(水) 11:30
variation constantとslopeとtangent rationの関係性を述べよという問題
なんですが、わかりません。どなたか助けてください。
824132人目の素数さん:2001/05/02(水) 12:40
>>823
slopeってdeterminant Dとpolarization H, rank r
として(D,H)/rのことか?
825 :2001/05/02(水) 13:01
826132人目の素数さん:2001/05/02(水) 13:21
以前>>411のような質問をして、>>413のようなお返事をいただいたのですが、
これって証明とかはどうやるのでしょう?

P.S:413サマ、遅れましたが、お返事ありがとうございました。
827地方の学生:2001/05/02(水) 14:58
突然の参入でもうしわけないのですが、
どうしてもわからないところがあるので
よろしければ教えてください。

(f(x)/x)exp(-ax^2) をxで0から∞まで積分すると、
lim a→∞ のとき 答えは f(0)になるらしいのですが、
なぜなのですか?どうしてもわかりません。
ここでf(∞)=0という条件だけ与えられています。
よろしくお願いします。
828132人目の素数さん:2001/05/02(水) 15:44
f(x) = f(0) + g(x)とおいてfの連続性を使え。
829132人目の素数さん:2001/05/02(水) 15:47
>>822は自分自身問題の意味をまるでわかってないだろ。
あまりに条件が不足していて意味不明だぞ。
830132人目の素数さん:2001/05/02(水) 15:52
>>827
とりあえず変数変換したらどお?
u = (a^(1/2))x
831132人目の素数さん:2001/05/02(水) 16:09
>>816
>たとえば Vの基底 e_i を V^*の基底 f_i へマップするようにすれば
>V->V^*への線形写像が定義できますよね。その際VにもV^*にも
>内積は入ってないです。

その方法で得られる同型写像は基底のとりかたに依存します。
832831:2001/05/02(水) 16:11
たとえば dim V = 2 で
{e_1, e_2}を V の基底、{f_1, f_2}をその双対基底とすると
V の別の基底{e_1, e_1 + e_2}に対してはその双対基底が
{f_1 - f_2, f_2}となります。

基底{e_1, e_2}を使って[>>816]の方法で得られる同型写像は
e_1 -> f_1
e_2 -> f_2
であり、基底{e_1, e_1 + e_2}を使って[>>816]の方法で
得られる同型写像は
e_1 -> f_1 - f_2,
e_1 + e_2 -> f_2
となります。このふたつの同型写像は明らかに異なる写像です
(e_1 の行く先が違う)。
「基底のとりかたに依存する」というのはそういう意味です。

V -> V^{**} のほうは V の基底とかに関係なくきまってしまう
同型写像であって上記の V->V^*([>>816]の方法によるもの)とは
異質(?)なものです。
833132人目の素数さん:2001/05/02(水) 16:17
>>822
cartesian productに適当な(適切な)ノルムを定めて、
ノルム空間A,Bの有界性を使う。(←この条件書き忘れ?)
例 ‖(x,y)‖=‖x‖_A+‖y‖_Bとか、
  
‖(x,y)‖=max{‖x‖_A、‖y‖_B}とか。

証明)

A×B のノルムを
‖(x,y)‖=‖x‖_A+‖y‖_Bで定める。(これは確かにノルムとなる)
但し、x∈A、y∈Bで、‖・‖_A、‖・‖_Bは、それぞれA、Bのノルム
A,Bは有界だから、Rを実数全体とすると、
∃a∈R、∃b∈R、∀x∈A、∀y∈B
such that ‖x‖_A≦a、‖y‖_B≦b
∴∀x∈A、∀y∈Bに対し、‖(x,y)‖=‖x‖_A+‖y‖_B≦a+b<∞
従って、A×Bは有界。
834833:2001/05/02(水) 16:25
>>822
上のはノルム空間の場合。
一般の距離空間の場合も、直積集合に適切な距離を定めて、
A,Bの有界性を使う。これは自分でやってみたら?
835833:2001/05/02(水) 16:35
いや待てよ。
ノルムや距離の定め方によって有界でない時もあるんだろうか。
だとすると、問題文に、直積集合のノルムか距離が与えられてるんじゃない?
従って、与えられたA×Bノルムや距離に、A,Bの性質を使って、
有界性を示せば良い。
836132人目の素数さん:2001/05/02(水) 16:53
質問です。なぜこの辺の人たちは単なる直積の事を
Cartesian Productと呼ぶのですか?
837833:2001/05/02(水) 17:01
>>836
822がcartesian productと呼んだから
838名無しゲノムのクローンさん:2001/05/02(水) 17:03
>>836
集合の本でも読んだら?
Set Theoryとか
839833:2001/05/02(水) 17:07
>>838
彼(>>836)は、知ってるでしょう。
ただ、直積と呼べばすむ話なのになぜわざわざcartesian productと
呼ぶのかというところに疑問を持ったのでしょう。多分
840827:2001/05/02(水) 17:24
突然の参入でもうしわけないのですが、
どうしてもわからないところがあるので
よろしければ教えてください。

(f(x)/x)exp(-ax^2) をxで0から∞まで積分すると、
lim a→∞ のとき 答えは f(0)になるらしいのですが、
なぜなのですか?どうしてもわかりません。
ここでf(∞)=0という条件だけ与えられています。
よろしくお願いします。

>828 830さん
それでもよくわかりません。
方針を教えてください。お願いします。
841↑式が変:2001/05/02(水) 17:58
842132人目の素数さん:2001/05/02(水) 18:00
>>836
>質問です。なぜこの辺の人たちは単なる直積の事を
>Cartesian Productと呼ぶのですか?

デカルトのことが好きなんでしょう、きっと。
843132人目の素数さん:2001/05/02(水) 19:35
4つの正の整数A、B、C、D があり、以下の三つの関係が成立しているとき、
それぞれの値はいくらか?
 3A=B+9
 3B−4C=2
 D+2B=15

おしえてください。。。。
844おさかなくわえた名無しさん:2001/05/02(水) 19:40
彼の父は、私の母の義母です。
私と彼の関係は?

(関係の概念は数学にもあります)
845132人目の素数さん:2001/05/02(水) 19:48
>>843
3A=B+9よりBは3の倍数。
3B−4C=2よりBは2の倍数。
D+2B=15より1≦B≦7。
よってB=6。あとは略。
846132人目の素数さん:2001/05/02(水) 20:48
△ABCに対して、その外側に3つの正三角形ABP、BCQ、CARをつくる。
これら3つの正三角形の重心をそれぞれG,H,Iとおくとき、
△GHIは正三角形であることを示せ。

この問題を初頭幾何で解くにはどうすればよいでしょうか。
847132人目の素数さん:2001/05/02(水) 21:29
>>844
セックスフレンド
848132人目の素数さん:2001/05/02(水) 21:39
任意の自然数 n について,{n, n+1, ..., 2n} の中に必ず平方数(ある自然数の 2 乗になる数)が存在することを証明してください
849hint:2001/05/02(水) 22:06
>>848

「任意のnについてn≦k^2≦2nを満たす整数kが存在」

「任意のnについて√n≦k≦√(2n)を満たす整数kが存在」

この整数kの存在を示すには√n,√(2n)がどのような条件を
満たしていればいいか、というあたりから考えてみよう。
850132人目の素数さん:2001/05/02(水) 23:16
>>840
「0で微分可能」の条件もいるだろ
851132人目の素数さん:2001/05/02(水) 23:53
先月から始まった院の授業でこんな問題が出ました。

次の関数を利用すれば
y=1−e^−μx、x≧0
区間[0,1]に属する点yと[0,∞]に属する点xとが一対一対応する事を示せ
ただしμは正定数とする
さらにyが区間[0,1]において一様分布するならが
対応するxが指数分布することを説明せよ
但し一般に一様分布と指数分布の分布関数は
それぞれ次ぎの様になる
P(X≦x)=0(x≦0),=x(0<x≦1),=1(x>1)
P(X≦x)=0(x<0),=1−e^-Cx(x≧1)
上式中Cは正定数である

前半はわかるけど後半はサッパリです
学部でこんな勉強してません
なんと言うジャンルの学問なのですが?
どなたか教えてください。
852132人目の素敵さん:2001/05/03(木) 00:02
>>785-789
ありがとうございました。共役にも色々あるとは・・・。
ちなみに、答えてくださった定義が私の聞いていたやつの定義と同じでした。
853132人目の素敵さん:2001/05/03(木) 00:02
>>785-789
ありがとうございました。共役にも色々あるとは・・・。
ちなみに、答えてくださった定義が私の聞いていたやつの定義と同じでした。
854132人目の素数さん:2001/05/03(木) 00:17
どうすればよいでしょうか?

f(x)は微分可能で、かつ定数r∈[0,1)が存在して、
任意のxについて|f'(x)|≦r が成り立つとする。
このとき、方程式f(x)=x はただ1つの解をもつことを示せ。
855132人目の素数さん:2001/05/03(木) 00:46
>>851
Yの分布が[0,1]上の一様分布、XがY=1-exp(-cX)を満たすならば、
P(X≦x)=P(Y≦1-exp(-cx))=1-exp(-cx) ,(x>0)

>なんと言うジャンルの学問なのですが?
…どこの院だ?
856132人目の素数さん:2001/05/03(木) 00:47
あのー問題じゃないんですが、「アッコにおまかせ」の難しい問題を解く奴はどうなったんでしょうか?
857132人目の素数さん:2001/05/03(木) 00:54
>>854
g(x)=x-f(x) とおく。次の2つを示す。
(1) 平均値の定理から g(x)-g(0)=g'(c)x 、これを使って lim[x→±∞]g(x)=±∞ をいう
(2) g(x) は単調増加
858132人目の素数さん:2001/05/03(木) 00:55
>829
ごめんなさい
>833,834,835
ちゃんと問題を説明していないのに証明を書いていただいてしまって申し訳ありませんでした。
>836
日本語で何と呼ぶのかしりませんでした

859132人目の素数さん:2001/05/03(木) 01:06
>>854
-r≦f'(t)≦+r を0からxまで積分して -rx≦f(x)-f(0)≦rx(∀t)。
よって -(r+1)x+f(0)≦f(x)-x≦(r-1)x+f(0)。これから
lim[x→+∞](f(x)-x)=-∞とlim[x→-∞](f(x)-x)=+∞をいいたまへ。
860132人目の素数さん:2001/05/03(木) 01:11
>>857
レスかぶりましたな。
>>859
∀tは∀xのまちがいであったようじゃ。あいすまぬ。
861132人目の素数さん:2001/05/03(木) 01:12
862132人目の素数さん:2001/05/03(木) 01:19
>>849
数学的帰納法を使うだけではだめなんでしょうか?
863854:2001/05/03(木) 01:32
>>857さん
レスありがとうございます。

(2)は簡単ですね。
(1)はこんなふうでいいでしょうか?
(x>0の場合)
cの値は、xに依存するが、
仮定よりg'(c)=1-f'(c)≧1-r>0なので、xによらず
g(x)-g(0)≧(1-r)x ・・・(♪)
となり、x→∞のとき(右辺)→∞ (∵1-r>0)なので、
左辺も(よってg(x)も)→∞となる。
(x<0のときは(♪)の不等号を「≦」として同様に考える)


ということは、元の仮定の「|f'(x)|≦r 」で、
“絶対値”はいらないんでしょうか?
864854:2001/05/03(木) 01:34
>>859さんもレスありがとうございます。
865279:2001/05/03(木) 01:35
>>844
父か父方のおじ

こんなんも誰も解けなかったとは ウフ
866132人目の素数さん:2001/05/03(木) 01:35
>>844
>彼の父は、私の母の義母です。

問.彼の父はいつ性転換したでしょう?
867132人目の素数さん:2001/05/03(木) 01:37
>>855さん
851です
何処の院かは詳しくいえませんが
授業は通信系の教科です。
通信に必要な学問らしいです。
868857:2001/05/03(木) 02:01
>>863=854
>“絶対値”はいらないんでしょうか?
不要だよね。ひょっとして出題者が、859さんと同じ勘違いをしていたりして。

>>859
一箇所ミスっています。
x<0 のときは、積分の上端・下端の大小関係が逆になるため、
-rx≦f(x)-f(0)≦rx
では不等号の向きが逆ですよ。
869132人目の素数さん:2001/05/03(木) 02:33
>>868
あらほんと。すまソ
870855:2001/05/03(木) 03:19
>>867
数学専攻じゃなかったのね。すまそ。
>>851の後半は確率論の基礎的な問題だな。
「確率変数の分布」という概念が解れば>>855で納得できるはず。
871132人目の素数さん:2001/05/03(木) 03:58
他スレじゃ誰もやってくれないので
あえてここへカキコ!

「x,y,zを複素数(またはベクトル)とするとき
|x+y|+|y+z|+|z+x|≦|x|+|y|+|z|+|x+y+z|
が成り立つことを示せ。」
872132人目の素数さん:2001/05/03(木) 04:25
三角不等式か
873shima:2001/05/03(木) 06:33
x=y=zなら
左辺−右辺=(|x|+|x|+|x|+|x+x+x|)-(|x+x|+|x+x|+|x+x|)≧0
x=yなら
(左辺-2|x|)^2−(右辺-2|x|)^2
=(|z|+|x+x+z|)^2-(|x+z|+|x+z|)^2
=z^2+(2x+z)^2+2|z||x+x+z|-4(x+z)^2
=2(|z||2x+z|-z(2x+z))≧0
よってx<y<zとして考える
(3数とも同符合のばあい)
0≦xなら
絶対値記号がそのまま外れるから成り立つ
(1つだけ異符号のばあい)
-y≦x<0<yなら
左辺−右辺=(-x+y+z+(x+y+z))-((x+y)+(y+z)+(z+x))≧-2x≧0
-z≦x<-y<0<yなら
左辺−右辺=(-x+y+z+(x+y+z))-((-x-y)+(y+z)+(z+x))≧2y≧0
-y-z≦x<-z<0<yなら
左辺−右辺=(-x+y+z+(x+y+z))-((-x-y)+(y+z)+(-z-x))≧2x+2y+2z≧0
x<-y-z<0<yなら
左辺−右辺=(-x+y+z+(-x-y-z))-((-x-y)+(y+z)+(-z-x))≧0
q.e.d
でどうでしょうか
874132人目の素数さん:2001/05/03(木) 06:50
875132人目の素数さん:2001/05/03(木) 07:00
>>873
>x,y,zを複素数(またはベクトル)とするとき

実数ではありませぬ。要再考
876うぎゃ:2001/05/03(木) 07:39
>>832
それは理解しました。ありがとうございました。
V^**->Vへの写像は定義しやすいんですね。
圏とかはさすがにしらないので良くわかりません。
群なんかでも自然な射影とかありますけど、おそらく
同じ概念なんでしょう。群で線形代数は語れますしね...

とりあえず一致しない例を考えてます...
頭悪いのでもちっと待ってください。

877132人目の素数さん:2001/05/03(木) 07:49
>>870さん
851です
ありがとうございます
あとは自力でなんとかしてみます!
878123番目の素数ちゃん:2001/05/03(木) 07:54
素朴な疑問です。
なぜ「『132』番目の素数さん」なの?
880訂正:132人目→132番目:2001/05/03(木) 08:01
881でじこ@数学板:2001/05/03(木) 08:03
>>878
小さい方から順に数えて132番目の素数は 743(ななしさん)なのです。
882132人目の素数さん:2001/05/03(木) 09:16
>>873
オイオイ…
883132人目の素数さん:2001/05/03(木) 13:00
>>862
この問題でどうやって帰納法を使うの?
884誤爆:2001/05/03(木) 13:30

6 名前: shima 投稿日: 2001/05/03(木) 06:50

ありゃ〜ベクトルだった^^;
ごめんなさい
885(@。@) 丸三証券:2001/05/03(木) 14:11
A〜Fの6人が総当り式でブタミントンの試合をした。
結果はACEがそれぞれ4勝でFは2勝、引き分けはなかった。
このことから確実に言えるのはどれか。
 1)CはEに勝ち、Aに破れた。
 2)EはAに勝ち、Bに破れた。
 3)FはBに勝ち、Cに破れた。
 4)DはFに勝ち、Eに破れた。
 5)BはDに勝ち、Fに破れた。

あてずぽうじゃなく、わかる奴いる?
886132人目の素数さん:2001/05/03(木) 14:55
>>885

6人による総当たり戦なので全部で15試合行われる。
引き分けがないので、全員の勝ち数の合計は15。
いまACEFの4人の勝ち数合計は14あるので、
残るBDのうち1人は「1勝」もう1人は「0勝」である。
ここでBが「1勝」として一般性を失わなず、また
BはDにのみ勝ったことになる。

よってFの「2勝」はBとDに対してのものであることがわかり、
ゆえに選択肢3)が確実にいえる。
887(@。@) 丸三証券:2001/05/03(木) 14:58
あんたすげぇーよ。
なんでわかるん?
888132人目の素数さん:2001/05/03(木) 15:33
15試合あるってわかったら、
あとは思考すればいいじゃん
889あつし:2001/05/03(木) 15:51
あのっ!! この問題の解き方忘れちゃったんですが、初めに
例えば f(2)≦0 とかって条件出して解きますよね?
それをこの問題の場合どうするのかおしえてください。お願いします。

二次関数 f(x)=x^2−x−2
       g(x)=x^2+2ax+3a+4 について、
f(x)≦0を満たすxの範囲と、g(x)≦0 を満たすxの範囲とが、共通部分をもつようなaの値の範囲を求めよ。
890132人目の素数さん:2001/05/03(木) 16:03
=0の定義域同じなんだから当然解を共有してるわな。
891-,-)/しつも〜〜ん:2001/05/03(木) 17:24
x^xの逆関数はなんですか?
892132人目の素数さん:2001/05/03(木) 17:24
>>889
f(x)=(x−2)(x+1)≦0
⇔-1≦x≦2

g(x)≦0 も同じく範囲を考えて
重なるようにすれ
893132人目の素数さん:2001/05/03(木) 17:29
>>891
知ってる関数の組み合わせでは書けないと思うよ
894KKK:2001/05/03(木) 19:57
えと、自分の体験談からなんですけど
今、MIUっていう海洋深層水の飲み物でボトルキャップの
キャンペーンをやっているんですけど。
それで、どうしても全9種類集めたかったんで、MIUを合計で
20本買って8種類は出ました。(最後の一種類は友達から
もらっちゃいました)

そこで、自分が運いいか、悪いか知りたいんで、
質問なんですけど20本買って全9種類集まる確率って
どれくらいになるんですか?自分でもやってみたんだけど
できませんでした。
あと、これをちょうどn回目で全k種類集まるって一般化
もできるんでしょうか?

長々と書いてすいません。もし、よろしければ御協力お願いします。
895-,-)/しつも〜〜ん:2001/05/03(木) 20:12
>>891
なるほど。やっぱそうなんですか。
どうもでした。
896132人目の素数さん:2001/05/03(木) 23:12
>>862
n=k での成立を仮定する。
すなわち、{k,k+1,…,2k} の中に平方数があるとする。その平方数を s^2 とおく。

n=k+1 での成立を示す。

(a) k≠s^2 のとき。
このときは、s^2 が {k+1,k+2,…,2(k+1)} にも含まれている。題意成立。

(b) k=s^2 のとき。
このときは、(s+1)^2 が {k+1,k+2,…,2(k+1)} に含まれていることをいう。
そのためには、k+1≦(s+1)^2≦2(k+1) をいえばよい。
k=s^2 なので、これは s^2+1≦(s+1)^2≦2(s^2+1) と同値。この不等式の成立は易しい。
897132人目の素数さん:2001/05/03(木) 23:15
>>846
A を中心に、△APC を (√(1/3)倍に縮小し、反時計回りに 30°回転すると △AGI になる。

B を中心に、△BPC を √(1/3)倍に縮小し、時計回りに 30°回転すると △BGH になる。

∴ GI=GH
898846:2001/05/04(金) 00:25
>>897

おおっ、なるほど!
わかりました、有難うございました。
899132人目の素数さん:2001/05/04(金) 00:28
>>894
一般化したやつは、

(1/k)^n*[n個からk個への全射の総数]

になるはず。で、これは、

(1/k)^n*Σ[i=1,k](-1)^(k-i)*kCi*i^n

になる。(>>527 >>535 >>569 >>575 を参照)
n=20, k=9 の計算は、ご自分で。
900132人目の素数さん:2001/05/04(金) 01:16
>>891 893
グラフ書けるのに関数が解からないとは不思議。
901132人目の素数さん:2001/05/04(金) 07:21
これやって見られ

問1
「複素数体C上の3変数有理関数体C(X_1,X_2X_3)に
3次交代群A_3が変数の添数の置換
        X_i→X_σ(i)
で作用しているとする。有理関数体C(X_1,X_2X_3)のA_3の作用に
ついての不変元の全体C(X_1,X_2X_3)^(A_3)はC上3個の元で生成されることを示せ。」

問2
「離散付値環Rとその素元tについて
R[X]/{(X^n -t)R[X]}は離散付値環になることを示せ」
902132人目の素数さん:2001/05/04(金) 07:45
それ院試の過去問でしょ。自分で解く努力をしてみた?
903:2001/05/04(金) 08:57
a[1] = a , a[2] = b , a[3] = c
6a[n+3] = 3a[n+2] + 2a[n+1] + a[n] (n >= 1)
一般項を求めよ

よろしくお願いします。
904132人目の素数さん:2001/05/04(金) 10:00
特性方程式 t^3=3t^2+2t+1 の相異なる根を t=α,β,γ として

a[n]=pα^(n-1)+qβ^(n-1)+rγ^(n-1)
p,q,r は定数とおく。

a[1]=p+q+r=a
a[2]=pα+qβ+rγ=b
a[3]=pα^2+qβ^2+rγ^2=c

から p,q,r を求める。以上。
905のぞみ:2001/05/04(金) 12:01
n種類の文字が2個ずつ(計2n個)ある。
これらを一列に並べる時、
同じ文字が連続して並ばないような並べ方の総数をa(n)とおく。

たとえばa(1)=0 , a(2)=2 , a(3)=30 となりますが、
このa(n)を一般に求めることはできますか?
あるいは、a(n)の漸化式でもわかれば嬉しいのですが・・・
906(@。@) 丸三証券:2001/05/04(金) 13:03
正多面体の面の数と頂点の数と辺の数って、なんか法則性ある?

   面の数  頂点の数 辺の数
  正四面体    4     4
  正六面体    8     12
  正八面体    6     12
 正十二面体   20    30
 正二十面体  12    30

ありそうなんだけどわからない。
907132人目の素数さん:2001/05/04(金) 13:06
>>905
とりあえず
a(n)=納j=0,n-1](2j+1)P(n,n-j)a(j)-na(n-1)
がとれますね。(ただしa(0)=1としておく。P(n,i)は順列のかず
P(n,i)=n・(n-1)・...・(n-i+1))
もうすこし簡単になるとは思いますけど。きっと誰かがもっと
簡単にしてくれるでしょう。...(^_^;)
908-,-)/しつも〜〜ん:2001/05/04(金) 13:06
>>900
どゆこと??
909132人目の素数さん:2001/05/04(金) 13:09
>>906
点-線+面=2...(Euler標数ってかんじ。)
910132人目の素数さん:2001/05/04(金) 13:15
正6面体と正8面体は双対。
正12面体と正20面体は双対。
正4面体は自己双対。
911のぞみ:2001/05/04(金) 13:31
>>907

・・・この式、どうやって導くんですか?
912132人目の素数さん:2001/05/04(金) 13:58
>>908
f(x)=x^xのグラフを書いてXY軸を逆にするという意味だと思われ。
913132人目のさくらたん:2001/05/04(金) 14:07
900を超えたので新スレに移転しました.

「わからない問題はここに書いてね・7」
http://cheese.2ch.net/test/read.cgi?bbs=math&key=988952592
914132人目のさくらたん:2001/05/04(金) 14:07

┌──────────────────────―─―┐
│                                      |
│                                      |
│                                      |
│                                      |
│                ∧_∧                      |
│               ( ・∀・)                |
│               (    )                 |
│               | | |                     |
│               (__)_)                  |
|              2ちゃんねる               |
│                                      |
│            スレッドを終了しています…           |
│                                      |
│                                      |
|                                      |
|                                      |
└───────────────────────――┘
915639:2001/05/07(月) 18:01
>>650
分解したりしてみましたがどうもしっくりくるのがありません。
こういうのの一般式はどいう表し方をするんですか?
916639:2001/05/07(月) 18:01
>>650
分解したりしてみましたがどうもしっくりくるのがありません。
こういうのの一般式はどういう表し方をするんですか?
917あゆ:2001/05/07(月) 23:36
正の整数値をとる確率変数Xの
確率密度関数f(x)が次の式で与えられるとき
f(x)=(c・3^x)/x! (x=1,2,3,,,)
(1)定数Cの値を求めてください。
(2)P(X=3)を求めて下さい。
(3)P(X<2)を求めて下さい。
918132人目の素数さん:2001/05/08(火) 02:07
だからもうこのスレは移転してるんだってば。
http://cheese.2ch.net/test/read.cgi?bbs=math&key=988952592
*****************************************************
******************* T H E   E N D ********************
*****************************************************
919T・I:2001/06/11(月) 00:58
X-2y=3と2x+y=0を連立方程式でといたらなんでX=3/5、y=-6/5になるんですか?
普通はそうならないとおもうんですけど、私の解き方がまちがっているんでしょうか?
連立方程式でとくものではないとか?
*チャート式解法と演習BEST数学U+BのPRACTICE3の(1)の問題
920T・I:2001/06/11(月) 01:01
移行してたのかー!リロードせんかったからわからなかった。
スレを上げてしまってすみません
921ぶくぶく:2001/06/11(月) 01:27
>>76
>∞は実数ではありません。そもそも数ではありません。

チラッと見て気になったので、古い話にレス。
∞を実数とするかどうかは選択公理だよん。
ようは、どっちでも無矛盾な公理系できるってこと。

そうゆう根本的でアレな話は工学屋でなくて
数学基礎論の人に聞こう。
922132人目の素数さん:2001/06/11(月) 03:01
↑ トンデモ発見
923132人目の素数さん:2001/06/11(月) 05:41
ド素人が啓蒙書を読んで知ったかぶると921のようになります。
924嵐勘:2001/06/28(木) 19:48
age
925☆スレッド終了のお知らせ☆
■■■■■■■■■■■■■■■■■■■■■■
 このスレは終了したので書かないで下さい
■■■■■■■■■■■■■■■■■■■■■■